UWorld Microbiology and Antimicrobials

Réussis tes devoirs et examens dès maintenant avec Quizwiz!

A group of researchers collected demographic and clinical data from multiple hospitals across the nation to determine epidemiological features of nosocomial bloodstream infections. In this study, a nosocomial bloodstream infection was diagnosed if blood cultures drawn 48 hours after admission yielded a pathogenic organism. Analysis of the data collected over several decades shows that staphylococci are increasingly responsible for the identified cases. An increased use of which of the following is most likely the underlying cause of the observed trend? Bioprosthetic heart valves Broad-spectrum antibiotics Illicit injection drugs Immunomodulating therapies Indwelling urinary catheters Intravascular catheters

Approximately 250,000 nosocomial bloodstream infections (BSIs) are estimated to occur in the United States each year. Prior to the 1980s, the leading cause was gram-negative aerobic organisms such as Escherichia coli that spread from primary infections in the genitourinary or gastrointestinal tract. However, the increasing use of intravascular catheters, which provide a portal of entry for skin flora to reach the bloodstream, has led to a shift in the underlying microbiology of BSI; most cases are now caused by skin commensals such as coagulase-negative staphylococci and Staphylococcus aureus. BSI is most common in those who have long-term (>12 days) central venous access as this provides sufficient time for skin flora to colonize the internal/external lumen of the catheter, to replicate within a biofilm of host (fibrin) and bacterial (glycoprotein) components, and to subsequently spread to the bloodstream. BSI should be suspected when a patient with an intravascular catheter develops new-onset fever or bacteremia. (Choice A) Although S aureus is a leading cause of infective endocarditis in patients with artificial heart valves, bacteremia is much more commonly caused by intravascular devices. (Choice B) The increased use of broad-spectrum antibiotics is associated with the development of drug-resistant organisms, including methicillin-resistant S aureus (MRSA). Although MRSA is a common cause of skin abscess and necrotizing pneumonia, the majority of S aureus bacteremia is caused by intravascular catheters. (Choice C) Injection drug use is a major risk factor for S aureus abscess, bacteremia, and infective endocarditis. However, injection drug use causes far fewer BSIs than intravenous catheters. (Choice D) Immunosuppression increases the risk of BSI due to reduced immunosurveillance. However, BSI is far more often due to the use of intravascular catheters. (Choice E) Indwelling urinary catheters increase the risk of urinary infection with subsequent hematogenous spread of the organism. However, most cases are caused by gram-negative enteric organisms such as E coli. Educational objective: Nosocomial bloodstream infections are usually associated with intravascular catheters, which allow skin commensals such as Staphylococcus aureus and coagulase-negative staphylococci to enter the bloodstream.

A 38-year-old man is undergoing treatment for acute myeloid leukemia. Initially, he had fevers and reported being lethargic and fatigued all the time. However, of late, he has noted increasingly severe right-sided headaches. He describes the pain to be throbbing in nature and located behind his right eye. He also has associated nasal stuffiness. The patient has no prior history of allergies. Physical examination reveals right-sided proptosis and periorbital tenderness. Biopsy of the right maxillary sinus mucosa reveals the following: Image shows non-septate, right angle yeast (ID=106) Which of the following is the most likely cause of this patient's condition? Malassezia furfur Microsporum canus Rhizopus species Aspergillus fumigatus Candida albicans Cryptococcus neoformans Blastomyces dermatitidis Histoplasma capsulatum Coccidioides immitis Sporothrix schenckii

Mucor, Rhizopus, and Absidia species are saprophytic fungi present in the environment. They are transmitted by spore inhalation and cause mucormycosis. Mucormycosis is very strongly associated with diabetic ketoacidosis. Patients with underlying immunosuppression (eg, due to solid organ transplantation, hematologic malignancies, or glucocorticoid therapy) are also at high risk. Mucormycosis tends to affect the paranasal sinuses. Symptoms include facial and periorbital pain, headache, and purulent nasal discharge. The fungi proliferate in the walls of blood vessels and cause necrosis of the corresponding tissue. Black eschar (necrotic tissue) may be seen on the palate or nasal turbinates. Mucormycosis is diagnosed by light microscopy of a tissue specimen. Mucor, Rhizopus, and Absidia fungi exist in mold form only. They form broad, nonseptate hyphae that branch at wide, often 90° angles. Mucormycosis is treated by surgical debridement of necrotic tissue and amphotericin B. (Choices A and B) Malassezia furfur and Microsporum canis cause cutaneous mycoses but do not affect the paranasal sinuses. (Choice D) Invasive aspergillosis involving the paranasal sinuses can show close clinical resemblance to mucormycosis. However, light microscopy of the affected tissue will differentiate these two diagnoses: unlike Mucor and Rhizopus, Aspergillus has septate hyphae that branch at 45° angles (V-shaped branching). (Choice E) Candida albicans can cause a variety of symptoms in immunosuppressed patients. Right-sided endocarditis, renal abscesses, and esophagitis are among the most common. Budding yeasts and pseudohyphae are seen on light microscopy. (Choice F) Cryptococcus neoformans primarily causes meningoencephalitis (eg, headache, confusion, altered mental state) in patients who are immunocompromised (particularly those with advanced AIDS). (Choices G, H, and I) Blastomyces, Histoplasma, and Coccidioides cause lung disease in healthy patients. Disseminated disease occurs in immunosuppressed individuals. Isolated paranasal involvement is not typical. (Choice J) Sporothrix schenckii can be transmitted to humans via subcutaneous inoculation. Manifestations include a papule at the site of exposure with subsequent papules forming along proximal lymphatic drainage pathways. Educational objective: Mucormycosis is an opportunistic infection caused by Rhizopus, Mucor, and Absidia species. The classic clinical picture is paranasal sinus involvement in a diabetic or immunosuppressed patient. The fungi form broad nonseptate hyphae that branch at right angles.

A 36-year-old man comes to the office to discuss a skin rash on his shoulders and upper chest. The patient first noticed the rash 2 months ago while vacationing at a beach resort. The rash has worsened progressively and is associated with mild pruritus, but he has no other associated symptoms. The patient is a road construction worker, and often works in hot, humid areas. Past medical history is notable for seasonal allergies and childhood asthma. He does not use tobacco or alcohol. Skin examination findings are shown in the image below. ID=104, white spots on shoulder Which of the following is the most likely cause of this patient's condition? Blastomyces dermatitidis Candida albicans Histoplasma capsulatum Malassezia globosa Microsoporum canis Rhizopus species Sporothrix schenckii

Pityriasis versicolor (also called tinea versicolor) is characterized by hypopigmented, hyperpigmented, or erythematous macules or patches on the upper body. It is caused by Malassezia species yeasts, and the infection is confined to the stratum corneum of the skin. It is common in areas with hot and humid climates and is often most visible after extensive sun exposure due to tanning of the adjacent skin. Most cases of pityriasis versicolor occur in healthy individuals with no underlying immunologic deficiencies. The diagnosis of pityriasis versicolor is made by potassium hydroxide (KOH) preparation of skin scrapings. Malassezia forms spores and hyphae, producing the characteristic "spaghetti and meatballs" appearance on light microscopy. The hyphae have a short, "cigar-butt" appearance. This condition is treated with topical antifungal agents or selenium-containing shampoo. (Choices A and C) Blastomycosis is characterized by progressive pulmonary infection. Skin involvement is uncommon but can manifest as papular or pustular lesions in exposed areas. Histoplasmosis also most commonly causes pulmonary disease, though it may become disseminated in immunodeficient patients. (Choice B) Intertrigo is characterized by well-defined erythematous plaques with satellite vesicles or pustules in warm, moist skin areas (eg, axillae, groin, skin folds). The infection is usually due to Candida species, and it is most common following antibiotic use or in immunocompromised states (eg, uncontrolled diabetes). (Choice E) Microsporum species are a frequent cause of tinea capitis and tinea corporis. (Choice F) Rhizopus is a major cause of mucormycosis, which is characterized by invasive, necrotic upper and lower respiratory infections in immunocompromised patients. (Choice G) Sporothrix molds are found in decaying horticultural matter and are acquired through breaks in the skin. They cause nodular and ulcerating subcutaneous lesions. Educational objective: Pityriasis versicolor (tinea versicolor) is a superficial skin infection caused by Malassezia species. It causes erythematous, hyper- or hypopigmented macules and patches. Malassezia forms spores and hyphae, producing the characteristic "spaghetti and meatballs" appearance on KOH preparation light microscopy.

A 77-year-old woman is brought to the emergency department due to fever and confusion. The patient has a history of Alzheimer dementia and lives in a nursing home. Temperature is 38.4 C (101.1 F), blood pressure is 104/74 mm Hg, pulse is 110/min, and respirations are 18/min. Suprapubic tenderness is present on physical examination. Urinalysis shows numerous bacteria and leukocytes, and urine culture grows Pseudomonas aeruginosa. Further analysis reveals that the isolated organism produces an enzyme that is located on the cytoplasmic surface of the cell membrane and that catalyzes the transfer of acetyl groups to exogenous substances. The bacteria are most likely resistant to which of the following antibiotics? Ceftazidime Ciprofloxacin Gentamicin Imipenem Piperacillin

Pseudomonas aeruginosa is a gram-negative, opportunistic pathogen with multiple mechanisms of antibiotic resistance, including beta-lactamases, efflux pumps, and antibiotic-modifying enzymes. Antibiotic-modifying enzymes add chemical groups (eg, acetyl, adenyl, phosphate) to a target drug and are the most common mechanisms of aminoglycoside (eg, gentamicin) resistance. Altered aminoglycosides have a reduced ability to bind to the 16S ribosomal RNA within the 30S ribosomal subunit, leading to increased minimum inhibitory concentrations and reduced bactericidal effects. Most antibiotic-modifying enzymes arise via transfer of plasmids or transposons rather than chromosomal mutations. (Choices A, D, and E) Cephalosporins (eg, ceftazidime), carbapenems (eg, imipenem), and penicillins (eg, piperacillin) are beta-lactam drugs that exert their effect by interfering with the cell wall. Resistance to cephalosporins and penicillins occurs largely because of the bacterial secretion of beta-lactamase, an enzyme that hydrolyzes the beta-lactam ring and inactivates the antibiotic. Carbapenems are not very susceptible to most beta-lactamases due to additional chemical components surrounding their beta-lactam ring; however, resistance to carbapenems can occur due to carbapenemase, which is primarily seen in Enterobacter species. (Choice B) Fluoroquinolones (eg, ciprofloxacin) block enzymes (DNA gyrase, topoisomerase) responsible for bacterial DNA synthesis. Resistance usually occurs due to genetic mutations that prevent drug binding to these enzymes or to the increased expression of antibiotic efflux pumps. Educational objective: Aminoglycoside (eg, gentamicin) resistance is most commonly due to antibiotic-modifying enzymes. These enzymes add chemical groups to the antibiotic, which diminishes its ability to bind to the 16S ribosomal RNA within the 30s ribosomal subunit.

A 31-year-old man comes to the office due to oral lesions that cause occasional discomfort during eating. He also reports fatigue and unintentional weight loss. The patient has no prior medical problems and takes no medications. Oral examination findings are shown below. Oral thrush on tongue (ID=111) The lesions can be easily scraped off, revealing erythematous mucosa underneath. There are several enlarged cervical and axillary lymph nodes. Which of the following diagnostic tests is most appropriate during further workup of this patient? HIV antigen and antibody testing Oral lesion biopsy to rule out malignancy PCR for coxsackievirus Scraping microscopy for multinucleated giant cells Serum vitamin B12 and folate levels

Thrush is an oropharyngeal Candida albicans infection that usually presents with creamy white, plaque-like lesions on the buccal mucosa, palate, tongue, or pharynx. The lesions can be easily scraped off with a tongue depressor, revealing underlying erythematous mucosa. The diagnosis is often made clinically, but microscopic examination will reveal budding yeasts with or without pseudohyphae. Candida albicans is a normal commensal of the human digestive tract that can cause mucocutaneous infections when local microbial flora is disrupted (eg, antibiotic use) or the cell-mediated immune response is impaired. Although Candida infections of the skin (eg, diaper rash) and vulvovaginal region often occur in otherwise healthy individuals, oropharyngeal infection is rare in the absence of antibiotics, chemotherapy, or inhaled corticosteroids. Young, healthy individuals who develop thrush and have no clear inciting cause should be tested for HIV, particularly when HIV risk factors (eg, multiple sexual partners) or symptoms of the virus (eg, lymphadenopathy, weight loss) are present. A fourth-generation HIV antigen/antibody test is typically used for screening. (Choice B) Leukoplakia is a benign lesion of the tongue caused by hyperplasia of the squamous mucosa that can evolve into dysplasia → carcinoma in situ → invasive carcinoma. Leukoplakia appears as white patches or plaques on the oral mucosa. However, these lesions cannot be easily removed with scraping (unlike thrush). (Choice C) Coxsackievirus causes herpangina, which is usually seen in infants and children. It is characterized by high fever, sore throat, and yellowish or grey-white papulovesicular lesions on the throat (tongue lesions are rare). Coxsackievirus also causes hand, foot, and mouth disease, which is characterized by tongue macules, vesicles, and ulcers. However, it is also usually associated with a vesicular or macular skin rash. (Choice D) Microscopy of herpes simplex virus (HSV) lesions will reveal multinucleated giant cells. HSV usually causes multiple vesicular lesions on an erythematous base, not white mucosal plaques. (Choice E) Atrophic glossitis can be caused by vitamin B12 or folate deficiency. It is characterized by atrophy of the filiform papillae leading to a smooth, glossy, erythematous tongue. Educational objective: Oral thrush is caused by Candida albicans infection and usually presents as white, plaque-like oropharyngeal lesions that are easily scraped off with a tongue depressor. Affected patients typically have disruption to local microbial flora (eg, antibiotic use) or impairment in cell-mediated immunity (eg, corticosteroids, HIV). Young, otherwise healthy patients with thrush should be tested for HIV.

A 46-year-old man comes to the emergency department due to fever. He returned from a vacation in Southeast Asia a week ago and began having symptoms toward the end of his trip. The patient describes continuous fever that has risen slowly to reach a high plateau and is relieved only by antipyretics. He has also had headaches and abdominal discomfort; he initially had constipation but has had loose stools for the past several days. Temperature is 40 C (104 F), blood pressure is 114/68 mm Hg, and pulse is 62/min. No oropharyngeal lesions, cervical lymphadenopathy, or neck rigidity is present. Lungs are clear on auscultation, and heart sounds are normal. Abdominal examination shows mild distension and generalized tenderness with hepatosplenomegaly. There is a faint, erythematous, macular skin rash over the trunk and abdomen. Laboratory studies reveal mild, normocytic, normochromic anemia and leukopenia with left shift. Peripheral blood smear is normal. Which of the following is the most likely route of transmission of this patient's infection? Bile from a vector insect Exposure to a carrier animal Ingestion of contaminated food Inhalation of infectious droplets Sexual contact with an infected person

Typhoid fever is a life-threatening infection caused by Salmonella Typhi or Paratyphi. These gram-negative, enteric pathogens reside only in the gastrointestinal tract of humans (Choice B). Transmission occurs via the fecal-oral route, primarily due to ingestion of food or water contaminated by human feces. Therefore, most cases arise in the developing world where sanitation is poor. Travelers who have not been vaccinated (with typhoid vaccine) are at risk and may return clinically ill. Typhoidal Salmonella species invade enterocytes, travel from the apical to the basolateral end of the cell in vacuoles, and emerge in the intestinal submucosa. Here, the organism is phagocytosed by macrophages but is able to evade destruction due to a specialized capsular antigen (Vi) that impairs the macrophage-mediated oxidative burst. The pathogen subsequently replicates in an unchecked fashion within macrophages and spreads through the lymphatic and reticuloendothelial system, leading to widespread, systemic illness. Manifestations typically develop over weeks in a step-wise fashion. Patients usually develop a progressive fever, often with relative bradycardia (pulse-temperature dissociation), followed by abdominal pain, salmon-colored macules on the trunk ("rose spots"), and constipation or diarrhea. Dissemination through the reticuloendothelial system often causes hepatosplenomegaly, anemia, and leukopenia. Later, ulceration of Peyer patches (lymphoid aggregations in the small intestine submucosa) can result in gastrointestinal bleeding or intestinal perforation. Antibiotic treatment is required to prevent complications and death. (Choice A) Malaria is transmitted by the bite of the female Anopheles mosquito and typically causes paroxysmal fever with nonspecific symptoms (eg, chills, fatigue, headache, myalgias). However, the fever is almost always associated with tachycardia (not bradycardia), and peripheral blood smear will usually demonstrate the parasite and/or signs of red blood cell lysis. (Choice D) Meningococcal meningitis is transmitted by aerosolized droplets from a carrier or infected patient and is associated with rapid-onset confusion, sepsis, hypotension, and a petechial/purpuric rash. (Choice E) Treponema pallidum, the spirochete that causes syphilis, is transmitted sexually. Secondary syphilis is usually characterized by a widespread maculopapular rash that involves the palms and soles, generalized lymphadenopathy, and constitutional symptoms such as fever, headache, and myalgias. This patient's pulse-temperature dissociation, limited rash, and lack of adenopathy make syphilis less likely than typhoid fever. Educational objective: Typhoid fever is caused by Salmonella Typhi or Paratyphi and presents with escalating fever, followed by abdominal pain, formation of rose spots on the chest/abdomen, and hemorrhagic enteritis with possible bowel perforation. Humans are the only reservoir; transmission is fecal-oral and primarily occurs due to ingestion of food or water contaminated with feces.

A 32-year-old Caucasian male presents with low grade fever, cough and generalized malaise. The patient has otherwise been healthy and has no significant medical history. His past surgical history includes tonsillectomy and tympanostomy tube placement at the age of 8. Careful questioning reveals that the patient recently came back from an "outdoor adventure" trip in the Great Lakes, and began feeling sick only after returning. Microscopic examination of a KOH preparation of an exudate specimen reveals a large yeast cell with a single bud. When incubated in a specific medium at 75.2ºF (24ºC), a multicellular structure with branching, tubular cells grows. The patient is most likely infected with which of the following: Aspergillus fumigatus Blastomyces dermatitidis Candida albicans Cryptococcus neoformans Malassezia furfur Rhizopus species

(Choice A) Aspergillus fumigatus has a mold form only and is not dimorphic. It is seen in biopsy specimens as septate hyphae that form acute angle V-shaped branches. Aspergillus mainly affects immunocompromised patients. (Choice C) Candida albicans is a component of normal human flora. It causes disseminated infections in immunocompromised patients. It is polymorphic, and it is commonly seen as a budding yeast or forming pseudohyphae. True hyphae also occur, usually only when Candida becomes invasive (in early stages, these hyphae can be identified as germ tubes). (Choice D) Cryptococcus neoformans causes meningitis in AIDS patients. This fungus exists in yeast form only. India ink stain of CSF reveals encapsulated yeast. (Choice E) Malassezia furfur causes a cutaneous mycosis (hypopigmented skin patches). KOH preparation of skin scrapings shows short hyphae and spores ("spaghetti and meatballs"). (Choice F) Rhizopus, Mucor, and Absidia are the saprophytic fungi that cause mucormycosis in immunosuppressed patients. Mold forms of Rhizopus and Mucor are found with nonseptate hyphae and wide angle branching (90º). The typical presentation is a paranasal infection in diabetic ketoacidosis patients. Broad non-septate hyphae are seen in tissue specimens. Educational Objective: Dimorphic fungi grow as molds at 25-30ºC and as yeast at body temperature (35-37ºC). Medically important dimorphic fungi include Sporothrix, Coccidioides, Histoplasma, Blastomyces and Paracoccidioides species.

A 32-year-old Caucasian male presents with low grade fever, cough and generalized malaise. The patient has otherwise been healthy and has no significant medical history. His past surgical history includes tonsillectomy and tympanostomy tube placement at the age of 8. Careful questioning reveals that the patient recently came back from an "outdoor adventure" trip in the Great Lakes, and began feeling sick only after returning. Microscopic examination of a KOH preparation of an exudate specimen reveals a large yeast cell with a single bud. When incubated in a specific medium at 75.2ºF (24ºC), a multicellular structure with branching, tubular cells grows. The patient is most likely infected with which of the following: Aspergillus fumigatus Blastomyces dermatitids Candida albicans Cryptococcus neofromans Malassezia furfur Rhizopus species

(Choice A) Aspergillus fumigatus has a mold form only and is not dimorphic. It is seen in biopsy specimens as septate hyphae that form acute angle V-shaped branches. Aspergillus mainly affects immunocompromised patients. (Choice C) Candida albicans is a component of normal human flora. It causes disseminated infections in immunocompromised patients. It is polymorphic, and it is commonly seen as a budding yeast or forming pseudohyphae. True hyphae also occur, usually only when Candida becomes invasive (in early stages, these hyphae can be identified as germ tubes). (Choice D) Cryptococcus neoformans causes meningitis in AIDS patients. This fungus exists in yeast form only. India ink stain of CSF reveals encapsulated yeast. (Choice E) Malassezia furfur causes a cutaneous mycosis (hypopigmented skin patches). KOH preparation of skin scrapings shows short hyphae and spores ("spaghetti and meatballs"). (Choice F) Rhizopus, Mucor, and Absidia are the saprophytic fungi that cause mucormycosis in immunosuppressed patients. Mold forms of Rhizopus and Mucor are found with nonseptate hyphae and wide angle branching (90º). The typical presentation is a paranasal infection in diabetic ketoacidosis patients. Broad non-septate hyphae are seen in tissue specimens. Educational Objective: Dimorphic fungi grow as molds at 25-30ºC and as yeast at body temperature (35-37ºC). Medically important dimorphic fungi include Sporothrix, Coccidioides, Histoplasma, Blastomyces and Paracoccidioides species.

A 45-year-old woman with advanced HIV infection has been receiving antiretroviral therapy for several years. When treatment was first initiated, the patient's viral load rapidly decreased to undetectable levels and her CD4+ cell count remained above 200/mm3. However, recent HIV RNA levels have increased above the detection threshold. After undergoing drug resistance testing, she is started on an integrase inhibitor, and her viral load decreases precipitously. Which of the following steps of viral replication is most likely to be inhibited as a result of this patient's new treatment? Cleavage of a viral polyprotein precursor Complementary DNA synthesis from a viral RNA template Fusion of viral and host phospholipid bilayers Production of viral messenger RNA Viral attachment to host cells

After an HIV virion enters a host cell, viral RNA is released from the nucleocapsid and transcribed into double-stranded DNA by reverse transcriptase. The viral DNA then enters the nucleus and, through the actions of integrase, permanently inserts into the host cell's chromosomes to become a provirus. RNA polymerase II then transcribes viral mRNA from the proviral DNA. Once synthesized, HIV mRNA exits through the nuclear pores into the cytoplasm, where it uses the host cell's ribosomes to translate the enzymes, glycoproteins, structural proteins, and regulatory proteins needed to promote viral replication and transmission. The full-length mRNA also serves as the viral genome that is packaged into newly synthesized virions. Raltegravir is an integrase inhibitor that disrupts the ability of double-stranded HIV DNA to integrate into the host cell's chromosomes. In the absence of integration, the viral genome cannot be transcribed by host cellular machinery and is eventually degraded by nucleases. (Choice A) Most of the proteins necessary for HIV replication are translated into a polyprotein precursor that is then cleaved into individual proteins by viral and cellular protease enzymes. The action of viral protease can be directly prevented by protease inhibitors such as saquinavir. (Choice B) Complementary DNA synthesis from the HIV RNA template is directly impaired by nucleoside and non-nucleoside reverse transcriptase inhibitors. (Choice C) Drugs that bind to gp41, HIV's transmembrane envelope protein, are known as fusion inhibitors (eg, enfuvirtide). These drugs prevent gp41 from undergoing the conformational changes necessary for viral membrane fusion with the host cell membrane. (Choice E) For HIV to infect a cell, the envelope glycoprotein gp120 must bind to both a CD4 molecule and a chemokine receptor (eg, CXCR4 or CCR5). CCR5 antagonists, such as maraviroc, exert their antiviral activity against HIV by blocking attachment and entry of CCR5-tropic viruses into CD4+ T cells and macrophages. Educational objective: Integration of double-stranded HIV DNA into the host cell's chromosomes is necessary to induce viral gene expression and prevent degradation of the viral genome. Raltegravir is an integrase inhibitor that disrupts HIV genome integration, preventing synthesis of viral mRNA.

A 65-year-old man comes to the emergency department due to fevers, chills, and confusion. He has a history of poorly controlled diabetes mellitus with a nonhealing ulcer of the left foot. His temperature is 38.3 C (101 F), blood pressure is 90/60 mm Hg, pulse is 112/min, and respirations are 22/min. On examination, there is slight erythema surrounding his left foot ulcer with foul-smelling discharge. No heart murmur is auscultated, the lungs are clear, and the abdomen is soft and nontender. Blood cultures are obtained, and he is started on broad-spectrum antibiotics. Twenty-four hours later, blood cultures grow β-lactamase-producing Bacteroides species. Which of the following is most likely to provide appropriate antimicrobial coverage for this patient? Doxycycline Fluconazole Metronidazole No additional medication Penicillin G benzathine Trimethoprim-sulfamethoxazole

Areas of necrosis and diminished blood supply (such as chronic, nonhealing ulcers) facilitate the growth of anaerobes. Bacteroides are gram-negative anaerobic rods that can produce β-lactamase, an enzyme that breaks down penicillins. β-lactamase inhibitors such as tazobactam, clavulanic acid, and sulbactam prevent these enzymes from functioning. The combination of piperacillin (extended-spectrum penicillin) and tazobactam is effective against a large number of gram-positive, gram-negative, and anaerobic bacteria and would be the most appropriate option among the medications listed. Other antimicrobials with anaerobic activity include metronidazole, carbapenems, and clindamycin (in most cases). In patients with diabetes who have a necrotic (foul-smelling) infected ulcer, broad-spectrum antimicrobial coverage should be used as polymicrobial infection with organisms other than Bacteroides may be present. (Choice A) Azithromycin is a macrolide antibiotic that binds to the 50S subunit of bacteria ribosomes, thereby inhibiting protein synthesis. It is used for infections with Chlamydia, Mycoplasma, Haemophilus influenzae, and Moraxella catarrhalis. Anaerobic activity against many Fusobacterium and some Bacteroides species may be limited. (Choice B) Ceftriaxone is a third-generation parenteral cephalosporin with poor anaerobic activity. It is active against H influenzae, Klebsiella pneumoniae, and Neisseria and Serratia species. (Choice C) Ciprofloxacin is a second-generation quinolone that acts by interfering with bacterial DNA gyrase (topoisomerase II). Ciprofloxacin is active against most gram-negative rods and bacilli, including Legionella and Pseudomonas aeruginosa. It is not effective against anaerobic infections. (Choice E) Vancomycin is a tricyclic glycopeptide that inhibits the synthesis of peptidoglycan, a main component of the cell wall of gram-positive bacteria. It is used against methicillin-resistant Staphylococcus aureus as well as Clostridium difficile. Educational objective: Piperacillin-tazobactam is a combination of extended-spectrum penicillin with β-lactamase inhibitor. It is effective against most gram-negative enteric rods (including Pseudomonas aeruginosa) and against Bacteroides fragilis.

A 34-year-old woman comes to the office for evaluation of recurrent transient pulmonary infiltrates. The patient has a history of bronchial asthma and has had several exacerbations over the past few years, particularly during the winter months. She is currently asymptomatic. She has no other medical problems and has never traveled outside the United States. Her medications include albuterol as needed and medium-dose inhaled glucocorticoids. Temperature is 37.1 C (98.8 F). Physical examination is unremarkable. Complete blood count shows eosinophilia. A chest CT scan reveals proximal bronchiectasis. This patient's condition is most likely related to colonization with which of the following? Adenovirus Aspergillus fumigatus Legionella pneumophila Pseudomonas aeruginosa Streptococcus pneumoniae Strongyloides sterocalis

Aspergillus fumigatus is a low virulence fungus that generally does not cause significant infections except in immunocompromised or debilitated patients. It may, however, colonize the bronchial mucosa. Patients with asthma or cystic fibrosis in particular may develop an allergic hypersensitivity reaction to the fungus. The result is allergic bronchopulmonary aspergillosis (ABPA), which occurs in 5%-10% of corticosteroid-dependent asthmatics. Patients with ABPA have very high serum IgE levels, eosinophilia, and IgE plus IgG serum antibodies to Aspergillus. There is intense airway inflammation and mucus plugging with exacerbations and remissions. Repeated exacerbations may produce transient pulmonary infiltrates and proximal bronchiectasis. (Choice A) Although viral respiratory infections can cause asthma exacerbations and adenovirus colonization of the lung may be associated with asthma chronicity, pure viral pneumonias do not progress to bronchiectasis. (Choice C) Asthmatics do not have a predisposition to lung colonization with Legionella. (Choice D) Pseudomonas aeruginosa is an opportunistic, aerobic gram-negative rod that is a frequent, and sometimes deadly, pulmonary pathogen in patients with cystic fibrosis or neutropenia. (Choice E) Although asthma may be a risk factor for the development of pneumococcal lung colonization and pneumonia, pneumococcal disease generally results in resolution with preservation of lung lobular architecture. Potential complications of pneumococcal bronchopneumonia or lobar pneumonia include lung abscess, empyema, and lung fibrosis. Bronchiectasis is not a common outcome of pneumococcal pulmonary disease. (Choice F) Strongyloidiasis is typically asymptomatic but can sometimes cause pulmonary symptoms (eg, cough, asthma-like bronchospasm, hemoptysis) or a transient pneumonia as the larvae migrate through the lungs on their way to the laryngopharynx. However, progression to bronchiectasis is not commonly reported. Educational objective: Allergic bronchopulmonary aspergillosis (ABPA) due to Aspergillus fumigatus may complicate asthma. ABPA can result in transient recurrent pulmonary infiltrates and eventual proximal bronchiectasis.

A 64-year-old man comes to the emergency department due to worsening left lower extremity pain, swelling, and redness over the past 3 days. The patient has had no trauma but reports feeling feverish. Medical history is significant for hypertension and obesity. Temperature is 38 C (100.4 F), blood pressure is 130/86 mm Hg, pulse is 92/min, and respirations are 18/min. BMI is 35 kg/m2. Physical examination shows diffuse erythema extending up to the left midcalf with indistinct border. There is increased warmth, tenderness, and edema of the left leg. No areas of fluctuation or purulent exudate are present, but the interdigital skin of the feet is macerated and fissured. Laboratory testing shows neutrophilic leukocytosis, and Doppler ultrasonography is negative for deep venous thrombosis. Infection with which of the following organisms is most likely responsible for this patient's current symptoms? Beta-hemolytic streptococci Candida albicans Clostridium perfingens Mycobacterium marinum Pseudomonas aeruginosa Staphylococcus epidermis

Cellulitis is an acute bacterial infection of the skin and subcutaneous tissues characterized by progressive skin erythema, warmth, and tenderness. Many patients also have associated fever, leukocytosis, and localized lymphadenopathy. Most cases are driven by disruptions to the skin barrier due to minor trauma, lymphedema, or dermatophytic infection, which allow a portal of entry for bacteria. Cases of cellulitis are generally divided as follows: Nonpurulent cellulitis is characterized by skin warmth, edema, and erythema with no fluctuant nodules. It is most often caused by beta-hemolytic streptococci (groups A, B, C, G, and F); group A streptococcus (Streptococcus pyogenes) accounts for the majority of cases. Purulent cellulitis is characterized by a painful, fluctuant nodule in the dermis or subcutaneous tissue with or without surrounding erythema. The most common cause is Staphylococcus aureus; strains that express the virulence factor Panton-Valentine leukocidin are particularly likely to cause skin abscess. (Choice B) Candida albicans is a skin commensal that can cause inflammatory, moist, weeping, erythematous lesions in skin folds. It is not a common cause of acute, spreading cellulitis. (Choice C) Clostridium perfringens causes gas gangrene after traumatic injury (eg, knife wound, surgery) to the skin and soft tissue. It is usually characterized by sudden-onset severe pain, bullous skin lesions, and signs of systemic toxicity/sepsis. (Choice D) Mycobacterium marinum skin infection is usually associated with saltwater exposure and is generally characterized by solitary papules that ulcerate and scar. (Choice E) Pseudomonas aeruginosa skin infections are most common in those who have had exposure to a hot tub ("hot tub folliculitis") or in patients with diabetes mellitus (eg, diabetic foot infection). It is a much less common cause of cellulitis than beta-hemolytic streptococci. (Choice F) Staphylococcus epidermidis is a low-virulence skin commensal that rarely causes cellulitis. However, it may occasionally cause bacteremia in patients with intravenous catheters. Educational objective: The most common cause of nonpurulent cellulitis is beta-hemolytic streptococci, particularly group A streptococcus. The most common cause of purulent cellulitis is Staphylococcus aureus.

A 28-year-old man comes to the emergency department due to 2 weeks of headaches, fever, and episodes of inattentiveness. The patient has a history of Hodgkin lymphoma and is undergoing treatment with combination chemotherapy. Temperature is 38.2 C (100.8 F), blood pressure is 138/84 mm Hg, and pulse is 72/min. Lumbar puncture reveals an elevated opening pressure, and cerebrospinal fluid analysis shows moderately increased protein levels and low glucose levels. Light microscopy of the cerebrospinal fluid is shown in the image. Light green halo (ID=116) Which of the following should be included in initial treatment for this patient's current condition? Amphotericin B Ampicillin Ceftriaxone Ganciclovir Pyimethamine Voriconazole

Cryptococcus neoformans is an opportunistic yeast that can be visualized with India ink staining. India ink stains the background of the slide, which allows the thick polysaccharide capsule of the yeast to be visualized (clear halo between the dark background and the center of the organism). India ink staining of the cerebrospinal fluid (CSF) is a common diagnostic modality in those with suspected cryptococcal meningitis. However, CSF cryptococcal (polysaccharide) antigen testing is more sensitive and specific, so it is also usually performed. Culture of the fungus on Sabouraud agar confirms the diagnosis. Patients with cryptococcal meningitis usually develop slowly worsening headache, fever, and lethargy. Immunocompromised individuals are at greatest risk, particularly those who have AIDS or hematologic cancers or patients on chronic glucocorticoids. CSF analysis usually shows low glucose, increased protein, and mild pleocytosis with a predominance of lymphocytes. Initial treatment involves induction therapy with amphotericin B and flucytosine. Once patients are stabilized, they are switched to long-term maintenance therapy with fluconazole. (Choices B and C) Ampicillin is used to treat Listeria meningitis; Listeria is a gram-positive rod with characteristic tumbling motility on light microscopy. Ceftriaxone can treat pneumococcal and Haemophilus influenzae meningitis. Ampicillin and ceftriaxone are beta-lactam antibiotics that inhibit peptidoglycan cross-linking and lead to bactericidal effects. Fungi cannot be treated with beta-lactam antibiotics as their cell walls do not contain peptidoglycan. (Choice D) Ganciclovir inhibits viral DNA synthesis and is used to treat cytomegalovirus meningoencephalitis, which is diagnosed using PCR. (Choice E) Pyrimethamine is an antifolate agent used in the treatment of malaria and toxoplasmosis. (Choice F) Azole drugs inhibit the synthesis of ergosterol, a crucial fungal cell membrane component. Spectrum of activity varies based on the particular agent; voriconazole is most effective against Aspergillus. Fluconazole is used for long-term maintenance therapy in cryptococcal meningitis, but is fungistatic at typically used doses and is less effective than amphotericin B for initial treatment. Educational objective: Patients with suspected cryptococcal meningitis can be diagnosed using India ink staining of the cerebrospinal fluid. Treatment with amphotericin B and flucytosine is required at first, followed by long-term fluconazole maintenance therapy.

A 21-year-old woman volunteers in a research study evaluating the effects of viral infection on immunoglobulin production. The patient is a college student who has no past medical history and does not take any medications. She has not felt ill recently. Her lymphocytes are isolated from whole blood and cultured. The mature B lymphocytes exposed to a specific virus in vitro acquire the ability to proliferate indefinitely while maintaining the capacity to secrete immunoglobulins. Which of the following tests is used to diagnose an in vivo infection with this virus? Circulating surface antigen Cryoglobulines Hemadosorption in the cell culture Heterophile antibodies Papanicolaou test Tzanck smear

Epstein-Barr virus (EBV) commonly infects B cells, stimulating them to enter the cell cycle and proliferate continuously ("transformation" or "immortalization"). This is accomplished when EBV-encoded oncogenes activate proliferative and anti-apoptotic signaling pathways within the infected B cell. In an immunocompetent host, a vigorous immune response holds EBV-induced B cell proliferation in check. The immortalized B cells maintain the ability to secrete immunoglobulins (Ig) and B-cell activation products (eg, CD23), with very few of them releasing virus particles at any one time. Diagnosing EBV infection relies on detecting a heterogeneous group of heterophile IgM antibodies that react with antigens on horse erythrocytes (or sheep erythrocytes in the classic Paul Bunnell test). Agglutination of these horse erythrocytes by human serum is a sensitive and highly specific (~100%) test for EBV infection in human B cells. The mechanism remains unclear; EBV either induces a humoral response to heterophile antigens or stimulates nonspecific polyclonal activation of B cells. (Choice A) EBV typically causes a latent infection of B cells, with viral genome persistence but no full virion replication, no lytic release, and sparse viral protein expression. Normally, only minimal amounts of EBV-specific antigens circulate in an immunocompetent host; therefore, blood tests for such antigens are insensitive to latent EBV infection. Detection of circulating surface antigen is used to diagnose hepatitis B virus infection. (Choice B) Cryoglobulins, cold-precipitable serum proteins that contain Igs, are most commonly associated with hepatitis C virus infection. (Choice C) Some influenza and parainfluenza viruses are associated with hemadsorption (hemagglutinins or glycoproteins with a high affinity for erythrocytes expressed on the host cell surface). Hemadsorption is not characteristic of EBV. (Choice E) The Papanicolaou test is used to screen cervical cytology specimens for dysplasia caused by oncogenic strains of human papillomavirus. (Choice F) Tzanck smear is used to diagnose herpes simplex or varicella zoster virus infection. The epithelial cells scraped from the ulcer base are prepared with a Wright-Giemsa stain and examined for multinucleated giant cells and intranuclear inclusions. Educational objective: Epstein-Barr virus (EBV) commonly infects B cells, stimulating them to proliferate continuously ("transformation" or "immortalization"). EBV is an oncogenic virus that promotes polyclonal B cell proliferation and heterophile antibody production. The heterophile antibody test is sensitive and highly specific for EBV infection.

A 30-year-old man is admitted to the intensive care unit after his wife found him unconscious at home. She says that he has lost a significant amount of weight recently despite no significant change in dietary habits. Laboratory studies show a serum pH of 7.1 and positive serum ketones. His mucus membranes are extremely dry. Treatment is initiated, and the patient's condition improves. However, he now has fever, headache, and eye pain. Examination of the nasal cavity shows a black necrotic eschar adherent to the inferior turbinate. Which of the following procedures would most likely reveal the pathogen responsible for this patient's infection? Blood cultures Lumbar puncture Mucosal biopsy Ophthalmoscopy Serologic testing Skin hypersensitivity test

Facial pain, headache, and nasal eschar in a patient with likely diabetic ketoacidosis (pH 7.1, positive ketones, and dry mucous membranes in a young patient with recent unintended weight loss) are highly suggestive of mucormycosis, caused by Mucor or Rhizopus, among others. Most cases are rhinocerebral; the infection (acquired by spore inhalation) ascends from the nasal passage to the sinuses/orbits and then sometimes to the brain, leading to confusion, neurological deficits, and death. The increased incidence of mucormycosis in patients with diabetes mellitus is due to the ketone reductase activity of Rhizopus, allowing it to thrive in ketoacidotic environments. Histologic examination (eg, mucosal biopsy) is necessary for diagnosis. The fungi appear as ribbon-like broad nonseptate hyphae with right-angle branching. Tissue invasion is seen along blood vessels; vascular thrombosis and tissue necrosis can occur. A black necrotic eschar in the nasal cavity is characteristic. Management includes amphotericin B and surgical debridement. Mucormycosis (common in diabetes) must be differentiated from invasive aspergillosis causing rhinosinusitis (common in neutropenic patients). On light microscopy, Aspergillus is seen as septated narrow hyphae with sharp-angle branching. (Choice A) Blood cultures are usually negative in mucormycosis despite the organism's angioinvasive nature. In contrast, blood cultures can be useful in diagnosing candidemia, which has a variable clinical presentation ranging from low-grade fever to severe sepsis with multiorgan failure. (Choice B) Lumbar puncture helps to diagnose cryptococcal meningitis, a fungal infection that typically has an indolent presentation and usually affects patients with AIDS. India ink staining of cerebrospinal fluid reveals the classic encapsulated yeast of Cryptococcus neoformans. (Choice D) Ophthalmoscopy is useful for the diagnosis of Candida endophthalmitis, which typically presents with impaired visual acuity. (Choices E and F) Serologic testing is important in the diagnosis of allergic bronchopulmonary aspergillosis (ABPA), where increased titers of IgE and antibodies to Aspergillus fumigatus are seen. Skin hypersensitivity testing is also useful. However, the typical presentation of ABPA is recurrent episodes of fever and bronchial obstruction in a patient with asthma. Educational objective: Facial pain, headache, and a black necrotic eschar in the nasal cavity of a patient with diabetic ketoacidosis are highly suggestive of mucormycosis. Histologic examination of the affected tissue is necessary to confirm the diagnosis. The fungi show broad nonseptate hyphae with right-angle branching. Treatment consists of surgical debridement and antifungal therapy.

A microbiologist is studying the structure and function of bacterial cell walls. In an experiment, group A streptococci are treated with chemicals to solubilize the cell wall. Various cell wall-associated proteins are subsequently extracted. Electron microscopic evaluation of a specific protein shows structural homology with mammalian tropomyosin and myosin. This protein acts as a virulence factor for certain species of the organism. Which of the following is the most likely function of this bacterial cell wall-associated protein? Excrete antibiotics Protect from osmotic lysis Provide mechanical cell support Resist phagocytosis Transport nutrients into the cell

Gram-positive organisms have cell walls composed primarily of a thick peptidoglycan layer and teichoic acid. However, additional cell wall components are often present that contribute to virulence, motility, or the ability to adhere to epithelial surfaces. For instance, the cell wall of group A streptococcus (also known as Streptococcus pyogenes) is studded with M protein, a virulence factor that inhibits phagocytosis, prevents complement binding, and aids in epithelial attachment. M protein is an alpha-helical coiled-coil protein that shares epitopes and structural homology with other alpha-helical coiled-coil proteins such as tropomyosin and myosin. In some patients, the protective antibodies against M protein that develop in S pyogenes acute infection may cross-react with myosin epitopes in the heart and cause rheumatic carditis. (Choice A) Efflux pumps are generally globular proteins with multiple domains. They reside in the bacterial cell membrane and excrete toxic substances from the interior of the cell. Bacteria can use efflux pumps to generate antibiotic resistance to drugs that operate in the interior of the cell, such as fluoroquinolones and aminoglycosides. (Choices B and C) The bacterial cell wall of S pyogenes provides mechanical support and protects it from osmotic lysis. It is composed primarily of peptidoglycan, which forms a mesh-like, cross-linked polymer of peptides and sugars. (Choice E) Porins in the cell membrane aid in the movement of nutrients into bacterial cells. They are composed of protein strands that group together to form a barrel (beta-barrel). Educational objective: The major virulence factor of Streptococcus pyogenes is M protein, an alpha-helical coiled-coil protein that shares structural homology with tropomyosin and myosin. It extends from the cell wall and prevents phagocytosis, inhibits complement binding, and mediates bacterial adherence. Antibodies against M protein form shortly after acute infection and may cross-react with epitopes on myosin, leading to rheumatic carditis.

A 10-year-old boy is brought to the office due to abrupt onset fever and sore throat that began a day ago. He has had similar illnesses in the past, which were treated with oral antibiotics. The patient has no other medical conditions and has received all recommended vaccinations. Temperature is 38.3 C (101 F). Physical examination shows erythema of the posterior pharyngeal wall and enlarged tonsils, which are covered by white exudate. There are no enlarged cervical lymph nodes. Throat swab specimen is obtained. Which of the following would be most helpful in determining the need for antibiotic treatment in this patient? Gram staining of the swab for gram-positive cocci Serum titer assay for anti-M protein antibodies Serum titer assay for anti-streptolysin O antibodies Swab culture to demonstrate colonies with incomplete hemolysis Swab immunoassay for streptococcal group A antigens

Group A streptococcus (GAS), also called Streptococcus pyogenes, is the leading cause of bacterial pharyngitis in children and adolescents. Manifestations include abrupt-onset fever and sore throat; patients may also have nausea, vomiting, and headache. Examination often reveals exudative tonsillopharyngitis and enlarged tonsils. An alternate diagnosis should be suspected in those with manifestations of viral illness (eg, coryza, conjunctivitis, cough, hoarseness, mouth ulcers). Patients with suspected GAS pharyngitis require microbiologic testing prior to initiation of antimicrobials. Throat swab samples are collected in-office and are subsequently evaluated by the following: Rapid antigen detection testing (RADT) - this immunoassay evaluates for GAS antigens and provides rapid, in-office results. However, due to the test's limited sensitivity, patients with negative RADT require throat culture to confirm they do not have the organism. Throat culture - samples are plated on blood agar and evaluated for beta-hemolysis within 48 hours. GAS is beta-hemolytic due to the expression of cytolysins (eg, streptolysin O), which create a broad zone of complete hemolysis (not incomplete hemolysis) around plated colonies (Choice D). Patients with positive RADT or throat culture require antibiotic therapy (eg, amoxicillin) to prevent complications of GAS (eg, rheumatic fever), limit transmission to others, and reduce duration/severity of symptoms. (Choice A) Gram-stain cannot be used to definitively determine if GAS is present because other oropharyngeal organisms (eg, viridans streptococci) also appear as gram-positive cocci in chains. (Choice B) Antibodies against M protein, a virulence factor that coats the surface of GAS, provide protective immunity against future infections with a particular GAS strain. However, antibody titers take several weeks to elevate after acute infection; therefore, antibody testing is not used to diagnose acute pharyngitis. (Choice C) Elevated antibody titers against streptolysin O indicate recent GAS infection and can aid the diagnosis of GAS complications (eg, rheumatic fever, glomerulonephritis). However, antibodies against streptolysin take several weeks to form; therefore, they are not used to diagnose acute pharyngitis. Educational objective: Group A streptococcus (GAS) should be suspected in those with acute-onset sore throat, exudative tonsillopharyngitis, and no evidence of viral symptoms (eg, coryza, cough, conjunctivitis). In-office throat swab with rapid antigen detection testing (immunoassay for GAS antigens) can provide on-site microbiologic confirmation, allowing for early initiation of treatment.

A pharmaceutical researcher is trying to develop a vaccine against the hepatitis C virus. She infects a chimpanzee with hepatitis C virus of known genotype and subtype. Several weeks later, a liver sample is obtained, and viral RNAs are extracted from the hepatocytes. A genetic study of the viral genomes reveals that the extracted RNA sequences vary significantly from that of the original infecting virus. This genetic instability is most likely due to the lack of which of the following features during the viral replication process? 3' to 5' exonuclease activity 5' to 3' exonuclease activity Glycosylase activity Ligase activity Nucleotide specificity Primase activity

Hepatitis C virus (HCV) has >6 genotypes and multiple subgenotypes. These variant strains differ primarily at hypervariable genomic regions, such as those found in the sequences encoding its 2 envelope glycoproteins. Moreover, there is no proofreading 3' → 5' exonuclease activity built into the virion-encoded RNA polymerase. As a result, the RNA polymerase makes many errors during replication, and several dozen subspecies of HCV are typically present in the blood of an infected individual at any one time. (Choice B) 5' → 3' exonuclease activity allows DNA polymerase I to engage in nick translation, which is important for both DNA repair and removal of RNA primers during replication. This feature is not the cause of instability in HCV. (Choice C) DNA glycosylase is important in base excision repair as it removes the nitrogen base from the sugar-phosphate backbone. An endonuclease then completes the remainder of the repair. This enzyme is not the cause of instability in HCV. (Choice D) DNA ligase is used to join 2 fragments of DNA and is therefore necessary for both DNA repair and replication. The HCV genome is a single-stranded, positive-sense RNA molecule that does not require ligase activity during replication. (Choice E) Nucleotide specificity refers to the importance of inserting the correct nucleotide (eg, adenine, guanine, cytosine, thymine, uracil) into a strand of DNA or RNA. It is not the cause of instability in HCV. (Choice F) DNA primase is a form of RNA polymerase. In bacteria, it binds with DNA helicase and synthesizes a short RNA primer to which nucleotides can be added by DNA polymerase. This enzyme is not the cause of instability in HCV. Educational objective: The hepatitis C virus is genetically unstable because it lacks proofreading 3' → 5' exonuclease activity in its RNA polymerase. Its envelope glycoprotein sequences also contain a hypervariable region prone to frequent genetic mutation.

A 4-year-old boy is brought to the office due to skin lesions on his chin for the past 2 days. Temperature is 36.1 C (97 F). The patient is well-appearing. There are mildly tender perioral papules and pustules, some of which appear to have broken down and are covered with amber-colored crusting. A wound culture is taken, and Gram stain shows gram-positive cocci in chains. Which of the following symptoms would be most likely to develop as a potential complication of this patient's infection? Ascending weakness Facial puffiness and dark urine Fatigue and heart murmur Joint pain and eye redness Unilateral facial drooping

This child has impetigo, a superficial bacterial infection that most commonly develops on the face or extremities in children. Patients usually develop mildly itchy or painful erythematous papules and pustules that later erupt, forming a honey-colored crusting. The most common pathogen is Staphylococcus aureus followed by group A Streptococcus (GAS, S pyogenes). Treatment of localized impetigo is with topical antibiotics (eg, mupirocin), and most patients have no long-term sequelae. However, a minority of patients may develop poststreptococcal glomerulonephritis (PSGN) several weeks after impetigo caused by GAS (as indicated by this patient's Gram stain showing gram-positive cocci in chains). Renal damage is caused by immune complex deposition as well as complement activation, leading to a massive inflammatory response that damages the glomerular basement membrane. Symptoms of PSGN range from asymptomatic, microscopic hematuria to gross hematuria and nephrotic-range proteinuria. In addition to tea- or cola-colored urine, patients may develop periorbital/facial swelling, generalized edema, and hypertension. Laboratory evaluation typically reveals elevated creatinine and decreased complement C3 level. (Choice A) Ascending weakness is a classic feature of Guillain-Barre syndrome, which may be preceded by an upper respiratory or gastrointestinal (eg, Campylobacter jejuni) infection. GAS disease is not associated with this condition. (Choice C) Fatigue and a new heart murmur due to carditis can be manifestations of acute rheumatic fever, a potential poststreptococcal sequelae associated with GAS pharyngitis. This disease is not associated with impetigo in developed regions, although this relationship is less clear in some parts of the developing world. (Choice D) Eye redness (due to conjunctivitis or anterior uveitis) and joint pain may be seen in reactive arthritis, which typically occurs following a diarrheal (eg, Salmonella) or genitourinary (eg, chlamydial) infection, not impetigo. (Choice E) Unilateral facial drooping due to a facial nerve palsy (ie, Bell palsy) is often associated with herpes simplex virus infection, not impetigo. Educational objective: Poststreptococcal glomerulonephritis is a potential complication of group A streptococcal infection (eg, impetigo, pharyngitis). Symptoms include proteinuria (often associated with periorbital or generalized edema) and hematuria. Acute rheumatic fever is a postinfectious sequelae of streptococcal pharyngitis only, not impetigo.

A 36-year-old man comes to the office due to 2 months of fatigue, intermittent arthralgias, and poor appetite. The patient has no prior medical conditions and takes no medications. He does not use tobacco or illicit drugs, and he drinks alcohol occasionally. The patient has had multiple sexual partners and uses condoms inconsistently. Physical examination shows no abnormalities. Liver aminotransferases are elevated. Hepatitis serology shows the following: Hepatitis A virus antibody, total negative Hepatitis B surface antibody (HBsAb) negative Hepatitis B core antibody (HBcAb), total positive Hepatitis B surface antigen (HBsAg) positive Hepatitis B E antigen (HBeAg) positive Hepatitis C virus antibody negative Which of the following genome replicative processes is most likely used by the virus infecting this patient? Double-stranded DNA to double-stranded DNA template to double-stranded DNA progeny Double-stranded DNA to +RNA template to partially double stranded DNA progeny Single-stranded DNA to double-stranded DNA template to single-stranded DNA progeny Single-stranded +RNA to double-stranded DNA template to single-stranded +RNA progeny Single-stranded +RNA to RNA template to single-stranded +RNA progeny Single-stranded +RNA to +RNA template to RNA progeny

Acute hepatitis B virus (HBV) infection is marked by the presence of HBsAg, HBeAg, and anti-HBc, and a lack of anti-HBs. Infections are often asymptomatic, but many patients have a few months of jaundice, fatigue, nausea, and right upper-quadrant discomfort. Perinatal transmission is most common in high-prevalence countries (eg, Sub-Saharan Africa); in low-prevalence regions (eg, United States), most cases occur due to unprotected sex or intravenous drug use. HBV is a hepadnavirus composed of: an outer lipid envelope that contains viral-encoded proteins (HBsAg) and host lipid components. an icosahedral nucleocapsid core that contains a circular, partially double-stranded DNA genome and a DNA polymerase with reverse transcriptase activity. HBV binds to a bile salt transporter on the surface of hepatocytes and enters the cell. After the virus is uncoated in the host cytoplasm, the single-stranded DNA portion of the viral genome is completed (repaired) by cellular DNA polymerases. This generates double-stranded viral DNA, which is subsequently transcribed by host RNA polymerase into a +single-stranded RNA pregenome. The +RNA template is then translated into the proteins that compose the virus (eg, envelope, core, polymerase); it is also converted by viral DNA polymerase/reverse transcriptase into the partially double-stranded DNA progeny of developing viral particles. (Choice A) This replicative sequence characterizes adenovirus, herpesvirus, and poxvirus. (Choice C) This replicative sequence characterizes parvovirus B19. (Choice D) This replicative sequence characterizes the retroviruses (eg, HIV). (Choice E) This replicative sequence characterizes poliovirus. (Choice F) This replicative sequence characterizes influenza virus, measles virus, and rabies virus. Educational objective: The hepatitis B virus (HBV) replicates via the following sequence: double-stranded DNA → +RNA template → partially double-stranded DNA progeny. Although it is a DNA virus, HBV uses reverse transcription to generate new viral DNA from a positive-sense RNA template.

A 49-year-old man comes to the office due to 4 months of cough and weight loss. His medical history is otherwise unremarkable and he takes no medications. The patient is a farmer who lives in Mississippi. He does not use tobacco, alcohol, or illicit drugs. His temperature is 37.9 C (100.2 F). Examination shows right lobe crackles. Chest x-ray reveals a pulmonary infiltrate in the right upper lobe. Bronchoscopy with transbronchial biopsy shows granulomatous inflammation. HIV testing is negative. Which of the following is most likely causing this patient's symptoms? Blastomyces dermatitidis Candida albicans Mucor species Penicillium marneffer Pneumocystis jiroveci

Blastomyces dermatitidis is a dimorphic fungus that can cause pulmonary infections in immunocompetent people and should be considered in individuals from an endemic area (states adjacent to and east of the Mississippi and Ohio river valleys; some overlap with Histoplasmosis). The fungus exists in the environment in mold form and is found in soil, organic matter, and many animals (eg, dogs, horses). It is transmitted by the respiratory route, entering the lungs and transforming into yeast form (thermal dimorphism). B dermatitidis infection can be asymptomatic or it can mimic community-acquired pneumonia (resolves spontaneously) or chronic pneumonia (may be indistinguishable from tuberculosis, other fungal infections, or cancer). Pulmonary blastomycosis is characterized by granuloma formation. Extrapulmonary disease (skin, bone, and genitourinary system) occurs in immunocompromised patients. Examination of tissues or respiratory fluids shows round yeast with thick walls and broad-based budding. Itraconazole is the preferred treatment for blastomycosis. (Choice B) Candida albicans is an opportunistic pathogen. It may cause superficial infections (eg, vulvovaginitis) in an immunocompetent host. Systemic disease, however, occurs only in the immunosuppressed (especially neutropenic patients). (Choice C) Mucor species infect the paranasal sinuses in patients with neutropenia, burns, or diabetes mellitus. Mucormycosis is strongly associated with diabetic ketoacidosis. (Choice D) Penicillium marneffei is an opportunistic infection in immunocompromised patients, primarily those with AIDS, in Southeast Asia. It is rare in immunocompetent individuals. (Choice E) Pneumocystis jiroveci causes pulmonary disease in immunocompromised patients, particularly those with HIV. P jiroveci does not cause symptoms in immunocompetent hosts. Educational objective: Blastomyces dermatitides can cause pulmonary disease in the immunocompetent host. Dissemination (most commonly to skin and bones) occurs in immunocompromised individuals.

A 48-year-old man comes to the physician with a 6-month history of cough and fatigue. He also has experienced malaise and weight loss. He does not recall any sick contacts. The patient is a carpenter. He smokes 1 pack of cigarettes daily. Temperature is 37.1 C (98.8 F). Examination shows left lower lobe crackles. HIV testing is negative. Chest x-ray reveals a pulmonary infiltrate in the lower lobe of the left lung. Results from sputum potassium hydroxide preparation are shown below. Broad-budding yeast (ID=120) Which of the following is the most likely cause of this patient's condition? Aspergillus fumigatus Blastomyces dermatitidis Candida albicans Cocciodiodes immitis Cryptococcus neoformans Histoplasma capsulatum Rhizopus species

Blastomyces dermatitidis, a fungus endemic in the southeastern United States (states east of the Mississippi River), is present in soil and rotten organic matter. Blastomyces is a dimorphic fungus, meaning it assumes different forms at different temperatures. In the human body (37-40 C), it assumes the yeast form (single cells). The image shows a typical large yeast with a single, broad-based bud. The mold form (branching hyphae) predominates in the environment, at average temperatures of 25-30 C. Infection occurs by inhalation of aerosolized fungus from the environment. In about half of immunocompetent individuals, blastomycosis may present as a lung infection or cause a flu-like illness (fever, chills, myalgia, headache, nonproductive cough) or pneumonia (fever, cough, pleuritic chest pain). Pulmonary blastomycosis is diagnosed by finding the typical yeast forms. In immunocompromised patients, blastomycosis can cause disseminated disease. Patients experience systemic symptoms (fever, weight loss, night sweats), lung involvement (cough, dyspnea), skin lesions (papules, pustules, ulcers, verrucous lesions), and bone pain (caused by lytic lesions). (Choice A) Aspergillus fumigatus can cause lung disease in immunocompromised patients. It has only a mold form; in tissue samples it is seen as septate hyphae that branch at 45-degree angles. (Choice C) Candida albicans is characterized by small oval yeast with narrow-based budding. Candida pneumonia is a very rare disease. (Choice D) Coccidioides immitis is also a dimorphic fungus. It is seen as spherules (round encapsulated structures containing many endospores) in tissue samples. (Choice E) Cryptococcus neoformans causes lung disease and meningitis in immunocompromised patients. In contrast to blastomycosis, it forms round yeast with variable sizes and narrow-based buds. Cryptococcus has a thick polysaccharide capsule that appears clear with India ink staining and stains red with mucicarmine. (Choice F) Histoplasma capsulatum can cause lung disease. Like Blastomyces, it is a dimorphic fungus. Unlike Blastomyces, the yeast form of Histoplasma is smaller and is often found intracellularly within macrophages. H capsulatum is endemic in parts of the central and eastern United States along the Ohio and Mississippi river valleys. (Choice G) Rhizopus species can cause lung disease in immunocompromised patients. Rhizopus is the most common genus causing mucormycosis. It has only a mold form, with broad ribbon-like hyphae with rare septations. Educational objective: Blastomyces dermatitidis is a dimorphic fungus seen in tissue as round or oval yeasts with thick walls and broad-based budding. It is endemic in the southeastern United States (states east of the Mississippi River). The lungs are the primary site of involvement, and the skin and bone are the major sites of dissemination.

A 45-year-old man comes to the emergency department due to 2 weeks of chest pain and cough. He has a history of advanced HIV and has taken his antiretroviral medications inconsistently over the past few months. Temperature is 38.1 C (100.6 F). Crackles are heard on pulmonary examination. CD4 cell count is 98/mm3. Chest x-ray reveals lung nodules and hilar lymphadenopathy. A bronchoscopy is performed. Mucicarmine staining of the patient's bronchoalveolar fluid reveals budding yeast forms with thick capsules. The microorganism responsible for this patient's pulmonary infection most frequently causes which of the following? Esophagitis Interstitial pneumonia Meningoencephalitis Oral plaques Sinusitis Skin infection

Budding yeasts with thick capsules that stain with mucicarmine are characteristic of Cryptococcus neoformans, which typically affects immunocompromised patients (ie, opportunistic pathogen). C neoformans is present in soil and pigeon droppings; it is transmitted by the respiratory route but not acquired via person-to-person contact. Inhaled yeast forms enter the lungs, and in immunocompetent persons, they are cleared by a macrophage response directed by T cells. In individuals with an impaired cellular immune response, C neoformans can disseminate and cause symptomatic disease, most commonly meningoencephalitis. Cryptococcal meningoencephalitis is typically seen in patients with HIV, sarcoidosis, or leukemia and in those on high-dose corticosteroid therapy. Headache, nausea, vomiting, and confusion are common symptoms. Diagnosis can be made by examining cerebrospinal fluid (CSF), which shows round, budding, encapsulated yeasts. With India ink staining, the thick polysaccharide capsules classically appear as peripheral clearings or halos. Serologic testing (latex agglutination) is used to detect the C neoformans capsular antigen in CSF. (Choices A and D) Esophagitis in a patient with HIV is most commonly caused by Candida infection; it presents with odynophagia and is diagnosed by endoscopy with biopsy. Oral thrush due to Candida, which manifests as white oral plaques, is associated with diabetes mellitus, immunosuppression (eg, HIV), and treatment with antibiotics and corticosteroids. (Choice B) Although C neoformans enters the body through the lungs, pulmonary symptoms are often not prominent. Some patients may develop cough, dyspnea, or pleuritic chest pain, but pulmonary cryptococcosis is a less common manifestation than cryptococcal meningoencephalitis. In contrast, Pneumocystis jirovecii is a frequent cause of pneumonia in immunocompromised individuals, but microscopy would show cup- or crescent-shaped organisms (not budding yeast with thick, mucicarmine-positive capsules). (Choice E) Mucormycosis, which classically affects the paranasal sinuses, occurs in immunocompromised patients and is strongly associated with diabetes mellitus and diabetic ketoacidosis. Aspergillus fumigatus is another fungus that may cause sinusitis. (Choice F) Cutaneous cryptococcal disease (eg, papules, pustules) is rare. Dimorphic fungi Blastomyces and Histoplasma may cause cutaneous lesions, but the yeast forms do not have mucicarmine-positive capsules. Educational objective: Meningoencephalitis is the most common symptomatic presentation of Cryptococcus neoformans infection and typically occurs in immunocompromised patients. Microscopically, C neoformans appears as budding yeasts with thick polysaccharide capsules, which can be demonstrated using mucicarmine or India ink staining.

A mother brings her 4-year-old son to see you because he has been experiencing decreased appetite, fever, and right-sided ear discharge for the past 24 hours. His past medical history is significant for three episodes of otitis media over the last year. You prescribe amoxicillin. Adding clavulanic acid to this treatment regimen would serve which of the following purposes? Kill bacteria via a different mechanism of action Decrease amoxicillin cleavage by immune cells Decrease amoxicillin cleavage by bacterial cells Slow liver metabolism of amoxicillin Decrease renal clearance of amoxicillin

Certain strains of bacteria produce the beta-lactamase enzyme, which hydrolyzes the beta-lactam ring of penicillin family antibiotics rendering them ineffective. To help prevent this effect, beta-lactamase inhibiting compounds have been developed. The beta-lactamase inhibitors include clavulanic acid, sulbactam and tazobactam. Concurrent administration of a beta-lactamase inhibitor with a beta-lactamase susceptible antibiotic allows that antibiotic to retain its efficacy against the offending bacterium. For example, giving clavulanate together with amoxicillin expands amoxicillin's spectrum of activity to include strains of beta-lactamase synthesizing bacteria which would otherwise have been resistant to amoxicillin alone. (Choice A) Clavulanic acid has negligible intrinsic antibacterial activity. (Choice B) Immune cells do not produce beta-lactamases. (Choice D) Clavulanic acid has no effect on the biotransformation of amoxicillin in the liver. Moreover amoxicillin undergoes enterohepatic recirculation and is excreted primarily in unchanged form in the urine. (Choice E) Probenecid is a compound that decreases the renal clearance of penicillins by inhibiting the tubular organic acid secretion system. This agent has been utilized to prolong the action of penicillin in some settings. Educational Objective: Clavulanic acid, sulbactam and tazobactam are beta-lactamase inhibitors. Concurrent administration of clavulanate with amoxicillin expands amoxicillin's spectrum of activity to include strains of β-lactamase synthesizing bacteria that are resistant to amoxicillin alone.

Researchers find that even though the current recombinant hepatitis B vaccine is immunogenic, about 10% of immunocompetent individuals do not develop high enough levels of anti-HB antibodies to confer immunity despite receiving the recommended 3 doses of the vaccine. To increase the vaccine immunogenicity, they use a lipopolysaccharide derivative as a vaccine adjuvant. The new vaccine induces an exaggerated immune response in previous nonresponders, which results in high antibody titers and, possibly, lifelong protection. Universal vaccination with the new vaccine would most likely lead to eradication of which of the following conditions? Fulminant viral hepatitis Hepatitis C infection Hepatitis D infection Hepatocellular carcinoma Membranous glomerulopathy Mixed cryoglobulinemia

Hepatitis D virus (HDV) is a small, enveloped RNA virus that does not contain a replicase or RNA polymerase. Therefore, the virus is completely reliant on host cell machinery for transcription and translation. In addition, HDV is considered a replication-defective virus because it does not encode for viral envelope proteins, which are necessary to generate viral progeny. HDV can only finish its replication cycle in the setting of coinfection with hepatitis B virus (HBV). HBV encodes for several envelope proteins (HBsAg) that are used to generate the envelope of HDV. Therefore, universal vaccination against HBV would completely eliminate HDV. HDV infection can occur as an acute coinfection with hepatitis B or as a superinfection in a chronic HBV carrier. Infection with HDV dramatically increases the already elevated risk of cirrhosis and hepatocellular carcinoma seen with HBV. (Choice A) Fulminant viral hepatitis is rare but can occur with any of the hepatitis viruses (A, B, C, D, or E). Therefore, vaccination against HBV would not eliminate fulminant viral hepatitis. (Choice B) There is currently no vaccine against hepatitis C virus (HCV). Vaccinating a patient against HBV would not prevent HCV infection. (Choice D) Hepatocellular carcinoma is most commonly associated with HCV or HBV infection. Although universal vaccination against HBV would reduce the incidence of hepatocellular carcinoma, it would not eradicate the condition. (Choices E and F) HBV and HCV can cause extrahepatic manifestations such as membranous glomerulopathy and mixed cryoglobulinemia due to the formation of circulating immune complexes. Although vaccination against HBV would reduce the incidence of these extrahepatic manifestations, it would not completely eliminate them. Educational objective: Hepatitis D virus (HDV) is a replication-defective RNA virus that is capable of causing infection only in the setting of coinfection with hepatitis B virus (HBV). HBV provides hepatitis B surface antigen for the HDV envelope.

A 23-year-old man has 2 days of fever, cough, sore throat, and runny nose. His temperature is 38 C (100.4 F). Lung sounds are clear to auscultation. A nasopharyngeal swab is obtained. Naked viral particles are seen, and purified RNA molecules are extracted from these particles. Once introduced into human cells, the purified RNA molecules induce viral protein synthesis and viral genome replication. Which of the following is the most likely cause of this patient's symptoms? HIV Influenza virus type A Respiratory syncytial virus Rhinovirus Rotavirus

For an RNA molecule purified from a virus (ie, without viral capsid or envelope) to be infectious (ie, able to induce viral protein synthesis and genome replication in the host cell) on its own, it must act as mRNA capable of using the host's intracellular machinery for translation. In other words, the purified RNA molecule must be single-stranded (SS) and positive (+) sense, or SS(+). The question describes a naked (ie, nonenveloped) virus with SS(+) RNA. Of the viruses listed, only rhinovirus (a picornavirus) has these features. Generally, only RNA molecules purified from SS(+) viruses are infectious; purified SS negative (-) sense RNA molecules and double-stranded (ds) RNA molecules are not as they require additional enzymes. (Choice A) HIV is a retrovirus that contains SS(+) RNA. However, viral infectivity requires the action of reverse transcriptase for integration into the host genome; purified RNA by itself cannot cause infection. In addition, HIV is enveloped, unlike the particles isolated from this patient. (Choices B and C) Influenza A is an orthomyxovirus that contains SS(-) RNA; similarly, respiratory syncytial virus is a paramyxovirus that contains SS(-) RNA. For these viruses to replicate in a host cell, an RNA-dependent RNA polymerase must also gain entry into the host cell. (Choice E) Rotavirus is a reovirus containing dsRNA. Its purified RNA is therefore incapable of inducing viral protein synthesis in a host cell on its own (ie, it is non-infectious). For this virus to replicate in a host cell, a specific viral RNA polymerase present in the intact virion must also gain entry into the host cell. Educational objective: For a purified RNA molecule to induce viral protein synthesis in a host cell, it must be able to act directly as mRNA using the host's intracellular machinery for translation. Therefore, in general, purified single-stranded positive-sense RNA can be infectious; single-stranded negative sense or double-stranded RNA is not.

A 21-year-old previously healthy man comes to the office due to fever, malaise, myalgia, and nonproductive cough. His symptoms began 2 weeks after returning from exploring caves in Missouri. The patient does not use tobacco, alcohol, or illicit drugs. Temperature is 38.3 C (100.9 F), blood pressure is 120/70 mm Hg, pulse is 96/min, and respirations are 16/min. Physical examination reveals lung crackles. Chest x-ray shows patchy parenchymal infiltrates with enlarged mediastinal and hilar lymph nodes. Which of the following is most likely to be present in this patient's pulmonary lesions? Budding yeasts with a thick capsule Macrophages with intracellular small yeasts Round spherules with multiple endospores Septate hyphae with dichotomous branching Yeasts with pseudohyphae with blastocondida

Histoplasma capsulatum is a dimorphic fungus that is endemic to the Ohio and Mississippi River valleys. It exists as a filamentous mold in the environment and is found primarily in soil contaminated with bird or bat droppings. Therefore, infected patients often report a history of exploring caves (exposure to bats) or cleaning bird cages or coops. H capsulatum is transmitted when fungal spores from contaminated soil are inhaled into the alveoli. At body temperature, the spores germinate into yeasts and are subsequently phagocytosed by alveolar macrophages. The pathogen eludes macrophage-mediated destruction by preventing acidification of the phagolysosome and initially replicates in an unchecked fashion. Therefore, light microscopy of the affected areas reveals macrophages with intracellular ovoid or round yeasts. Spread to the draining mediastinal and hilar lymph nodes prompts a cell-mediated immune response that helps contain the infection within caseating granulomas (similar to the immune response to Mycobacterium tuberculosis). Patients with intact immune systems usually remain asymptomatic, but a minority develop subacute pneumonia (ie, cough, fever, pleuritic chest pain, pulmonary infiltrates) with hilar and mediastinal lymphadenopathy. In immunocompromised patients (eg, AIDS), infected macrophages can spread the organism throughout the reticuloendothelial system, resulting in widespread disseminated disease. (Choice A) Cryptococcus neoformans appears as a budding yeast with a thick polysaccharide capsule. Although C neoformans is often transmitted from soil contaminated with bird (pigeon) droppings, this pathogen primarily causes central nervous system disease (meningoencephalitis) in patients with severe immunocompromise (eg, advanced AIDS). (Choice C) Coccidioides immitis appears as spherules packed with endospores. Coccidioides can cause pulmonary disease in healthy individuals, but this pathogen is endemic to the southwestern United States deserts (not Missouri) and is not associated with cave exploration. (Choice D) Aspergillus fumigatus appears as septate hyphae with V-shaped branching. Invasive pulmonary aspergillosis is seen primarily in patients with severe immunocompromise (eg, AIDS). (Choice E) Candida species are yeasts that form pseudohyphae and blastoconidia (budded cells off of the pseudohyphae). Patients with intact immune systems rarely develop invasive Candida infections. Educational objective: Histoplasma capsulatum is endemic to the Ohio and Mississippi River valleys and is found primarily in soil contaminated with bird or bat droppings. It exists in tissues as an ovoid/round yeast predominantly within the intracellular space of macrophages. Immunocompetent patients are often asymptomatic but can infrequently develop subacute pneumonia with hilar and mediastinal lymphadenopathy.

A 41-year-old woman comes to the emergency department due to 2 days of severe shortness of breath, high fevers, cough, and myalgias. The patient is a businesswoman who works in the poultry industry. She recently returned from a trip to China, where she visited several factories in different parts of the country. Her temperature is 39.1 C (102.4 F). The patient is in respiratory distress, and crackles and diffuse wheezes are present on pulmonary examination. Laboratory results show that she contracted an orthomyxovirus that was responsible for an upper respiratory infection epidemic in China. Transmission of the virus was originally thought to occur only through contact with poultry, but during the outbreak, sustained human-to-human transmission was observed. Which of the following is most likely responsible for the infectivity of this virus in humans? Antigenic drift Complementation Frameshift mutation Genetic reassortment Phenotypic mixing

Influenza viruses (orthomyxoviruses) are respiratory pathogens that can affect several species, including humans, birds, and swine. They possess the surface proteins hemagglutinin (HA) and neuraminidase (NA), which are virulence factors required for infectivity. These proteins are also targets of the immune system, and neutralizing antibodies against them can confer immunity to specific influenza strains. As such, HA and NA are under constant selective pressure both to maintain species-specific virulence and evade immune recognition. Orthomyxoviruses contain a segmented genome, and HA and NA are coded by separate RNA segments. This allows for genetic reassortment when 2 distinct strains infect the same cell. For instance, avian coinfection with a human influenza A virus (which can also infect birds, the reservoir species for all influenza A subtypes) and an animal influenza A virus can lead to the human-type HA and the animal-type NA being packaged together into the same virion. This has the potential to create a novel strain of virus to which humans are susceptible but have no immunologic resistance. This phenomenon is known as antigenic shift, and is responsible for the majority of pandemics and epidemics of influenza A. (Choice A) Antigenic drift refers to point mutations in HA and NA genes that slightly alter the product proteins, allowing them to evade immune recognition and possibly increasing infectivity of the virus. However, a change in species-to-species transmission likely represents a major modification in protein structure that is better explained by RNA segment reassortment. (Choice B) In genetics, complementation occurs when 2 different strains of a mutant organism are able to produce wild-type offspring. Typically, both parents have homozygous mutations in different genes within the same metabolic pathway. When they are crossed, the offspring inherits 1 normal allele from each parent, allowing them to bypass both of the metabolic blockades and display the wild-type phenotype. (Choice C) Frameshift mutations result when a number of bases that are not a multiple of 3 are added to or deleted from the coding region of a gene. This typically results in premature termination of translation and a truncated protein containing the wrong amino acids. (Choice E) Phenotypic mixing occurs when 2 viruses infect the same cell and progeny viruses exhibit coat or envelope proteins not coded for by the genetic material packaged within them. However, subsequent progeny express only the proteins encoded by their genome. The above scenario is not an example of phenotypic mixing as human-to-human transmission was sustained. Educational objective: Influenza epidemics and pandemics are typically caused by reassortment of the RNA segments coding for hemagglutinin or neuraminidase proteins (major antigenic shifts). This process can occur between human and animal strains of influenza A virus in avian or swine hosts.

A 62-year-old man comes to the emergency department due to sudden-onset of high fever, shaking chills, shortness of breath, and productive cough. The patient has smoked a pack of cigarettes daily for 40 years. Examination shows dullness to percussion over the left side of the chest. Chest x-ray reveals consolidation in the left upper lobe. Gram stain of the sputum shows numerous polymorphonuclear leukocytes and gram-positive cocci in pairs. When cultured on a blood agar plate under aerobic conditions, the bacterial colonies are surrounded by a zone of incomplete hemolysis. The organism responsible for this patient's condition is most likely to exhibit which of the following virulence features? Inhibition of host protein synthesis Motility within host tissue Prevention of phagosome-lysosome fusion Resistance to phagocytosis Survival in extreme environments

Lobar pneumonia typically presents with acute-onset fever, cough, and lobar consolidation on chest x-ray. Although a number of different bacterial pathogens cause lobar pneumonia (eg, Haemophilus influenzae, Moraxella catarrhalis, Staphylococcus aureus), the most common cause is Streptococcus pneumoniae. S pneumoniae is a gram-positive diplococci that exhibits partial (α) hemolysis on blood agar (green colonies) and is bile-soluble and Optochin-sensitive. Its major virulence factor is a thick polysaccharide capsule that encases the organism and prevents phagocytosis and complement binding. Under the microscope, the capsule swells and appears as a halo around the blue-stained bacterial cells when specific anti-capsular antibodies and methylene blue dye are added ("quellung reaction"). Infection with S pneumoniae leads to the generation of anti-capsular antibodies that are protective against future infection with that strain. However, over 90 different antigenic variations (serotypes) of capsular polysaccharides have been identified, so future infection can occur with a strain that has not yet been encountered. Therefore, patients at high risk for invasive pneumococcal disease (eg, HIV, extremes of age) are generally given the pneumococcal vaccine, which induces immunity against the most common strains. Other virulence factors of S pneumoniae include IgA protease (inactivates secretory IgA), adhesins (necessary for adhesion to epithelial cells), and pneumolysin (cytotoxin that causes pores in cell membranes and cell lysis). (Choice A) Certain bacterial exotoxins (eg, Shiga toxin) enter the host cell and cleave a nucleobase from the host ribosome, thereby inhibiting host protein synthesis. Others (eg, diphtheria toxin) prevent protein elongation. S pneumoniae does not directly inhibit host protein synthesis. (Choice B) Motility within host tissue can be provided by flagella. S pneumoniae is a nonmotile organism. (Choice C) Facultative intracellular bacteria such as Mycobacterium tuberculosis and Listeria monocytogenes have virulence factors that prevent the bacteria from being destroyed by the phagolysosome of macrophages. S pneumoniae is not an intracellular pathogen. (Choice E) Spores are resistant to chemical disinfectants, irradiation, desiccation, and temperatures as high as 120 C (248 F). Bacillus and Clostridium species commonly produce spores, but S pneumoniae does not. Educational objective: The primary virulence factor of Streptococcus pneumoniae is a polysaccharide capsule that inhibits opsonization and phagocytosis. The polysaccharide capsule of the most virulent strains is targeted by the pneumococcal vaccine, which confers immunity against those subtypes.

A 34-year-old man comes to the emergency department due to 5 days of progressive anorexia, nausea, and abdominal pain. The patient is a known hepatitis B carrier. He admits to using IV drugs and has shared needles with other drug users on several occasions. His temperature is 37.7 C (99.9 F). Examination shows scleral icterus and mild, tender hepatomegaly. Laboratory studies are notable for highly elevated levels of liver aminotransferases and serum bilirubin and also show the following results: Hepatitis B surface antigen positive Hepatitis C virus antibody negative HIV-1 antibody negative Hepatitis D RNA positive This patient's chronic infection assists which of the following life cycle aspects of the current infectious agent? Coating of viral particles Integration into host genome Intracellular survival Replication of viral RNA Translation of viral transcripts

Often referred to as the delta agent, hepatitis D virus (HDV) is a 35-nm, double-shelled particle that resembles the Dane particle of hepatitis B virus (HBV). The internal polypeptide assembly of HDV is designated hepatitis D antigen (HDAg). Associated with this antigen is a very short, circular molecule of single-stranded RNA. HDAg is considered replication defective as it must be coated by the external coat hepatitis B surface antigen (HBsAg) of HBV to penetrate the hepatocyte. HDV infection can arise either as an acute coinfection with HBV or as a superinfection of a chronic HBV carrier. (Choices B, C, D, and E) Once coated with HBsAg, HDV is able to penetrate the hepatocyte, survive within the cell, integrate into the host genome, replicate its viral RNA, and translate its genome into protein. Educational objective: The hepatitis B surface antigen of hepatitis B virus must coat the hepatitis D antigen of hepatitis D virus before it can infect hepatocytes and multiply.

An isolate of Streptococcus pneumoniae from a patient with meningitis is incubated with low-dose radiolabeled ceftriaxone and then subjected to protein electrophoresis. Five distinct bands are detected by radioautography. These bands most likely represent radiolabeled ceftriaxone that is bound to which of the following? Cell wall glycoproteins DNA replication proteins Porins Ribosomal proteins Transpeptidases

Penicillins and cephalosporins function by irreversibly binding to penicillin-binding proteins. Transpeptidases are one form of penicillin-binding protein that function to cross-link peptidoglycan in the bacterial cell wall. Inhibition of transpeptidase (which occurs with the use of the cephalosporin ceftriaxone) leads to cell wall instability and bacteriolysis. Many bacterial species synthesize multiple different penicillin-binding proteins; the laboratory study described in this question stem identifies 5 such proteins that are produced by this isolate of Streptococcus pneumoniae. (Choice A) Vancomycin binds to the terminal D-alanine residues of cell wall glycoproteins and prevents transpeptidases from forming cross-links. (Choice B) Fluoroquinolones interfere with DNA replication by binding to proteins such as DNA gyrase. (Choice C) Porins are utilized by bacterial organisms as a means of controlling diffusion of small to medium-sized metabolites. Antibiotic resistance can develop through porin mutations that prevent antibiotic diffusion through the outer membrane of gram-negative bacteria. No known antibiotics function by binding to porins. (Choice D) Ribosomal proteins are bound by multiple antibiotics including macrolides, tetracyclines, and others. Educational objective: Penicillins and cephalosporins function by irreversibly binding to penicillin-binding proteins such as transpeptidases.

A previously healthy 2-year-old boy is brought to the clinic with fever and mouth pain that began yesterday. He has consumed an adequate amount of fluids but refuses to eat due to the pain. The patient has no medical problems and takes no medications. Physical examination reveals swollen gums and vesicular, inflamed lesions on his hard palate and lips. He has enlarged and tender cervical lymph nodes. Which of the following is most likely responsible for this patient's condition? DNA virus, double-stranded, enveloped DNA virus, double-stranded, non-enveloped DNA virus, single-stranded, non-enveloped RNA virus, double-stranded, positive-sense RNA virus, single-stranded, negative-sense RNA virus, single-stranded positive sense

Primary herpes simplex virus type 1 (HSV-1) infection is characterized by the gingivostomatitis (vesicles on the lips and hard palate, fever, lymphadenopathy) seen in this young patient. Recurrent HSV-1 infection causes lip lesions that are typically less severe due to existing cellular and humoral immunity from prior infection. HSV-1 and other herpesviruses are enveloped and possess double-stranded DNA genomes. Transmission is usually by direct contact with infected oral secretions or lesions. Inoculation occurs on mucosa or skin where the virus replicates rapidly in the host cell nucleus and causes abnormal cell division, resulting in intranuclear inclusion bodies and multinucleated giant cells visible on Tzanck smear. The vesicular lesions result from cell lysis and necrosis as well as tissue destruction, inflammation, and fluid accumulation between the dermis and epidermis. (Choice B) Most DNA viruses are double-stranded. They are enveloped (ie, surrounded by a lipid layer) or non-enveloped (ie, lacking a surrounding lipid layer). Adenovirus is a non-enveloped, double-stranded DNA virus that typically causes self-limited upper respiratory infection or gastroenteritis. (Choice C) Parvovirus is a single-stranded, non-enveloped DNA virus that causes erythema infectiosum, or fifth disease. Patients classically present with fever, cough, and rhinorrhea followed by a "slapped-cheek" and lacy, reticular truncal rash. (Choice D) Rotavirus, a type of reovirus, is a double-stranded, positive-sense, non-enveloped RNA virus. Rotavirus infection is a vaccine-preventable disease characterized by profuse, watery diarrhea in children. (Choices E and F) Most RNA viruses are single-stranded and can be classified as negative- or positive-sense viruses. Positive-sense viruses can be directly translated into proteins, but negative-sense stranded viruses must first have their RNA converted into positive-sense strands before translation. Coxsackievirus, a single-stranded, positive-sense RNA virus of the enterovirus family, causes hand, foot, and mouth disease, which presents with vesicular lesions on the buccal mucosa, tongue, and soft palate. The gums and hard palate are usually involved in HSV gingivostomatitis, not hand-foot-mouth disease. Educational objective: Primary herpes simplex virus type 1 (HSV-1) infection in children causes gingivostomatitis (vesicular lesions on the lips and hard palate). HSV-1 and other herpesviruses are double-stranded, enveloped DNA viruses.

A 52-year-old woman comes to the office due to worsening skin lesions on her right hand. The patient injured her hand while cleaning decaying vegetation from her rose garden 3 weeks ago. She washed the wound with soap and water and noticed a mildly itchy papule several days later. The lesion progressively enlarged, and 2 other similar papules appeared proximal to the original lesion. The patient has had no fever, chills, or severe pain. She has no previous medical conditions and takes no medications. The patient lives in the southeastern United States and has not traveled recently. Physical examination shows a 1-cm, nontender, red nodule on the dorsum of the right hand with 2 smaller papules in linear distribution proximal to it. Which of the following histopathologic findings is most likely present in this patient's skin lesions? Dense inflammatory infiltrate surrounding sulfur granules Granulomas with amastigote parasites inside dermal macrophages Mixed granulomatous and neutrophilic inflammatory reaction Perivascular lymphocytic inflammation and epidermal spongiosis Prominent plasma cell infiltrates and obliterative endarteritis

Sporotrichosis is caused by Sporothrix schenckii, a dimorphic fungus found in soil and plant matter that may be traumatically inoculated into the skin (eg, thorn prick) during outdoor activities (eg, gardening, landscaping). A mildly pruritic, erythematous, ulcerated papule often forms at the inoculation site (eg, hand, forearm) within weeks. As the organism spreads, additional lesions may develop along the draining lymphatic chain (ie, nodular lymphangitis). Biopsy of the lesions reveals mixed inflammation, including granulomas and neutrophilic microabscesses in the dermis and subcutaneous tissue. Stains for fungal organisms may highlight rare, cigar-shaped yeast. The gold standard for definitive diagnosis is culture. Sporotrichosis is typically limited to the skin and soft tissue, but disseminated infection may develop in immunocompromised individuals. Itraconazole is the preferred treatment for lymphocutaneous sporotrichosis. (Choice A) Histologic features of actinomycosis include sulfur granules (aggregates of Actinomyces filaments) surrounded by a dense inflammatory infiltrate. Actinomycosis most commonly presents as a firm cervicofacial (eg, mandibular) mass with abscess and sinus tract formation after surgery (eg, tooth extraction) or trauma. (Choice B) Cutaneous leishmaniasis is histologically characterized by granulomatous inflammation and intracellular amastigote organisms within macrophages. Although it may present with ulcerated papules and nodules, cutaneous leishmaniasis is most frequently seen in the Middle East and Central and South America; this patient lives in the southeastern United States and has not traveled recently. (Choice D) Spongiosis (ie, epidermal intercellular edema) and superficial dermal perivascular inflammation are histologic features of acute eczematous dermatitis, such as allergic contact dermatitis (eg, poison ivy). Clinical manifestations typically include erythematous, pruritic papules and vesicles, rather than nodular lesions. (Choice E) Prominent plasma cell infiltrates and obliterative endarteritis are characteristic of syphilitic lesions. Depending on the stage of infection, cutaneous syphilitic lesions may include painless ulcers (eg, chancre), gray-white plaques (eg, condylomata lata), a maculopapular rash, and gummas. Educational objective: Sporotrichosis typically presents as papulonodular lesions distributed along the lymphatics and is histologically characterized by granulomatous and neutrophilic inflammation. It is caused by the dimorphic fungus Sporothrix schenckii, which is often inoculated into the skin during outdoor activities such as gardening.

A 22-year-old man comes to the office due to a painful lesion on his upper back. The patient developed a small tender papule a few days ago that progressively enlarged and became red and painful. He has had 3 similar lesions on his upper back over the past 6 months, which were treated with incision and drainage and antibiotics. The patient has no other medical issues and takes no medications. He does not use tobacco, alcohol, or illicit drugs. On physical examination, there is a 3-cm tender pustule with surrounding erythema on his right upper back. Gram staining of purulent drainage from the lesion is shown below. Purple cocci Which of the following most likely predisposed this patient to recurrent infections? Colonization in anterior nares Dormant infection in granulomas Formation of elementary bodies Harborage in epidermal burrows Spores from contaminated surfaces

Staphylococcus aureus is a gram-positive bacterium that grows in grape-like clusters. Approximately 30% of individuals in the community-setting are colonized with S aureus. The pathogen is transmitted by direct contact, colonizes the anterior nares, and spreads to the skin. Defects in the mucosal or cutaneous barrier can lead to invasion. The most common manifestation is skin and soft-tissue abscess; patients usually develop a localized pustule that evolves over days into a painful nodule with a necrotic, purulent center and surrounding indurated erythema. Treatment with incision/drainage and oral antibiotics is usually curative of the localized infection. However, oral antibiotics do not eliminate S aureus colonization of the nares and skin so recurrent infections are common. Colonization with methicillin-sensitive S aureus (MSSA) is associated with a low risk of illness. In contrast,methicillin-resistant S aureus (MRSA) colonization is much more likely to cause disease due to expression of associated virulence factors (eg, leukocidin). (Choice B) Latent Mycobacterium tuberculosis is characterized by the presence of organisms in walled-off granulomas. Patients with latent tuberculosis are asymptomatic and most will never have reactivation of disease. Reactivation is classically characterized by the formation of apical, cavitary lesions in the lung, weight loss, and cough. (Choice C) Chlamydia trachomatis is an obligate intracellular organism that invades as a small elementary body and then evolves to the metabolically active reticulate body once inside host cells. Recurrent Chlamydia symptoms (eg, dysuria, purulent urethra discharge) in patients who have been adequately treated (eg, azithromycin) usually indicates a new infection due to continued high-risk behavior (eg, unprotected sex). (Choice D) Scabies is a skin mite (Sarcoptes scabiei) that causes intensely pruritic eruptions, particularly of the finger webs, wrists, axilla, areola, and genitalia. The mite burrows into the epidermis and lays eggs. Topical treatment with permethrin is usually curative, but recurrence can occur if close contacts and environmental harbors (eg, sheets) are not adequately treated. (Choice E) Clostridium difficile spores are hardy and resistant to heat and soap. Infection usually causes colitis with watery diarrhea, fever, and abdominal pain. Recurrent infection after treatment with oral antibiotics (eg, vancomycin) is common and may require a repeat pharmacologic treatment or, in some cases, fecal transplant. Educational objective: Staphylococcus aureus is the most common cause of skin and soft-tissue abscess (eg, furuncle). Treatment of the abscess with incision and drainage and antibiotics can eliminate the local infection but does not eliminate colonization of the anterior nares and skin. Therefore, recurrent infections are common.

A 30-year-old man comes to the emergency department due to a week of fever, chills, and generalized malaise. The patient uses intravenous heroin and has been hospitalized several times due to skin and soft tissue infections. Cardiac auscultation shows a systolic ejection murmur, and subsequent echocardiography reveals a vegetative lesion of the aortic valve. Blood cultures from admission grow Enterococcus faecalis; subsequent testing reveals D-lactate rather than D-alanine as the terminal amino acid in the bacteria's pentapeptide peptidoglycan cell wall precursor. This substitution is likely to significantly decrease the efficacy of which of the following antibiotics? Ceftriaxone Fosfomycin Penicillin Polymycin Tetracycline Vancomycin

The bacterial cell wall is composed of cross-linked peptidoglycan subunits on a backbone of N-acetylmuramic acid (NAM) and N-acetylglucosamine (NAG). Because cell wall disruption results in bacterial lysis, many antibiotics target cell wall synthesis, including the following: Fosfomycin blocks development of the NAM-NAG polymer backbone by inhibiting the enzyme MurA. Resistance stems primarily from efflux pumps or alterations to MurA that prevent fosfomycin binding (Choice B). Vancomycin inhibits transpeptidase from cross linking a growing peptidoglycan chain by binding to the D-alanine-D-alanine terminus of pentapeptide peptidoglycan subunits. Resistance to vancomycin stems primarily from replacement of the terminal D-alanine-D-alanine with D-alanine-D-lactate, which reduces vancomycin binding >1000-fold. Penicillin and cephalosporins (eg, ceftriaxone) are beta-lactam antibiotics that bind to and directly inhibit transpeptidase. Resistance stems primarily from alterations to transpeptidase that reduce beta-lactam binding or the production of enzymes (eg, penicillinase) that break down the drug (Choices A and C). (Choice D) Polymyxin binds to lipopolysaccharide, leading to disruption of the bacterial cytoplasmic membrane. It does not interfere with the bacterial cell wall. (Choice E) Tetracycline binds to the 30S ribosomal subunit and prevents binding of aminoacyl-transfer RNAs. Resistance stems primarily from an efflux pump or alteration to the ribosome that allows translation to take place even when tetracycline is present. Educational objective: Vancomycin binds to the terminal D-alanine-D-alanine in bacterial pentapeptide peptidoglycan subunits, which prevents transpeptidase from binding to the pentapeptide, thereby inhibiting cell wall cross-linking. Resistance stems primarily from substitution of the terminal D-alanine with D-lactate, which prevents vancomycin binding.

A 23-year-old woman participates in a research study evaluating the effect of monoclonal antibodies on viral infection. The patient has no past medical history and takes no medications. She has not been ill recently. During the study, a peripheral blood specimen is obtained. CD19 positive cells are isolated, purified, and exposed to monoclonal antibodies against cell surface complement receptor CD21. The cells are subsequently incubated along with several viruses being studied. Initial exposure to monoclonal antibodies against CD21 is most likely to prevent cell infection with which of the following viruses? Adenovirus Cytomegalovirus Epstein-Barr virus Human immunodeficiency virus Parvovirus B19

The initial attachment of the virion envelope or capsid surface proteins to the complementary host cell surface receptors is essential to viral tropism for specific tissues and invasion of cells. Many viruses bind to normal host cell plasma membrane receptors to enter host cells. Epstein-Barr virus (EBV) is a herpesvirus responsible for acute infectious mononucleosis, nasopharyngeal carcinoma, and certain lymphomas (eg, Burkitt lymphoma). More than 90% of the normal adult population is seropositive for EBV, which is primarily transmitted through contact with oropharyngeal secretions. The EBV envelope glycoprotein gp350 binds to CD21 (also known as CR2), the cellular receptor for the C3d complement component. CD21 is normally present on the surface of B cells (CD19-positive cells) and nasopharyngeal epithelial cells. Therefore, exposure to a monoclonal anti-CD21 antibody could interfere with EBV attachment to B cells. (Choice A) Adenovirus has hexon and penton capsomeres on its surface. Rodlike structures ("fibers") that project from the penton base capsomeres are responsible for mediating adsorption to host cells. The cell receptor for most adenovirus fibers is a transmembrane protein member of the immunoglobulin superfamily. (Choice B) Like other herpesviruses, cytomegalovirus requires initial contact with glycosaminoglycan chains on host cell surface proteoglycans for entry into the host cell. It does not require contact with CD21. (Choice D) HIV viruses attach to their major target host cells (CD4+ T cells) primarily via the binding of viral envelope glycoprotein gp120 to the cellular CD4 transmembrane glycoprotein and the coreceptor (CCR5 or CXCR4). The HIV envelope then undergoes a conformational change that activates gp41 and initiates membrane fusion. (Choice E) Parvovirus B19 is thought to attach to human erythroid cells via the blood group P antigen (globoside), which is expressed by mature erythrocytes, erythroid progenitors, megakaryocytes, placenta, and the fetal liver and heart. Immature cells of the erythroid family (eg, adult bone marrow, fetal liver) are most vulnerable to parvovirus B19 infection. Educational objective: The initial attachment of the virion envelope or capsid surface proteins to the complementary host cell surface receptors is essential to viral tropism for specific tissues and invasion of cells. Many viruses bind to normal host cell plasma membrane receptors to enter host cells. Known host cell receptor and virion/virion protein binding specificities include: CD4 with HIV gp120, CD21 with Epstein-Barr virus gp350, and erythrocyte P antigen with parvovirus B19.

Another isolate of Streptococcus pneumoniae from a patient with meningitis is also incubated with low-dose radioactive ceftriaxone and subjected to protein electrophoresis. Only two bands are detected using radioautography (originally 5). Which of the following best explains the observed finding? Action of beta-lactamases Change in protein structure Salvage metabolic pathway Transmembranous efflux pumps Upregulation of protein synthesis

The isolate of S. pneumoniae in the prior question had five penicillin-binding proteins identified, whereas the isolate in the current question only has two. This indicates that three of the penicillin-binding proteins have been altered in such a way that inhibits the binding of ceftriaxone. Structural changes in penicillin-binding proteins that prevent ceftriaxone from binding is one mechanism for ceftriaxone resistance. (Choice A) Beta-lactamases function to degrade penicillin and cephalosporins. Production of a beta-lactamase would prevent ceftriaxone from being able to bind to the penicillin-binding proteins. Because ceftriaxone is so much smaller than the penicillin-binding proteins, unbound ceftriaxone would likely accumulate at one of the electrodes. This would lead to no bands being found within the electrophoresis area. (Choice C) A salvage metabolic pathway can restore metabolic function that would otherwise be inhibited by an antibiotic such as trimethoprim or sulfamethoxazole. (Choice D) Transmembranous efflux pumps confer resistance to antibiotics that require entry into the cell to function (e.g., tetracyclines and macrolides). However, penicillins and cephalosporins function outside of the bacterial cell at the peptidoglycan cell wall. (Choice E) Upregulation of protein synthesis can occur to overcome the effect of an antibiotic that binds to and irreversibly inhibits the action of that protein. However, if this were the mechanism for resistance, five bands would still be identified on the radioautography test. Educational objective: A change in the structure of penicillin-binding proteins that prevents cephalosporin binding is one mechanism of bacterial resistance to cephalosporins.

A 72-year-old man is brought to the emergency department from a skilled nursing facility due to altered mental status, skin flushing, and high fever. He uses a wheelchair for mobility since a stroke several years ago. Temperature is 38.9 C (102 F), blood pressure is 80/40 mm Hg, pulse is 120/min, and respirations are 26/min. A blood sample is obtained, and culture is positive for bacterial growth within a few hours. Multiplex PCR testing of the blood detects Escherichia coli. Which of the following is the most likely source of bacteremia in this patient? Diverticulities Intestinal perforation Pneumonia Pressure ulcer Urinary tract infection

This elderly patient with fever, hypotension, and altered mental status has sepsis. The presence of Escherichia coli bacteremia suggests the source is a urinary tract infection (UTI). E coli is a normal gastrointestinal commensal organism. UTI arises when the organism contaminates the periurethral area and uses special adhesive proteins called pili to attach to and ascend the uroepithelium. Predisposing factors include urinary obstruction (eg, benign prostatic hypertrophy), fecal incontinence, neurogenic bladder (eg, due to diabetes), and indwelling catheterization. E coli is the leading cause of UTIs, and most cases of E coli bacteremia are due to hematologic dissemination from the urinary system. Bloodstream infection can lead to sepsis due to overwhelming release of inflammatory cytokines; clinical manifestations include hyper/hypothermia, tachycardia, tachypnea, leukocytosis, and organ dysfunction (eg, confusion, acute kidney failure). (Choice A) Diverticulitis can cause E coli bacteremia due to spread of the organism through the colon wall. However, patients usually experience constant, worsening abdominal pain over several days. This patient is more likely to have a UTI, which frequently causes abrupt mental status changes in elderly patients. (Choice B) Intestinal perforation can occur as a consequence of surgery, trauma, or wall rupture from ischemia; it often causes septic shock and anaerobic or gram-negative rod bacteremia. This patient is less likely to have intestinal perforation than a UTI. (Choice C) Patients with a history of stroke are at risk for pneumonia due to aspiration. However, E coli is an atypical cause of pneumonia in those who are not intubated. (Choice D) Wheelchair use may increase risk of pressure ulcers. Although E coli can contaminate a pressure ulcer, most skin infections are caused by gram-positive organisms (eg, Staphylococcus aureus, Streptococcus species). This patient is far more likely to have E coli bacteremia due to a UTI. Educational objective: Escherichia coli is the most common cause of urinary tract infection (UTI) in both healthy adults and elderly patients. E coli is part of the normal gastrointestinal flora, and special adhesive proteins (pili) allow some strains to colonize and ascend the urinary tract, causing UTI, pyelonephritis, and bacteremia/sepsis. UTIs are the most common cause of E coli bacteremia.

A 32-year-old man comes to the office for a follow-up appointment. The patient recently returned from a 10-day trip to Honduras. On the fifth day of his trip, he developed malaise, anorexia, and abdominal cramps, quickly followed by watery diarrhea. The patient had 5 or 6 stools daily but no fever. He took no medications and remained hydrated by drinking bottled water and soup. The patient felt better after 2 days and has had no additional symptoms since. He has no other medical conditions. Vital signs are within normal limits and physical examination shows no abnormalities. Which of the following factors was most likely involved in the pathogenesis of this patient's gastrointestinal symptoms? Bacterial endotoxin Cholera-like enterotoxin Intestinal biofilm production Intestinal mucosal invasion Shiga toxin

This patient had self-limited traveler's diarrhea, which is most commonly caused by enterotoxigenic Escherichia coli (ETEC), a gram-negative, motile, enteric rod bacterium. Patients traveling to resource-limited regions with poor sanitation are at greatest risk. Following ingestion, ETEC colonizes and adheres to small intestine enterocytes (mediated by pili). The typical manifestations of abdominal cramping, nausea/vomiting, and (occasionally) low-grade fever occur due to the elaboration of plasmid-encoded heat-labile (LT) and/or heat-stable (ST) enterotoxins: The LT enterotoxin, which resembles cholera toxin in structure and mode of action, increases cyclic AMP in gut mucosal cells by activating adenylate cyclase. The ST enterotoxin (which is not inactivated by heat likely due to its small molecular size) causes an increase in cyclic GMP by activating guanylate cyclase. Both mechanisms result in increased chloride secretion and decreased sodium reabsorption by enterocytes, resulting in watery diarrhea. (Choice A) Endotoxin is a lipopolysaccharide found in the cell membrane of gram-negative bacteria. When released into the bloodstream, endotoxin causes a severe inflammatory response (eg, fever, hypotension) mediated by TNF-alpha and IL-1 secreted from activated macrophages. However, endotoxin is present at high concentrations in the normal gut microbiota and does not cause gastrointestinal illness. (Choice C) Enteroaggregative E coli is a less common cause of self-limited watery diarrhea in resource-limited regions; adhesion of the organisms to intestinal mucosa in a "stacked-brick" pattern is characteristic. Biofilm formation can occur but is thought to primarily play a role in cases of persistent (not acute) disease. (Choice D) Intestinal invasion is characteristic of gastroenteritis caused by Salmonella, Shigella, and Campylobacter jejuni, as well as the protozoan Entamoeba histolytica. Due to mucosal cell necrosis, invasion frequently leads to bloody diarrhea (not seen here). (Choice E) Shiga toxin-producing E coli (STEC) and Shigella dysenteriae produce Shiga toxin that inactivates the 60S ribosomal subunit in human cells, thereby inhibiting cell protein production. Bloody diarrhea due to mucosal cell death is typical. Educational objective: Traveler's diarrhea is most frequently due to enterotoxigenic Escherichia coli, which produces plasmid-encoded, heat-labile (LT) and heat-stable (ST) enterotoxins. LT resembles cholera toxin and activates adenylate cyclase, leading to increased intracellular cyclic AMP; ST activates guanylate cyclase, leading to increased intracellular cyclic GMP. Both can contribute to watery diarrhea due to loss of water and electrolytes.

A 61-year-old man comes to the office due to a painful rash on his face for the past 3 days. The patient has no other medical conditions and takes no medications. He is a retired schoolteacher and lives with his wife. Skin examination findings are shown in the exhibit. The remainder of the physical examination shows no abnormalities. Which of the following events most likely immediately preceded the appearance of skin rash in this patient? Alteration in the patient's normal skin flora Close-physical contact with an infected person Inhalation of pathogen-containing aerosols Reactivation of latent organism in sensory ganglia Waning of organism-specific antibodies

This patient has a crusted vesicular rash in a dermatomal distribution consistent with shingles (herpes zoster). Shingles is caused by reactivation of varicella-zoster virus (VZV), a double-stranded DNA virus of the herpesvirus family. Primary VZV infection occurs most commonly in childhood. Inoculation in the nasopharyngeal mucosa is followed by migration of the virus to the regional lymphatics. Subsequent dermatotrophic migration produces a diffuse vesicular rash associated with fever, malaise, and pharyngitis (chickenpox). Then, the virus migrates via sensory nerves to cranial nerve and dorsal spinal ganglia, where it lies dormant for decades. Most patients who have had primary chickenpox do not develop recurrent diffuse disease. Over time, however, waning cell-mediated immune function allows reactivation of the virus, which typically spreads down a single spinal nerve to cause a painful, erythematous, vesicular rash in a dermatomal distribution. The primary risk factor for reactivation is increasing age; other factors can include malignancy, autoimmune disease, and immune-suppressing therapy. The varicella vaccine is recommended for prevention of primary disease in children, women of childbearing age, adults with sustained risk of exposure, and household contacts of immunocompromised hosts. Zoster vaccine is recommended for adults age ≥50 and reduces the risk of reactivation. (Choice A) Colonization of the skin by transient flora (eg, Staphylococcus aureus, Candida species, gram-negative bacilli) occasionally leads to clinical disease, especially following antibiotic exposure (eg, vaginal candidiasis). However, VZV does not remain on the surface of the skin for prolonged periods. (Choices B and C) Primary VZV infection (ie, chickenpox) can be acquired by direct contact or airborne aerosol from patients with active chickenpox or zoster lesions. However, primary VZV infection causes diffuse lesions; lesions in a dermatomal distribution are seen only with reactivation from sensory ganglia. (Choice E) Declining VZV-specific immunity increases the risk of reactivation. However, the immune response to VZV is primarily based on cell-mediated, rather than humoral (ie, antibodies), factors. Educational objective: Primary varicella-zoster virus infection (chickenpox) occurs most commonly in childhood. Subsequently, the virus migrates to the sensory ganglia, where it lies dormant for decades. Over time, waning cell-mediated immunity allows reactivation of the virus, which spreads down a single nerve to cause a painful, erythematous, vesicular rash in a dermatomal distribution.

A 46-year-old man comes to the office due to skin lesions on his left hand and arm that developed over the past several days. He first noticed a small papule on his left hand that has increased in size. Additional nodules then appeared proximally on his arm. The lesions are not particularly painful. The patient has never had similar symptoms. He has had no fever and otherwise feels fine. He has no significant medical history, takes no medications, and has no allergies. Temperature is 37.1 C (98.8 F). The lesions are shown in the image below. ID=270 red pimples ascending from hand to arm Fluid aspirated from a lesion is examined microscopically, and several elongated yeast cells with narrow-based buds are visualized. Which of the following risk factors is most likely a component of this patient's history? Bat guano environmental exposure Contact with soil and plant materials Exposure to seawater Prolonged antibiotic use Recent flea bite

This patient has a series of nodular lesions that progressed proximately from a distal extremity lesion, indicative of nodular lymphangitis. Although a handful of fairly uncommon pathogens (eg, Mycobacterium marinum, Nocardia, Leishmania) can cause nodular lymphangitis, the most likely pathogen in this case is Sporothrix schenckii. S schenckii is a dimorphic fungus that exists as cigar-shaped yeasts at body temperature and reproduces by narrow-based budding. It exists as a mold in the environment and tends to grow in decaying plant matter and soil, particularly on sphagnum moss, hay, and decaying wood. Transmission typically occurs when the organism is introduced into cutaneous or subcutaneous tissue while a person is farming, landscaping, or gardening. Manifestations of infection progress slowly over days or weeks and usually begin with a painless papule at the site of inoculation that eventually ulcerates; notably, the lesion does not drain pus or odorous material. Similar lesions arise over several days more proximally along the lymphatic channel. Systemic symptoms (eg, fever) do not generally occur. Most cases can be diagnosed clinically, but culture of a sample from a lesion can definitively identify the organism. (Choice A) Bat guano can carry Histoplasma capsulatum, which typically causes a mild, subacute pulmonary infection. Nodular lesions can develop as part of disseminated infection, but lesions are usually bilateral and do not exhibit proximal spread. In addition, disseminated infections are almost always seen in severely immunocompromised patients (eg, AIDS). (Choice C) Mycobacterium marinum is an uncommon pathogen that causes infection following inoculation of a wound with contaminated freshwater or seawater. Most cases are marked by a solitary papule that ulcerates, but nodular lymphangitis can occasionally occur. However, acid-fast bacilli, not yeasts, would be seen on culture. (Choice D) Prolonged antibiotic use can disrupt mucocutaneous flora, leading to Candida infection. Although Candida is a yeast that buds, it generally causes intertriginous infection marked by erythematous plaques and erosions. Nodular lymphangitis would be uncommon. (Choice E) Feline flea bite can transmit Bartonella henselae, the cause of catscratch disease. Patients often develop a papule at the inoculation site, but the major symptom is extremely tender regional lymphadenitis. Gram-negative rods, not yeasts, would be seen on culture. Educational objective: Sporothrix schenckii is a dimorphic fungus that exists as yeasts at body temperature. It is found in decaying plant matter and soil and is usually transmitted via cutaneous or subcutaneous inoculation while a person is farming, gardening, or landscaping. The major manifestation is a slowly progressive nonpurulent, nonpainful, nodular lymphangitis that progresses proximally along the lymphatic chain.

An 84-year-old woman is sent to the hospital from a nursing home due to 3 days of fever, abdominal pain, and progressive lethargy. The patient has a history of recurrent hospitalizations with multiple antibiotic courses for urinary tract infection, infected decubitus ulcer, and pneumonia. Her temperature is 38.2 C (101 F), blood pressure is 100/60 mm Hg, and pulse is 110/min. She has mild suprapubic and right costovertebral angle tenderness. Urine and blood cultures are obtained, and the patient is started on intravenous fluids and empiric ceftriaxone. Her urine culture shows extended-spectrum beta-lactamase (ESBL)-producing Escherichia coli. Which of the following features is most likely to be found in this organism? DNA gyrase with altered A subunit Outer membrane with novel porin subunit Penicillin-binding protein with reduced antibiotic affinity Peptidoglycan cell wall with amino acid modification Plasmid with drug resistance gene Ribosome with methylated 16S rRNA

This patient has a urinary tract infection caused by an extended-spectrum beta-lactamase (ESBL)-producing Escherichia coli. ESBL enzymes can inactivate extended-spectrum penicillins and cephalosporins (including third- and fourth-generation cephalosporins) and monobactams (aztreonam). Genes encoding these enzymes are often located on plasmids and therefore can be transferred between organisms and between different species through conjugation. Some have been incorporated into bacterial chromosomes. Carbapenems (eg, imipenem) are the treatment of choice for ESBL-producing organisms (although unfortunately, organisms with carbapenem resistance have emerged). (Choice A) Mutations in DNA gyrase can lead to fluoroquinolone resistance and generally do not impact beta-lactam antibiotic activity. (Choice B) Porin mutations can confer penicillin and aminoglycoside resistance and are particularly important in gram-negative organisms such as Pseudomonas. (Choice C) Penicillin-binding protein alterations are the mechanism that leads to methicillin resistance in Staphylococcus aureus. (Choice D) Vancomycin inhibits glycopeptide polymerization through binding to D-alanyl-D-alanine in the cell wall precursor; peptidoglycan changes (eg, D-alanine-D-lactate) can develop as a resistance mechanism. (Choice F) Ribosome mutations can confer resistance to ribosome-targeting antibiotics such as aminoglycosides and macrolides. Educational objective: Extended-spectrum beta-lactamases can be produced by gram-negative bacteria, rendering cephalosporins and other beta-lactam antibiotics inactive. These genes can be transmitted between organisms through plasmid conjugation.

An 18-year-old man comes to the emergency department due to 3 days of fever, myalgias, and profound fatigue. He was feeling fine prior to the onset of symptoms. The patient has a history of intermittent asthma for which he uses an inhaler as needed. He is a senior in high school and plays on the football team. Temperature is 38 C (100.4 F), blood pressure is 110/80 mm Hg, pulse is 88/min, and respirations are 16/min. BMI is 22.1 kg/m2. On physical examination, the patient is found to have swollen lymph nodes in his neck and behind his ears. The tip of the spleen is palpable 1 cm below the left costal margin. Laboratory results are notable for the following: Complete blood count Hemoglobin 14.4 g/dL Platelets 310,000/mm3 Leukocytes 11,000/mm3 Lymphocytes 70% Which of the following is most likely to be elicited on further history? Anal sexual intercourse Contact with saliva Exposure to urine Prior blood transfusion Recent tick bite School trip to South Asia

This patient has many of the classic features of Epstein-Barr virus (EBV)-induced mononucleosis, including fever, profound fatigue, lymphadenopathy, lymphocytosis (with atypical lymphocytes), and splenomegaly (palpable spleen tip). EBV is typically transmitted from an asymptomatic virus shedder to a susceptible individual through saliva transfer (eg, kissing). (Choice A) Anal sexual intercourse is more likely to transmit pathogens such as HIV, Neisseria gonorrhoeae, and Chlamydia trachomatis. (Choices C, D, and E) Urine can harbor pathogens such as cytomegalovirus, adenovirus, Escherichia coli, Staphylococcus saprophyticus, and Klebsiella pneumoniae; blood can harbor hepatitis B virus, hepatitis C virus, and HIV; and arthropods (eg, ticks) can harbor Rickettsia rickettsii and Borrelia burgdorferi, among others. (Choice F) Malaria and leishmaniasis, which can cause splenomegaly, are found in South Asia. However, malaria would classically be associated with anemia and thrombocytopenia. Visceral leishmaniasis (kala-azar) manifests as progressive splenic enlargement and weight loss over months. Educational objective: Typical clinical and laboratory features of Epstein-Barr virus (EBV) mononucleosis include fever, pharyngitis, lymphadenopathy, splenomegaly, and atypical lymphocytosis. EBV is generally transmitted from an asymptomatic virus shedder to a susceptible individual through saliva transfer (eg, kissing).

A 59-year-old woman is brought to the emergency department with fever, skin flushing, and an altered level of consciousness. Her blood pressure is 50/20 mmHg, and her heart rate is 120/min despite receiving IV fluids. If blood cultures are positive for E. coli, which of the following bacterial factors is most directly responsible for this patient's current condition? Capsule Fimbrial antigen Heat-stable exotoxin Lipid A O antigen

This patient has septic shock (ie, fluid-refractory hypotension in a patient with sepsis) as a result of the release of bacterial endotoxins into the bloodstream. Endotoxins are found in the outer membrane of Gram-negative bacteria (eg, E. coli), which is composed of lipopolysaccharide (LPS). LPS is released by destruction of the bacterial cell wall or during cell division. LPS is a very long, heat-stable molecule arranged into three regions: O antigen, core polysaccharide, and Lipid A. Lipid A is toxic to mammalian cells and acts as a virulence factor by inducing the release of endogenous pyrogens (eg, IL-1, prostaglandins) and inflammatory mediators (eg, TNFa, interferon) from activated macrophages. These cytokines cause the manifestations of septic shock: fever, hypotension, increased vascular permeability with third-spacing of fluids, and organ failure. (Choice A) E. coli strains can produce a capsule (eg, K-1 antigen) which provides a defense against phagocytosis by immune cells but does not directly contribute to the manifestations of septic shock. (Choice B) Fimbriae, or pili, allow bacteria to adhere to the target tissue and establish infection. Although Enterotoxigenic E. coli is an example of an organism that uses fimbriae, bacterial adhesion is not directly responsible for the manifestations of septic shock. (Choice C) Heat-stable exotoxin is one of the enterotoxins produced by Enterotoxigenic E. coli but is responsible for secretory diarrhea rather than the manifestations of septic shock. (Choice E) Although the O antigen is part of the lipopolysaccharide of a gram-negative bacteria and can be used for identification of different strains, it is not directly involved in the manifestations of septic shock. Educational objective: Gram-negative sepsis is caused by the release of lipopolysaccharides from bacterial cells during cell division or bacteriolysis. Lipid A is the toxic component of lipopolysaccharides and induces the widespread release of IL-1 and TNF-alpha from activated macrophages. These cytokines cause the signs and symptoms of septic shock (eg, fever, hypotension, organ dysfunction).

A 43-year-old man comes to the emergency department due to 2 weeks of intermittent fevers, malaise, and headaches. He has a history of HIV and has not been taking his antiretroviral drugs recently. Five years ago, the patient spent a year in jail. Temperature is 38.2 C (100.8 F). Cerebrospinal fluid (CSF) analysis shows markedly elevated protein and low glucose, and CSF cultures grow Mycobacterium tuberculosis. Resistance testing shows the following: Rifampin sensitive Isoniazid sensitive Pyrazinamide sensitive Ethambutol sensitive Streptomycin resistant Moxifloxacin sensitive Which of the following best explains the resistance pattern seen in these bacteria? Altered structure of bacterial ribosomal proteins Altered structure of enzymes involved in DNA winding-unwinding Altered structure of enzymes involved in RNA synthesis Decreased activity of bacterial catalase-peroxidase Increased activity of enzymes involved in cell wall polysaccharide synthesis

This patient with HIV has fever, headache, and central nervous system (eg, spinal fluid) culture growing Mycobacterium tuberculosis, consistent with M tuberculosis meningitis. Initial management is best accomplished through combination therapy with isoniazid (INH), rifampin, pyrazinamide, and either an injectable aminoglycoside or a fluoroquinolone. Streptomycin is one of the older drugs in the aminoglycoside family; therefore, bacterial resistance to this antibiotic is widespread, resulting in its current usage being limited mainly to the treatment of tuberculosis, plague, and tularemia. Like other aminoglycosides, streptomycin can be administered only parenterally. It works by interfering with the 16s rRNA of the bacterial 30S ribosomal subunit, thereby preventing bacterial protein synthesis. Mutations of the genes that encode ribosomal proteins are responsible for aminoglycoside resistance because they lead to modifications in the ribosomal binding sites for these drugs. Other resistance mechanisms to aminoglycosides include aminoglycoside-modifying enzymes (eg, transferases) and mutated porins. (Choice B) Structural alteration of enzymes involved in DNA winding-unwinding is the means by which many microorganisms become resistant to fluoroquinolone antibiotics. Fluoroquinolones inhibit the bacterial enzymes DNA gyrase (topoisomerase II) and topoisomerase IV. (Choice C) Structural alteration of enzymes involved in RNA synthesis is the mechanism through which organisms become resistant to rifampin. Rifampin inhibits the bacterial DNA-dependent RNA polymerase, thereby preventing the transcription of DNA into mRNA. (Choice D) Decreased activity of bacterial catalase-peroxidase is a mechanism of mycobacterial resistance to INH. Mycobacterial catalase-peroxidase is required for the initial enzymatic conversion of INH to its active metabolite within the mycobacterial cells; without this enzyme, INH is unable to inhibit mycobacterial mycolic acid synthesis. (Choice E) Increased activity of enzymes involved in cell wall polysaccharide synthesis is the means by which mycobacteria develop resistance to ethambutol. Ethambutol interferes specifically with mycobacterial peptidoglycan cell wall synthesis, possibly through inhibition of arabinosyl transferase. This drug is ineffective against organisms other than mycobacteria. Educational objective: The aminoglycoside streptomycin inhibits protein synthesis by inactivating the 30S (small) ribosomal subunit. Decreased activity of bacterial catalase-peroxidase is a mechanism of mycobacterial resistance to isoniazid. Structural alteration of enzymes involved in RNA synthesis (DNA-dependent RNA polymerase) is the mechanism through which organisms become resistant to rifampin.

A 34-year-old woman comes to the office due to vulvar and perianal itching and vaginal discharge. She has no other concerns. The patient is sexually active with her fiancé and takes combined estrogen-progestin oral contraceptive pills daily. She has regular monthly menstrual cycles and does not use barrier contraception. The patient took antibiotics 2 weeks ago for a urinary tract infection but has no chronic medical conditions. Pelvic examination shows an erythematous vulva and perineum with excoriations. A thick, white discharge is present that adheres to the vaginal walls. The remainder of the physical examination is normal. Which of the following is the most likely underlying cause of this patient's current condition? Decreased glycogen concentration in the vaginal epithelium Decreased number of gram-positive bacteria in the vagina Enterobial larva and egg deposition on the perianal skin Increased alkaline discharge by the cervical mucosa Viral DNA incorporation in the lower genital epithelia

This patient likely has a vaginal yeast infection due to overgrowth of Candida albicans. C albicans is a normal component of vaginal flora and exists in balance with the predominant bacteria of the vaginal microbiome, gram-positive Lactobacillus spp. Alterations in the balance of vaginal microbiome allow for excessive Candida proliferation and symptomatic vulvovaginal candidiasis. Common triggers include: Antibiotic use High estrogen levels (eg, pregnancy, oral contraceptive pills) Systemic corticosteroid therapy Diabetes mellitus, particularly if uncontrolled Immunosuppression (eg, HIV) Antibiotic use, as in this patient recently treated for a urinary tract infection, results in decreased number of lactobacilli and is the most common predisposing cause of Candida vaginitis. Patients can develop vulvar, vaginal, and perianal inflammation that results in erythema; pruritus; and a thick, white, clumpy vaginal discharge. Diagnosis is made via wet mount microscopy, which typically shows budding yeast and pseudohyphae. (Choice A) Decreased glycogen concentration in the vaginal epithelium is typical in postmenopausal and lactating women. This occurs due to low estrogen levels and can result in atrophic vaginitis (eg, vaginal dryness, pruritis). This patient's regular monthly menstrual cycles indicate adequate estrogen levels. (Choice C) Enterobial larva and egg deposition occurs with pinworm infection (Enterobius vermicularis). Although this common helminth infection causes perianal and vulvar pruritis that worsens at night, there is no association with thick, white vaginal discharge. (Choice D) Elevated (alkaline) vaginal pH (>4.5) is associated with bacterial vaginosis and Trichomonas vaginitis, which cannot thrive under acidic conditions. In contrast, C albicans readily adapts to its surrounding pH level; therefore, vaginal pH associated with Candida vaginitis is typically normal (3.8-4.5). (Choice E) Human papillomavirus can incorporate its DNA into anogenital, cervical, and vaginal epithelial cells. As a result, patients can develop anogenital warts, dysplasia, and malignancy. However, patients do not typically have an associated thick, white vaginal discharge. Educational objective: Antibiotic use alters the balance of normal vaginal flora (eg, decreases the number of gram-positive lactobacilli) and facilitates Candida overgrowth. Patients can develop vulvovaginal candidiasis, which presents with vulvar/vaginal/perianal pruritis; erythema; and a thick, white vaginal discharge.

A 12-year-old boy is brought to the office with fever, chills, and a rash that appeared this morning. Two days earlier, the child started complaining of a sore throat. His temperature is 38.3 C (101 F). Examination shows a diffuse erythematous rash on his chest and abdomen that blanches with pressure, along with numerous 1- to 2-mm papules. The throat is erythematous with gray-white tonsillar exudates and the tongue is bright red. Which of the following complications is most likely to develop in this patient? Aplastic anemia Coronary artery aneurysm Encephalitis Orchitis Rheumatic fever

This patient most likely has scarlet fever, which is caused by strains of Group A streptococcus that produce pyrogenic exotoxins. Scarlet fever is most often associated with streptococcal pharyngitis, which begins acutely after an incubation period of 1-5 days. Initial symptoms include fever, malaise, abdominal pain, and sore throat. The pharynx is typically erythematous, swollen, and possibly covered with gray-white exudates. In addition, the tongue can have inflamed red papillae with an appearance similar to that of a red strawberry. After 1-2 days, a rash appears on the neck, armpits, and groin that subsequently generalizes to the rest of the body. The rash begins as scarlet spots or blotches, giving a boiled lobster appearance. As the rash progresses and becomes more widespread, it begins to resemble a sunburn with goose pimples ("sandpaper-like" rash). The cheeks commonly appear flushed, giving the area around the mouth a pale appearance in comparison (circumoral pallor). Toward the end of the first week, desquamation begins and is most pronounced in the armpits, groin, and tips of the fingers and toes. As with any streptococcal upper respiratory infection, scarlet fever can predispose to acute rheumatic fever and glomerulonephritis. Treatment with penicillin V can prevent the development of rheumatic fever, although its role in preventing glomerulonephritis is uncertain. (Choice A) Aplastic anemia can complicate parvovirus B19 infection in patients with sickle cell anemia or immunocompromisation. (Choice B) Coronary artery aneurysm is the most dreaded complication of Kawasaki disease, which can present with fever, strawberry tongue, and a rash. However, tonsillar exudates are more typical of bacterial disease, and this patient lacks other manifestations of Kawasaki disease (eg, bilateral conjunctival injection). (Choice C) Measles virus infection is associated with a rash that classically starts on the face and Koplik spots (small whitish/blue lesions on buccal mucosa). Neurological complications can include encephalitis (acutely), acute disseminated encephalomyelitis (during recovery), and subacute sclerosing panencephalitis (years later). (Choice D) Orchitis is one of the most common complications of mumps. Mumps characteristically presents with fever, malaise, headaches, and myalgias followed by the development of parotitis within 48 hours. Educational objective: Scarlet fever is characterized by fever, pharyngitis, sandpaper-like rash, circumoral pallor, and a strawberry tongue. It is caused by strains of Group A streptococcus that produce pyrogenic exotoxins. Scarlet fever can predispose to acute rheumatic fever and glomerulonephritis.

A 48-year-old previously healthy man is evaluated for progressive fatigue, exertional dyspnea, and excessive bruising. Physical examination reveals mucosal pallor and scattered ecchymoses. Blood cell count shows anemia, thrombocytopenia, and leukocytosis with numerous circulating myeloblasts. Bone marrow biopsy confirms acute myeloid leukemia. The patient receives induction chemotherapy. Four days after completion of chemotherapy, the patient develops fever and malaise. New clusters of macular skin lesions are noted. Complete blood count shows severe neutropenia. Blood cultures grow smooth, creamy white, glistening colonies of small yeast forms. Incubating the pathogen in human serum at 37 C for 2-3 hours leads to formation of small projections from the cell surface with no constriction, as shown in the exhibit. Which of the following is the most likely organism responsible for this patient's condition? Aspergillus fumigatus Candida albicans Coccidioides immitis Cryptococcus neoformans Histoplasma capsulatum Rhizopus species

This patient with acute myeloid leukemia underwent induction chemotherapy prior to hematologic stem cell transplantation. Because induction chemotherapy ablates bone marrow cells, patients typically develop profound pancytopenia within a few days. Although anemia can cause dyspnea and fatigue, and thrombocytopenia can cause bruising and bleeding, the most life-threatening issue following induction chemotherapy is severe neutropenia, which dramatically increases risk of infection with virulent (eg, Streptococcus pneumoniae, gram-negative rods) and less virulent (eg, Candida albicans) organisms. In this case, the patient has a bloodstream infection with C albicans, a normal commensal of the mouth, skin, and gastrointestinal tract. Although mucocutaneous Candida infections are common in otherwise healthy individuals, invasive infection typically occurs only in those with indwelling central catheters or significant neutropenia. Manifestations can include organ abscesses (eg, kidney, heart, brain, eye, liver) and clusters of macular skin lesions. Blood cultures will reveal smooth, creamy white, glistening colonies. Microscopic evaluation will show small, ovoid budding yeasts. Pseudohyphae, an important diagnostic clue, form when yeasts do not separate after budding, creating a sausage-like chain of elongated yeasts joined together end-to-end with constrictions. Incubating the sample in human serum at 37 C for 2-3 hours (ie, germ tube test) leads to the formation of true hyphae, projections from the yeast with no constrictions. A positive germ tube test is diagnostic of C albicans and distinguishes it from other Candida species (eg, C tropicalis, C glabrata). (Choice A) Aspergillus fumigatus is a mold that forms powdery colonies. Microscopy reveals septate hyphae; yeasts are not seen. (Choice C) Coccidioides immitis is a dimorphic fungus that grows in mold form when cultured at 25 C (77 F). It appears as yeast (spherules containing endospores) at body temperature. Coccidioides does not form germ tubes. (Choice D) Cryptococcus neoformans, which grows as white mucoid colonies, also shows budding yeast on microscopy. However, it does not form germ tubes or pseudohyphae. In addition, special stains (eg, India ink) reveal a thick capsule. (Choice E) Histoplasma capsulatum is a slow-growing, dimorphic fungus that grows in mold form when cultured at 25 C. It appears as oval-shaped, budding yeasts at body temperature. Histoplasma does not form germ tubes. (Choice F) Rhizopus is a mold that grows as white, cotton-like colonies that turn gray-brown over time. Microscopy reveals broad, nonseptate hyphae; yeasts are not seen. Educational objective: Microscopic examination of Candida albicans, a common invasive infection in patients with neutropenia, reveals budding yeasts and pseudohyphae. It can be differentiated from other Candida species by a positive germ tube test.

A 34-year-old woman comes to the office due to fever, dysuria, and flank pain. Temperature is 37.8 C (100 F), blood pressure is 122/78 mm Hg, and pulse is 84/min. Physical examination elicits suprapubic and costovertebral angle tenderness. Urine dipstick analysis reveals positive leukocyte esterase and nitrite. A subsequent urine culture grows colonies of motile gram-negative rods demonstrating a green metallic sheen on eosin methylene blue (EMB) agar and hemolysis on blood agar. Which of the following virulence factors is most important for the development of this patient's condition? Capsule Heat-labile enterotoxin Lipid A O antigen P fimbriae

This patient with fever, dysuria, and flank pain has bacterial growth in the urine, findings diagnostic of a urinary tract infection (UTI). Escherichia coli, a motile, gram-negative bacillus, is the most common cause of UTI and grows well on blood, MacConkey, and eosin methylene blue (EMB) agar plates. Due to its ability to ferment lactose, E coli binds to dye in the EMB agar and produces colonies that have a distinct green metallic sheen; most strains of E coli also demonstrate beta-hemolysis on blood agar. Varying strains of E coli have specific virulence factors that determine disease characteristics; in the case of urinary tract infections, P fimbriae (pyelonephritis-associated pili) are the primary virulence factors due to their ability to adhere to uroepithelium. Normally, E coli is a commensal bacterium in the intestinal tract; however, it can spread from the perianal region to the periurethral regions (eg, via sexual intercourse or bladder catheterization). P fimbriae allow uropathogenic E coli to ascend into the bladder, ureters, and kidneys, leading to cystitis and pyelonephritis. (Choice A) The K1 capsular antigen promotes bloodstream survival of E coli because its sialic acid composition is similar to human cells (ie, molecular mimicry), which helps prevent phagocytosis and facilitates invasive infections. It is the major virulence factor for E coli strains causing neonatal meningitis (not UTIs). (Choice B) Enterotoxigenic E coli produces heat-stable (ST) and heat-labile (LT) enterotoxins in the gastrointestinal tract. ST activates guanylate cyclase, causing an increase in cyclic GMP. LT activates adenylate cyclase, increasing cyclic AMP levels. These enterotoxins lead to watery diarrhea, not UTI symptoms. (Choices C and D) Lipid A and O antigen are components of lipopolysaccharides, which make up the majority of the outer membrane of gram-negative bacteria. Lipid A is a virulence factor that activates macrophages, leading to widespread release of inflammatory cytokines (eg, IL-1, IL-6, tumor necrosis factor-alpha), which in turn causes endothelial injury, increased vascular permeability, and septic shock. The O antigen is on the outer surface and is a variable polysaccharide used to classify gram-negative bacteria. Neither component is a major virulence factor for uropathogenic E coli. Educational objective: Escherichia coli is the most common cause of urinary tract infections. The primary virulence factor of uropathogenic E coli are P fimbriae (pili) which allow the bacteria to adhere to uroepithelial cells and infect the bladder, ureters, and kidneys.

A 78-year-old man is brought to the emergency department due to fever, cough, and shortness of breath. The patient recently moved into an assisted living facility after living with his family that owned several pets. He has a 40-pack-year smoking history. Temperature is 39.4 C (103 F), blood pressure is 106/62 mm Hg, pulse is 112/min, and respirations are 28/min. There is dullness to percussion and bronchial breath sounds over the left lung. Chest x-ray reveals a left lower lobe consolidation. Sputum microscopy shows gram-positive diplococci. Which of the following would have been most helpful in preventing this patient's lung infection? Avoidance of exposure to bird droppings Immediate chemoprophylaxis after exposure Immunization with inactivated microbial agent Periodic culture and disinfection of water supply Vaccination with bacterial polysaccharide

This patient with fever, pulmonary symptoms, and a lobar consolidation on chest x-ray has pneumonia. The presence of gram-positive diplococci in the sputum indicates the underlying pathogen is Streptococcus pneumoniae, the leading cause of community-acquired pneumonia. Over 90 strains of S pneumoniae have been identified; they are distinguished based on antigenic variations in the capsular polysaccharide, the major virulence factor of the bacteria. Antibodies against the polysaccharide capsule are generated during infection and provide long-lasting immunity against that strain. The pneumococcal vaccine contains polysaccharide antigens from the most common disease-causing serotypes leading to the generation of protective antibodies against these strains. In the United States, 2 types of pneumococcal vaccinations are available: Pneumococcal polysaccharide vaccine contains capsular material from 23 serotypes. Because polysaccharides cannot be displayed by the major histocompatibility complex of antigen-presenting cells (only peptides can), immunogenicity to this vaccine is T-cell independent and generates a mild antibody response with no memory cells. Although polysaccharide vaccines are fairly effective for most adult patients, they are not useful for infants (age <2). Pneumococcal conjugate vaccine consists of capsular polysaccharides from 13 serotypes that have been covalently attached to recombinant, inactivated diphtheria toxin. Protein conjugation allows the polysaccharide to be displayed by the major histocompatibility complex and induces a much stronger immunogenic response that involves T-cell-mediated B-lymphocyte activation. This generates high- affinity IgG antibodies and memory B-cells and also creates mucosal IgA antibodies, which reduce colonization rates. Pneumococcal vaccination significantly reduces the risk of invasive pneumococcal disease. Routine vaccination is recommended for all children as part of their childhood immunization series. Adults age >65 and those at high risk for invasive disease (eg, HIV, asplenia, other immunosuppressed states) should also be vaccinated. (Choice A) Histoplasma is a fungal infection that is primarily transmitted by inhalation of particles from soil contaminated with bird or bat droppings. Diagnosis is usually made by urine or serum antigen testing, but yeasts can sometimes be identified on histology/culture using specialized stains. (Choice B) Close contacts of patients who have pertussis should be given antibiotic chemoprophylaxis. Pertussis is a gram-negative encapsulated coccobacillus; infected adults usually develop a few weeks of malaise and rhinorrhea followed by several weeks of severe, paroxysmal cough. (Choice C) The influenza vaccine contains 3 or 4 strains of inactivated influenza virus. Influenza usually causes abrupt-onset fever, headache, myalgia, and malaise and would not appear on Gram stain. (Choice D) Legionella is a common cause of atypical pneumonia and is usually transmitted by contaminated water systems. However, Legionella is a gram-negative bacterium (not a gram-positive diplococcus) and is usually diagnosed with urine antigen testing. Educational objective: Streptococcus pneumoniae vaccination reduces the risk of invasive disease and is recommended for young patients and the elderly. The pneumococcal polysaccharide vaccine is an unconjugated vaccine that induces a T-cell-independent humoral immune response. In contrast, the pneumococcal conjugate vaccine contains polysaccharide material attached to a protein antigen, which creates a robust T-cell-mediated humoral immune response.

An 83-year-old woman is brought to the emergency department from a nursing home due to fever and altered mental status. The patient has a history of hypertension, diabetes, dementia, fecal incontinence, and bladder dysfunction requiring intermittent urinary catheterization. Her temperature is 38.3 C (100.9 F), blood pressure is 100/68 mm Hg, and pulse is 114/min. She is lethargic and unable to answer questions. Her abdomen is soft, with suprapubic tenderness. Lungs are clear bilaterally. There is no cardiac murmur or skin rash. Laboratory results are as follows: Blood Leukocytes 17,000/mm3 Potassium3.8 mEq/L Bicarbonate22 mEq/L Creatinine0.8 mg/dL Urinalysis Leukocyte esterase positive Nitrites positive Leukocytes 10/hpf Chest x-ray reveals normal-appearing lungs. Blood and urine cultures are performed, and empiric antibiotics are initiated. Urine cultures are positive for bacteria that grow readily on MacConkey agar, are lactose-fermenting, and are indole-positive. Which of the following is the most likely organism causing this patient's infection? Enterobacter cloacae Escherichia coli Proteus mirabilis Pseudomonas aeruginosa Staphylococcus saprophyticus

This patient with fever, tachycardia, and altered mental status has sepsis; the presence of leukocytes, nitrites, leukocyte esterase, and bacteria in the urine make a urinary tract infection (UTI) the most likely infectious source. The pathogenic organism can be determined based on microbiological features. For instance, MacConkey agar is selective for gram-negative rods because gram-positive bacteria are unable to grow on this culture medium. Some gram-negative rods can ferment lactose, causing colonies to turn pink, and both Escherichia coli and Enterobacter cloacae are fast lactose fermenters. The ability to convert tryptophan to indole (ie, indole-positive) can further differentiate bacteria, and the indole-positivity of E coli distinguishes it from E cloacae (Choice A). Among the organisms listed, this patient with lactose-fermenting, indole-positive bacterial growth on MacConkey agar has infection due to E coli, the most common cause of UTI. Women are more likely than men to develop UTIs due to a shorter distance between the anus and urethra, and fecal incontinence increases the risk of introducing gastrointestinal flora such as E coli into the urinary tract. Other predisposing factors for E coli UTI—particularly in older women—include diabetes, urinary retention or incontinence, and instrumentation of the urethra (eg, intermittent catheterization), all of which are present in this patient. (Choices C and D) Proteus mirabilis and Pseudomonas aeruginosa are gram-negative rods that can cause UTI, most commonly in patients with indwelling Foley catheters. However, in contrast to E coli, these organisms are lactose nonfermenters (ie, do not form pink colonies on MacConkey agar). (Choice E) Staphylococcus saprophyticus is the second-most frequent cause of UTI after E coli in young, sexually active women but is less common in the elderly population. Moreover, staphylococci are gram-positive, and therefore would not grow on MacConkey agar. Educational objective: Escherichia coli are lactose-fermenting, indole-positive, gram-negative rods that grow on MacConkey agar and are the most common pathogen causing urinary tract infection.

A 64-year-old man comes to the office due to several days of low-grade fever and productive cough. He has smoked a pack of cigarettes daily for 30 years and has been hospitalized several times for respiratory infections. Temperature is 37.2 C (99 F), blood pressure is 130/70 mm Hg, pulse is 82/min, and respirations are 14/min. Pulse oximetry shows 98% on ambient air. Lung auscultation reveals rhonchi that clear with coughing. Sputum cultures in specialized media grow round-to-oval, budding yeast, which form germ tubes when incubated at 37 C in serum. Which of the following best explains the presence of the organism in this patient's sputum sample? Colonization of a preexisting lung cavity Contamination by normal oral flora Embolization of infectious microthrombi Lung-infection from inhaled microconidia Proliferation of previously latent organism

This patient's 30-pack-year smoking history, low-grade fever, productive cough, normal pulse oximetry, and pulmonary rhonchi suggest a chronic obstructive pulmonary disease (COPD) exacerbation. Although expectorated sputum cultures are not routinely obtained in patients with COPD exacerbation, they may help identify the underlying respiratory pathogen in certain cases (eg, concern for drug-resistant organism). However, positive expectorated sputum results often indicate contamination with oral flora rather than true infection. This patient's sputum culture grew Candida albicans, an oval, budding yeast that forms true hyphae ("germ tubes") when incubated at body temperature (37 C). C albicans is a normal commensal of the skin and gastrointestinal tract that occasionally causes mucocutaneous infection (eg, vulvovaginitis, cutaneous candidiasis) but rarely causes invasive disease (eg, sepsis, chorioretinitis, endocarditis); invasive infection primarily occurs in those with significant neutropenia or indwelling vascular catheters (which provide a portal of entry from the skin into the bloodstream). Therefore, the presence of C albicans on a sputum sample almost always indicates contamination with oral flora rather than true infection. Other common oral contaminants (that almost never indicate true infection) include coagulase-negative staphylococci and enterococci. (Choice A) Aspergillus fumigatus, a fungus with dichotomous, branching hyphae, can colonize preexisting lung cavities, thereby creating a "fungus ball" (aspergilloma). Most aspergillomas are asymptomatic, and the organism does not resemble a yeast in culture. (Choice C) Candida infective endocarditis can cause pulmonary infection due to septic pulmonary emboli. However, this condition is rare and is typically seen in patients with indwelling vascular catheters or history of intravenous drug use. (Choice D) Histoplasma capsulatum is a dimorphic fungus that is inhaled as microconidia and converts to a small, ovoid yeast in the lungs. It replicates by forming narrow-based buds, not germ tubes. In addition, most clinically significant cases are associated with pneumonia, not bronchitis. (Choice E) Mycobacterium tuberculosis pulmonary infection is usually marked by months or years of asymptomatic latency (within granulomas) that may later reactivate and cause cavitary lung disease. Sputum culture would reveal acid-fast bacilli, not yeast. Educational objective: Expectorated sputum cultures are often contaminated by normal oral flora. The growth of Candida albicans, a normal commensal of the gastrointestinal tract and skin, almost always indicates oral contamination rather than true pulmonary infection.

A 6-month-old boy is brought to the emergency department due to poor feeding, irritability, and rash. He was well-appearing until 2 days ago, when he developed little interest in drinking and a progressive rash. Physical examination reveals diffuse erythema and desquamation that is most prominent at the neck, axillae, inguinal folds, and perioral region. The perioral area also has crusting and the lips are dry and cracked, but the mucosal membranes are normal. The epidermis easily comes off with gentle pressure at the erythematous areas. Which of the following is the most likely cause of this patient's symptoms? Autoantibody binding to epithelial cell surface Cell-mediated hypersensitivity Endotoxin-mediated inflammatory response Exotoxin mediated skin damage Mast cell degranulation

This patient's fever and desquamating rash are most consistent with staphylococcal scalded skin syndrome (SSSS), an infection caused by strains of exotoxin-producing Staphylococcus aureus. Exfoliative exotoxins act as proteases that cleave desmoglein in desmosomes, which are junctions that adhere epidermal cells together. Skin damage occurs as the exotoxins spread, causing separation of keratinocytes and superficial epidermal blistering. Gentle pressure causes detachment and shedding of the outer layer of skin (ie, Nikolsky sign). The rash, although diffusely erythematous and painful ("scalded"), is most prominent in the skin folds (eg, neck, axillae, groin). Perioral involvement (eg, erythema, crusting) is common, but the mucous membranes are spared. SSSS is most common in infants and young children, and skin findings are often preceded or accompanied by fever, irritability, malaise, and poor feeding. Treatment includes antistaphylococcal antibiotics (eg, nafcillin, oxacillin). (Choice A) Pemphigus vulgaris is caused by an autoantibody to desmoglein and results in a positive Nikolsky sign and epidermal erosions, blisters, and bullae. This disorder is uncommon in children, and mucous membranes are almost always involved. (Choice B) Cell-mediated (ie, type IV delayed-type) hypersensitivity, in which sensitized TH1 cells secrete cytokines to attract cytotoxic T cells and NK cells, is responsible for Stevens-Johnson syndrome and toxic epidermal necrolysis. Blistering, bullae, and Nikolsky sign may be seen, but mucosal involvement is almost always present. In addition, a preceding trigger (eg, medication, Mycoplasma pneumoniae infection) would be expected. (Choice C) Lipopolysaccharides (ie, endotoxin) released into the blood by gram-negative bacteria can cause a severe, widespread inflammatory response (septic shock). Purpuric lesions may occur with gram-negative sepsis, but Nikolsky sign would not be present. (Choice E) Crosslinking of IgE on the cell surface causes mast cell degranulation that is associated with allergic reactions. Urticaria (hives) appears as itchy, erythematous, blanching papules, not diffuse erythema or desquamation. Educational objective: Staphylococcal scalded skin syndrome occurs in infants and children due to the production of exfoliative exotoxins by Staphylococcus aureus. The toxins cleave desmoglein in desmosomes, leading to widespread epidermal blistering and shedding, especially with gentle pressure (ie, Nikolsky sign). Mucous membranes are spared.

A 52-year-old woman is evaluated due to 2 weeks of fever, fatigue, nonproductive cough, and dyspnea, which began shortly after visiting her family in Ohio. She also lost 1.5 kg (3.3 lb) over the same period. The patient has a history of rheumatoid arthritis and has been taking adalimumab for the past 6 months. She had negative tuberculosis skin testing prior to beginning the drug. Temperature is 38 C (100.4 F), blood pressure is 130/70 mm Hg, pulse is 92/min, and respirations are 18/min. Physical examination reveals bilateral lung crackles, mild generalized lymphadenopathy, and hepatosplenomegaly. Chest x-ray shows bilateral nodular densities and hilar lymphadenopathy. Urine testing is positive for a fungal antigen. Which of the following pathogenic processes is most important during the development of this patient's infection? intracellular proliferation within the macrophages Invasion of the vascular endothelial lining Overgrowth and tissue invasion by endogenous flora Production of an antiphagocytic polysaccharide capsule Transformation into a mycelial form in tissues

This patient's rheumatoid arthritis is being treated with adalimumab, a tumor necrosis factor (TNF)-alpha inhibitor. TNF-alpha is a crucial cytokine of the cell-mediated immune response that aids in macrophage activation, phagolysosome function, and the formation and maintenance of granulomas. Blockade of TNF-alpha increases the risk of infection with pathogens controlled by the cell-mediated immune response such as intracellular bacteria (eg, Mycobacterium tuberculosis), viruses, and granulomatous fungi. This patient's pulmonary and reticuloendothelial (eg, lymphadenopathy, splenomegaly) findings and positive urine fungal antigen test raise strong suspicion for infection with Histoplasma capsulatum, a dimorphic fungus endemic to the Ohio and Mississippi River Valleys. H capsulatum is a mold that is inhaled from contaminated soil and converts to a yeast form in the lungs. Phagocytosis by alveolar macrophages does not result in destruction of the organism due to microbial virulence factors that prevent phagolysosome acidification. Therefore, for the first several weeks of infection, H capsulatum is able to proliferate within macrophages in an unchecked fashion. Eventually, the cell-mediated immune response controls the infection in most healthy patients. However, those with impaired immunity (as in this case) are at high risk for life-threatening disseminated disease. The lungs, reticuloendothelial system, and mouth (oral ulcers) are most commonly affected. Urine Histoplasma antigen testing is often used to quickly confirm the diagnosis. (Choice B) Aspergillus fumigatus is a monomorphic mold that can cause invasive disease in those who are immunocompromised. This organism is inhaled into the lungs and can invade the vascular endothelial lining, leading to dissemination through the bloodstream. However, patients usually have fever, chest pain, and hemoptysis. (Choice C) Candida species are part of the endogenous human flora, but overgrowth can occur in those with immunocompromisation and lead to mucocutaneous (eg, thrush) or invasive (pneumonia, fungemia) disease. Invasive Candida infections are generally diagnosed by blood culture or biopsy with histopathology, not urine antigen testing. (Choice D) Cryptococcus neoformans is a yeast with a thick, antiphagocytic, polysaccharide capsule that primarily causes infections in those who are immunocompromised (particularly patients with advanced AIDS). Although pulmonary infection can occur, disseminated disease is usually marked by meningoencephalitis (eg, headache, confusion, altered mental state). (Choice E) Malassezia is a dimorphic fungus that is part of normal skin flora. Conversion from yeast form to the pathogenic mycelial form results in tinea versicolor, which is associated with the characteristic spaghetti and meatballs appearance on microscopy (ie, both hyphae and yeasts seen). Educational objective: Tumor necrosis factor-alpha inhibitors are associated with impairments to the cell-mediated immune response and increase the risk of infection with intracellular bacteria and granulomatous fungi (eg, Histoplasma capsulatum). Histoplasma replicates within the intracellular space of macrophages and can spread from the lungs to the lymph nodes and the reticuloendothelial system (liver, spleen, bone marrow).

A 47-year-old man comes to the office due to persistent fever, night sweats, and fatigue. Thorough evaluation yields a diagnosis of chronic myeloid leukemia. While undergoing treatment for his malignancy, the patient experiences headaches, scant nasal discharge, and left eye symptoms. Physical examination reveals tenderness over the paranasal sinuses and left-sided orbital swelling and cellulitis. Mild proptosis and ptosis of the left eye are also present. Biopsy of the sinus mucosa is shown in the image below: ID=105 septate hyphae with acute angles Aspergillus fumigatus Blastomyces dermatitdis Candida albicans Coccidiodes immitis Cryptococcus neoformans Hisoplasma capsulatum Malassezia furfur Microsporum canis Rhizopus species Sporothrix schenckii

This patient's symptoms are consistent with fungal rhinosinusitis. The biopsy shows fungal hyphae branching at acute angles with septations, findings characteristic of Aspergillus fumigatus. This fungus is widely distributed in the environment and commonly grows on decaying vegetables. It is monomorphic, existing only in mold form (ie, multicellular hyphae). Aspergillus causes the following conditions: Invasive aspergillosis develops in immunosuppressed patients. The prolonged neutropenia associated with leukemia and lymphoma treatment is a strong risk factor. It most commonly affects the lung and can cause fever, pleuritic chest pain, and hemoptysis. Aspergillus has a predilection for blood vessels and can spread hematogenously, causing infection and infarcts involving the skin, paranasal sinuses, kidneys, endocardium, and brain. Diagnosis is made by light microscopy, which reveals V-shaped, narrow, septate hyphae invading the tissue. Voriconazole is used to treat invasive aspergillosis. Aspergillomas are Aspergillus fungus balls that grow in old lung cavities (eg, produced by tuberculosis or bronchiectasis). They are limited to the preexisting cavity; invasion of the surrounding lung tissue is rare in immunocompetent patients. Aspergillomas are often surgically removed. Allergic bronchopulmonary aspergillosis occurs in asthma and presents with wheezing and migratory pulmonary infiltrates. Increased serum IgE and increased titers of antibodies against Aspergillus are characteristic. Treatment is with corticosteroids. (Choices B, D, and F) Blastomyces, Coccidioides, and Histoplasma are dimorphic fungi. They grow as molds in nature but exist in yeast form in humans (Coccidioides exists in endospore form at body temperature). As a result, hyphae would not be seen in biopsy specimens. (Choice C) Candida albicans is a component of normal human mucous membrane flora. On light microscopy, it appears as an oval, budding yeast with pseudohyphae (ie, not true hyphae but rather elongated yeast cells). (Choice E) Cryptococcus neoformans, an oval, budding yeast with a thick capsule, causes meningitis and lung infections in AIDS. India ink stain of cerebrospinal fluid shows a wide, clear zone around the cell corresponding to the unstained capsule. (Choices G and H) Malassezia furfur and Microsporum canis can cause cutaneous fungal infections. They do not typically cause rhinosinusitis. (Choice I) Rhizopus and Mucor form broad, ribbon-shaped, nonseptate hyphae that branch at right or wide angles. They cause mucormycosis, which typically presents as a paranasal infection in immunocompromised patients or those with diabetes mellitus. However, Aspergillus can also cause rhinosinusitis identical in presentation to mucormycosis. The presence of narrow hyphae with septations makes invasive aspergillosis more likely. (Choice J) Sporothrix schenckii, often inoculated into the skin during outdoor activities, typically presents as papulonodular lesions that form along the draining lymphatics. Educational objective: Immunosuppressed patients are at risk for Aspergillus fumigatus infection. This fungus produces thin, septate hyphae with acute, V-shaped branching, causing invasive aspergillosis, aspergillomas, and allergic bronchopulmonary aspergillosis.

A 23-year-old man comes to the office due to several days of dysuria. He has had no fever or hematuria. The patient has no other medical conditions, takes no medications, and has no known allergies. Temperature is 37.1 C (98.8 F). On genitourinary examination, mucopurulent discharge is noted at the urethral meatus. Microscopy of the discharge shows numerous neutrophils, some of which contain intracellular gram-negative diplococci. A single intramuscular injection of ceftriaxone is administered. Which of the following additional medications is recommended at this time? Doxycycline Fluconazole Metronidazole No additional medication Penicillin G benzathine Trimethoprim-sulfamethoxazole

This young man with dysuria and penile discharge likely has infectious urethritis due to sexually transmitted infection. Most cases are caused by Neisseria gonorrhoeae, Chlamydia trachomatis, or Mycoplasma genitalium; simultaneous infection with multiple pathogens is common. Nucleic acid amplification testing (NAAT) of urethral, vaginal, or anal swabs is used to identify the causative organism. When available, light microscopy with Gram stain is performed to rapidly diagnose N gonorrhoeae infection while awaiting NAAT results. Gonococcal urethritis is diagnosed when there are leukocytes with intracellular gram-negative diplococci; nongonococcal urethritis is diagnosed when there are no organisms present (because C trachomatis does not have significant peptidoglycan in the cell wall, it cannot be identified on Gram stain). Treatment is as follows: Gonorrhea with positive/uncertain Chlamydia status (as in this patient) is treated with ceftriaxone and doxycycline to provide dual coverage for gonorrhea (ceftriaxone plus doxycycline) and coverage for potential Chlamydia coinfection (doxycycline). Gonorrhea with negative Chlamydia NAAT is treated with ceftriaxone alone (Choice D). Positive Chlamydia with no gonorrhea is treated with azithromycin or doxycycline monotherapy. (Choice B) Fluconazole is a triazole-class antifungal agent used to treat candidiasis, cryptococcosis, histoplasmosis, blastomycosis, and coccidioidomycosis. Fungal urethritis is uncommon in otherwise healthy men. (Choice C) Metronidazole treats anaerobic bacteria and protozoa. It has no activity against N gonorrhoeae or C trachomatis but can be used for urethritis caused by Trichomonas vaginalis. Because T vaginalis is a less common cause of infectious urethritis in men, empiric coverage against T vaginalis is not recommended. (Choice E) Penicillin G benzathine is used to treat syphilis infection, which typically presents with a chancre (primary syphilis) or diffuse rash and lymphadenopathy (secondary syphilis). Treatment for syphilis is not indicated without these characteristic symptoms or a positive syphilis test result. (Choice F) Trimethoprim-sulfamethoxazole is used to treat cystitis or proctitis due to coliform bacilli (eg, Escherichia coli). These organisms can cause urethritis in older men in the setting of prostatic obstruction of the urethra. However, infectious urethritis in young men is almost always caused by sexually transmitted infection, not coliform organisms. Educational objective: Urethritis in young men is usually due to sexually transmitted infection. Treatment for gonococcal urethritis with uncertain or positive Chlamydia coinfection status is ceftriaxone plus doxycycline. Treatment for gonococcal urethritis with negative Chlamydia is ceftriaxone alone. Patients with no gonorrhea who have Chlamydia are treated with azithromycin or doxycycline monotherapy.

A 76-year-old woman comes to the emergency department due to fever and burning with urination for 2 days and right-sided back pain for 1 day. Her temperature is 38.3 C (101 F), blood pressure is 110/80 mm Hg, pulse is 94/min, and respirations are 18/min. Examination is notable for right-sided flank tenderness to palpation. Urinalysis is positive for nitrites, leukocyte esterase, and bacteria. Urine and blood cultures are obtained, and the patient is started on antibiotic therapy. The next day, urine culture grows >100,000 colony-forming units/mL of Escherichia coli. The organism is found to have a methyltransferase that methylates ribosomal RNA. This enzyme most likely confers resistance to which of the following classes of antibiotics? Aminoglycosides Carbapenems Glycopeptides Penicillin Quinolones

Aminoglycosides interfere with the aminoacyl binding site on the 30S ribosomal subunit. Binding of aminoglycoside causes the cell to misread mRNA and, as a result, it is unable to perform protein synthesis. An important mechanism of resistance to aminoglycosides is the methylation of the aminoglycoside-binding portion of the ribosome, which inhibits the ability of aminoglycosides to interfere with protein translation. Other common resistance mechanisms are the production of enzymes that inactivate the drug by altering its chemical structure or the production of an efflux pump that decreases the drug's intracellular concentration. (Choices B and D) Carbapenems and penicillins function similarly to other beta-lactam antibiotics by inhibiting a family of enzymes responsible for cell wall construction. Resistance to both can occur through alteration of the antibiotic binding site on the enzyme; decreased permeability of the antibiotic into the cell wall; or production of beta-lactamases, which alter the antibiotic structure by cleaving protein bonds in the antibiotic and inactivating it. (Choice C) Vancomycin is a glycopeptide antibiotic used to treat methicillin-resistant Staphylococcus aureus infections. It binds to D-alanine-D-alanine, thereby preventing peptidoglycan cell wall synthesis. Bacteria have been identified in which a D-alanine is replaced by a D-lactate, leading to vancomycin resistance. (Choice E) Resistance to the quinolones occurs by alteration of the genes encoding for DNA gyrase and topoisomerase IV, development of an efflux pump, or alteration of the membrane diffusion channels used by the antibiotic. Educational objective: Aminoglycosides work by interfering with the 30S ribosomal subunit and causing the cell to misread messenger RNA, thereby halting protein synthesis. An important mechanism of resistance is the methylation of the aminoglycoside-binding portion of the ribosome, which inhibits the ability of aminoglycoside to interfere with protein translation.

A 65-year-old man comes to the emergency department due to an episode of hemoptysis that occurred earlier this morning. The patient estimates that he coughed up about 100 mL of blood. Prior to this episode he was in his usual state of health and felt well, with no fevers, night sweats, or weight loss. He has had occasional episodes of blood-tinged sputum over the past few months. The patient's past medical history is significant for tuberculosis (TB) that was effectively treated several years ago. He had smoked a pack of cigarettes daily for 45 years but quit 4 years ago. A representative cut of the CT scan, shown in the image below, demonstrates changes consistent with prior TB infection, including an old left upper lobe cavity. Compared to prior scans, the main difference is that the cavity now appears to be filled with a round mass. ID=108 Fungus ball Which of the following best describes this patient's condition? Allergic Colonizing Contagious Invasive Malignant

Aspergillus fumigatus is a mold that is widely present in organic matter. It forms septate hyphae that branch at 45-degree angles (V-shaped branching). The spores are inhaled with the air and are typically cleared by the mucus and ciliated epithelium of the respiratory tract. In individuals with suppressed immune defenses, Aspergillus causes a wide spectrum of diseases. This patient likely has hemoptysis due to an aspergilloma (mycetoma), which represents Aspergillus colonization. Aspergillomas develop in old lung cavities (from tuberculosis, emphysema, sarcoidosis). Aspergillus colonizes the cavity, forming a "fungus ball" (seen in the left upper lobe on this patient's CT) without lung tissue invasion. Classically, an aspergilloma will appear on chest x-ray as a radiopaque structure that shifts when the patient changes position. This condition may be asymptomatic or it may cause cough and hemoptysis. (Choice A) Allergic bronchopulmonary aspergillosis (ABPA) occurs in patients with asthma. It presents with wheezing and migratory pulmonary infiltrates. Increased titers of IgE and antibodies to Aspergillus are diagnostic of ABPA. (Choice C) Aspergillomas are not contagious. This cavity is unlikely to represent recurrent tuberculosis (no fevers, night sweats, weight loss). (Choice D) Invasive pulmonary aspergillosis occurs in severely immunosuppressed and neutropenic patients and is characterized by primary lung involvement with cough, hemoptysis, pleuritic chest pain, and fever. Necrotizing pneumonia and granuloma formation also occur. Extrapulmonary involvement is common. (Choice E) Although it could cause hemoptysis, a malignant lesion would not typically develop as a circumscribed round mass within an old cavity and is a less likely diagnosis in this patient who has felt well with no weight loss recently. Educational objective: Aspergillus fumigatus causes a wide spectrum of disease. It can be an opportunistic infection in immunosuppressed and neutropenic patients (invasive pulmonary aspergillosis). Aspergillosis can be colonizing (aspergilloma) when it forms a fungus ball within a preexisting lung cavity. It can also cause a lung hypersensitivity reaction in allergic bronchopulmonary aspergillosis in individuals with asthma.

A 45-year-old man comes to the physician due to pain, swelling, and erythema affecting his right leg. He says that he suffered a minor cut to his leg a few days ago while cleaning his garage. Physical examination shows an indurated region surrounding a minor laceration that is draining a purulent exudate. He is diagnosed with cellulitis and started on the appropriate treatment. Gram stain of the exudate shows gram-positive cocci in clusters. The organism most likely responsible for this patient's infection synthesizes a protein as part of its peptidoglycan cell wall that does which of the following? Activates complement Binds the Fc portion of IgG Causes hemolysis Cleaves IgA Interacts with MHC class II antigens

Clusters of gram-positive cocci on Gram stain is a characteristic finding of staphylococcal species such as Staphylococcus aureus. Protein A is a virulence factor that forms part of the outer peptidoglycan layer of S aureus. Protein A binds with the Fc portion of IgG antibodies at the complement-binding site, preventing complement activation (Choice A). This results in decreased production of C3b, leading to impaired opsonization and phagocytosis. (Choice C) Hemolysin is secreted by staphylococci and causes hemolysis as well as the destruction of neutrophils, macrophages, and platelets. It is a secreted factor that is not bound to the cell wall. (Choice D) Streptococcus pneumoniae and Neisseria gonorrhoeae produce IgA proteases that cause cleavage of IgA antibodies, preventing them from interfering with bacterial adhesion to mucous membranes. (Choice E) Major histocompatibility complex (MHC) class II (found on an antigen presenting cells) normally interacts with processed antigens, presenting them to T-lymphocytes to initiate an immune response. In contrast, superantigens (enterotoxins, toxic shock syndrome toxin) interact with MHC class II and the T-cell receptor outside of standard antigen binding sites to initiate widespread and nonspecific activation of T- lymphocytes. Although superantigens are synthesized by staphylococci, they are not bound to the peptidoglycan cell wall. Educational objective: Protein A is a virulence factor found in the peptidoglycan cell wall of Staphylococcus aureus that binds to the Fc portion of IgG, leading to impaired complement activation, opsonization, and phagocytosis.

A 45-year-old man comes to the urgent care clinic because of fever, severe headache, myalgia, and pleuritic chest pain. He has had these symptoms for several days. Physical examination shows fever and mild tachycardia. Lung auscultation reveals mild crackles. Radiographic examination is consistent with segmental pulmonary infiltrates. The patient fails to respond to empiric antibacterial antibiotic therapy. Microscopic examination of lung tissue obtained from this patient shows spherules packed with endospores. This patient's history is most likely to reveal which of the following? Recent travel to Arizona Exposure to pigeon droppings Cave exploration in Ohio Previous cavitary tuberculosis Chemotherapy for leukemia Long history of asthma

Coccidioides immitis is a dimorphic fungus that has a mold form (hyphae) at 25° C-30° C and an endospore form (spherules containing endospores, a unique characteristic of Coccidioides) at body temperature (37° C-40° C ). C. immitis is endemic to the southwestern United States (i.e., southern and central California, Arizona, New Mexico, and western Texas), northern Mexico, and some regions of Central and South America. Patients with coccidioidomycosis are likely to live in or have recently traveled to an endemic area. C. immitis is transmitted by spore inhalation. Spores are formed by fragmentation of hyphae. Once inside the lungs, the spores turn into spherules that contain endospores. The spherules subsequently rupture and release endospores that disseminate to other organs and tissues. Each endospore is capable of forming a new spherule. In immunocompetent hosts, C. immitis causes lung disease, which can be asymptomatic or cause flu-like symptoms (e.g., cough, fever, and myalgia) accompanied by erythema nodosum. In total, C. immitis can present in five ways: acute pneumonia (most common), chronic progressive pneumonia, pulmonary nodules and cavities, extrapulmonary nonmeningeal disease, and meningitis. The more severe manifestations are largely reserved for immunocompromised hosts. (Choice B) The yeast form of Cryptococcus neoformans is present in pigeon droppings. This fungus causes pulmonary disease and meningoencephalitis in immunocompromised patients. (Choice C) Histoplasma capsulatum is endemic to the Mississippi and Ohio River basins and found in bird and bat droppings. Patients with histoplasmosis often have a history of cleaning bird coops or caving. (Choice D) Aspergillus fumigatus can colonize old lung cavities (e.g., those formed by tuberculosis) to form a "fungal ball" (i.e., aspergilloma). Symptoms include cough, dyspnea, and hemoptysis. (Choice E) Neutropenic patients are at high risk for developing opportunistic mycoses. Candida albicans, A. fumigatus, Mucor, and Rhizopus species can cause severe disease in this population. (Choice F) Patients with asthma are at risk for developing an allergic reaction to A. fumigatus called allergic bronchopulmonary aspergillosis. Signs and symptoms include cough, dyspnea, wheezing, fever, and migratory pulmonary infiltrates. Educational objective: Coccidioides immitis is a dimorphic fungus endemic to the southwestern United States. It exists in the environment as a mold (with hyphae) that forms spores. These spores are inhaled and turn into spherules in the lungs.

A 45-year-old man comes to the emergency department due to several days of cough and fever. His temperature is 38.4 C (101.2 F), blood pressure is 117/76 mm Hg, pulse is 92/min, and respirations are 18/min. Physical examination of the right lung reveals occasional rales and decreased breath sounds at the base. A chest x-ray demonstrates a lung infiltrate, hilar adenopathy, and a right-sided pleural effusion. Lung tissue obtained from the patient reveals the findings in the image below: ID=268; spherules, review image after Which of the following is the most likely cause of this patient's condition? Aspergillus fumigatus Blastomyces dermatitidis Coccidiodes immitis Cryptococcus neoformans Cytomegalovirus Histoplasma capsulatum

Coccidioides immitis is a dimorphic fungus that is endemic to the desert areas of the United States and Mexico. It can cause lung disease in immunocompetent patients and disseminated disease in the immunocompromised. Coccidioides exists in the environment in the form of mold (hyphae); in humans, microscopic examination of affected tissues shows thick-walled spherules packed with endospores. As seen in the image above, some spherules may be rupturing and others may be empty. The spherules are larger than surrounding red blood cells (RBCs). Culture on Sabouraud agar and serology are also important in making the diagnosis. (Choice A) Aspergillus fumigatus commonly causes pulmonary disease in immunocompromised patients. It has a mold form only. Aspergillus have septate hyphae that branch at 45-degree angles. (Choice B) The characteristic appearance of Blastomyces dermatitidis is that of a round yeast with broad-based budding. These yeasts have thick, doubly refractive walls. (Choice D) Cryptococcus neoformans is a yeast with a thick capsule. When tissue specimens infected with C neoformans are stained with India ink, the capsule does not absorb the ink, resulting in a clear halo. The most common manifestation of C neoformans infection is meningitis, particularly among immunocompromised patients. (Choice E) Cytomegalovirus can cause pneumonia in transplant patients. Prominent intranuclear inclusions surrounded by a clear halo are virtually pathognomonic for pulmonary cytomegalovirus. (Choice F) Histoplasma capsulatum would appear histopathologically as small oval yeast forms within macrophages; unlike the spherules of Coccidioides, Histoplasma yeast forms are smaller than surrounding RBCs. (Choice G) Rhizopus species typically cause rhino-orbito-cerebral mucormycosis but can cause lung disease in immunocompromised patients. The species has only a mold form, with broad ribbonlike hyphae with rare septations. (Choice H) The pustular and ulcerated lesions of sporotrichosis (caused by the fungus Sporothrix schenckii and typically acquired by direct skin inoculation [eg, scrapes from rose bushes]) are localized to the site of the wound or associated lymphatic channels. On histologic examination, cigar-shaped yeast may be seen. Educational objective: Coccidioides immitis infection can be asymptomatic or it can cause pulmonary disease ranging from a flulike illness to chronic pneumonia. It causes disseminated disease in immunocompromised patients. Spherules containing endospores are found in tissue samples.

A 25-year-old man comes to the office due to 4 days of persistent diarrhea. He has a history of HIV and is compliant with antiretroviral therapy. The diarrhea was initially watery but has become bloody over the past day. A stool culture grows Shigella flexneri, which is resistant to both azithromycin and fluoroquinolones. A recent study tracking resistance patterns in Shigella species reveals rapid spread of resistance genes between different bacterial populations due to the transfer of nonchromosomal DNA between adjacent bacteria via a small hair-like appendage. Which of the following best describes the process by which the bacteria responsible for this patient's infection likely developed resistance? Conjugation Transduction Transformation Transposition Vertical gene transfer

Conjugation is a form of horizontal gene transfer that involves direct cell-to-cell contact between two adjacent bacteria. It allows for the propagation of genetic elements that confer beneficial traits such as antibiotic resistance, the ability to use new metabolites, or the ability to tolerate a difficult microenvironment. The transferred genetic element is a conjugative plasmid (separate from the genome) or an integrated conjugative element (present in the genome). Conjugation begins when proteins encoded by the conjugative element aggregate on the plasma membrane of the donor bacteria, forming a pilus (a hair-like appendage). The pilus elongates until it attaches to an adjacent bacterium and then progressively shortens, which pulls the bacteria together. The plasmid DNA in the donor cell is then nicked and relaxed, and a single DNA strand is transferred through the mating bridge from the donor to the recipient bacterium. A complimentary DNA strand is then generated in each bacterium, which completes the plasmid transfer. Conjugation is a major mechanism for Shigella antibiotic resistance. A single Shigella plasmid encodes resistance to ampicillin, chloramphenicol, tetracycline, sulfonamides, streptomycin, and trimethoprim. Other plasmids encode resistance to azithromycin and fluoroquinolones. Risk of resistant strains is greatest in patients with HIV and/or men who have sex with men, but antibiotic susceptibility testing is required for all infections due to high rates of resistance across patient populations. (Choices B and C) The two other forms of horizontal gene transfer are transduction and transformation. Transduction involves the introduction of foreign DNA into a bacterial cell by a virus or viral vector. Transformation involves the uptake of free DNA from the environment through the cell membrane. Neither involves direct contact between bacterial cells or the formation of pili. (Choice D) Transposition involves the movement of a DNA sequence from one location to another within the genome (frequently with duplication of the transposon), which often results in genetic mutations due to incorrect DNA repair and/or insertional mutagenesis. (Choice E) Vertical gene transfer is the transmission of genes from parent to offspring. This occurs in bacteria when they divide by binary fission; it does not involve pili. Educational objective: Conjugation is a form of horizontal gene transfer that involves the formation of a pilus and the direct passing of genetic material from one bacterium to another. It is a common source of antibiotic resistance and environmental adaptation. The other forms of bacterial horizontal gene transfer are transduction (viral vector) and transformation (uptake of free DNA).

A 34-year-old man comes to the emergency department due to 2 weeks of slowly progressive global headache, malaise, and fevers. He has also had unintentional weight loss over the past several months. The patient has a history of injection drug use and was diagnosed with HIV 3 years ago. He has not been compliant with antiretroviral therapy. Temperature is 38.3 C (100.9 F). Examination shows white patches on the oropharyngeal mucosa and mild generalized lymphadenopathy. Brain imaging reveals no intracranial masses. India ink staining of cerebrospinal fluid demonstrates spherical yeast forms with thick capsules. Which of the following is the most likely site of primary infection in this patient? Intestine Lungs Nasopharynx Oral cavity Skin

Cryptococcus neoformans is a low-virulence yeast with a thick polysaccharide capsule. The organism resides in soil contaminated by bird (eg, pigeon) droppings, and human exposure primarily occurs due to inhalation into the lungs. Alveolar macrophages ingest the yeast, travel to draining lymph nodes, and activate CD4 T-lymphocytes that subsequently promote granuloma formation and enhanced destruction of the pathogen within phagolysosomes. Although a minority of patients clear the organism, most individuals develop life-long latent infection, similar to tuberculosis; those who are severely immunocompromised (eg, AIDS with CD4 count <100/mm3) are at high risk for reactivation. Cryptococcal meningitis primarily occurs when C neoformans reactivates from a latent pulmonary focus, spreads through the circulation, and penetrates the endothelial cells of the cerebral capillaries. Replication within the CNS leads to slowly worsening headache, fever, and confusion. Visualization of the organism (ie, round/oval yeast with a thick capsule) on cerebrospinal fluid samples using India ink stain confirms the diagnosis. Polysaccharide antigen (cryptococcal antigen) testing can also be helpful. (Choice A) Ingestion of C neoformans in contaminated food can lead to infection. However, this route is much less common than inhalation into the lungs. (Choice C) Acute bacterial meningitis is caused primarily by pathogens (eg, Streptococcus pneumoniae, Neisseria meningitidis, Haemophilus influenzae) that colonize the nasopharynx. Manifestations of acute bacterial meningitis usually include the rapid onset fever, neck stiffness, and altered mental state. Cerebrospinal fluid Gram stain will be positive. (Choice D) Patients who have untreated AIDS often develop multiple opportunistic infections; the white patches on this patient's oral mucosa likely indicate oropharyngeal candidiasis. Superficial Candida infections rarely cause invasive disease. In addition, Candida does not have a thick polysaccharide capsule and is not seen on India ink stain. (Choice E) Skin bacteria (eg, Staphylococcus aureus, Streptococcus pyogenes) can cause systemic infection following skin breakdown. Systemic infection from skin fungus (eg, Candida) following skin breakdown is uncommon. This patient's encapsulated yeast on India ink stain indicates Cryptococcus, making inhalation the most likely route of infection. Educational objective: Cryptococcus neoformans is a yeast with a thick polysaccharide capsule that is found primarily in soil contaminated by bird droppings. Inhalation of the pathogen into the lungs leads to asymptomatic primary infection usually followed by life-long latent infection. However, immunocompromised (eg, advanced AIDS) patients are at high risk for reactivation with spread to the CNS.

A 35-year-old man who received a kidney transplant one year ago comes to the emergency department with pleuritic chest pain and cough. He is found to have a low-grade fever. Chest x-ray reveals an infiltrate in the lower lobe of the right lung. The patient is started on broad-spectrum antibiotics, including vancomycin, ceftriaxone, and azithromycin. Despite these measures, his condition worsens over the next several days. Bronchoscopy with bronchoalveolar lavage is performed. Mucicarmine staining of his bronchoalveolar fluid reveals the following on light microscopy: ID=117 red circles with black dots, review Which of the following is the most likely cause of this patient's condition? Aspergillus fumigatus Blastomyces dermatitids Candida albicans Coccidiodies immitis Cryptococcus neoformans Histoplasma capsulatum Rhizopus species

Cryptococcus neoformans is a yeast that has an antiphagocytic polysaccharide capsule, a major virulence factor. The polysaccharide capsule appears as a clear, unstained zone with India ink and stains red with mucicarmine (as seen in this patient). Methenamine silver stain can identify the yeast forms, seen in tissue as round cells with narrow-based buds. C neoformans usually affects immunocompromised patients (eg, kidney transplantation patients on chronic immunosuppression). It is a neurotropic fungus that is transmitted via the respiratory route and most commonly presents with subacute or chronic meningoencephalitis. Cryptococcal lung disease can occasionally cause pneumonia-like symptoms, including cough with scant sputum production, pleuritic chest pain, dyspnea, and hemoptysis. Chest x-ray findings are nonspecific (eg, infiltrates, nodules). Diagnosis is usually confirmed by identifying Cryptococcus in sputum, bronchoalveolar washings, or tissue samples. (Choice A) Aspergillus fumigatus can cause pulmonary disease and invasive aspergillosis, most commonly in immunocompromised patients. However, microscopy typically shows septate hyphae. (Choice B) Blastomyces dermatitidis can cause both lung disease and disseminated mycosis. Its microscopic appearance in tissue is that of a round yeast with broad-based budding and a thick, doubly reflective wall. (Choice C) Candida albicans most commonly causes oropharyngeal, mucocutaneous, and esophageal disease. It rarely causes pneumonia. Microscopy typically shows budding yeast with pseudo-hyphae. (Choice D) Coccidioides immitis causes lung disease in immunocompetent individuals and disseminated mycosis in immunocompromised patients. In tissue samples, it appears as large, irregularly sized, thick-walled spherules that contain small, round endospores. (Choice F) Histoplasma capsulatum is a dimorphic fungus that causes tuberculosis-like pulmonary disease. It can also cause disseminated mycosis in immunocompromised patients. It is found intracellularly in tissue (within macrophages), appearing as small, ovoid, and budding yeast cells. (Choice G) Rhizopus species typically causes rhino-orbito-cerebral infection but can cause pulmonary disease, predominantly in immunocompromised patients. Histology typically shows broad hyphae with irregular branching and rare septations. Educational objective: Cryptococcus neoformans is the only pathogenic fungus that has a polysaccharide capsule. The capsule appears red on mucicarmine stain and as a clear unstained zone with India ink.

A 54-year-old man with HIV is part of a large clinical study. He was diagnosed with HIV-1 infection 5 years ago. Blood samples are obtained periodically to perform sequencing analysis of the viral genome. Current testing reveals a large number of isolates with mutation in the pol gene compared to the previous analyses. This finding is most closely associated with which of the following? Evasion of humoral immune response Inconsistent use of antiretroviral therapy Ineffective CD8 cytotoxicity Prevention of CD4 cell apoptosis Reduction of viral replication

HIV reverse transcriptase does not have a proofreading mechanism. This allows the virus to rapidly accumulate mutations (~1 nucleotide base pair per replication cycle) that may make it resistant to antiretroviral monotherapy. However, the consistent use of combination antiretroviral therapy, which contains medications that target multiple steps in the HIV replication cycle, effectively prevents the development of resistant strains. This patient was found to have a large number of viral isolates with new mutations to the pol gene, which encodes for the major HIV enzymes protease, reverse transcriptase, and integrase. Most first-line antiretroviral medications, including protease inhibitors, reverse transcriptase inhibitors, and integrase strand transfer inhibitors, target these enzymes. The emergence of a new predominant strain with mutation to the pol gene suggests that this patient has become resistant to his antiretroviral therapy. Nearly all cases of acquired antiretroviral resistance are due to inconsistent use (transmitted resistance can also occur when an individual is inoculated with a resistant strain). This patient will likely need a new antiretroviral regimen and should be encouraged to take it as prescribed. (Choice A) HIV infection results in the generation of antibodies that can reduce free virus in the circulatory system but cannot eliminate the infection due to viral integration into the host genome. The primary target of neutralizing antibodies against HIV is the surface glycoprotein GP120, which is encoded by the env gene, not the pol gene. (Choice C) The HIV nef gene encodes for a protein that downregulates major histocompatibility complex class I proteins, reducing the ability of cytotoxic T cells to recognize host cells infected with HIV. Pol gene mutations are unlikely to directly influence the ability of CD8 cells to respond to the infection. (Choice D) HIV infection triggers CD4 cell apoptosis through many mechanisms, including chronic immunologic stimulation, activation of the CD4 receptor by envelope glycoproteins (during viral attachment), and increased production of cytotoxic ligands by immune cells. Mutations to the pol gene are unlikely to result in a reduction in CD4 apoptosis. (Choice E) Viral replication rate is generally positively correlated with the mutation rate. Educational objective: HIV pol gene mutations are responsible for acquired resistance to reverse transcriptase inhibitors, protease inhibitors, and integrase strand transfer inhibitors. Mutations of the env gene enable escape from host-neutralizing antibodies.

A 54-year-old man with HIV comes to the emergency department due to nonproductive cough, low-grade fevers, and worsening fatigue for the past month. He has also lost 3.6 kg (8 lb) over the same period. The patient does not take antiretroviral therapy consistently and has not followed up with his primary care physician for the past several months. Physical examination reveals hepatosplenomegaly. Laboratory testing shows pancytopenia and elevated liver aminotransferases. Bone marrow aspiration is performed, and light microscopy of the specimen is shown below. Organism within macrophage (ID=267) Which of the following is the most likely cause of this patient's current condition? Aspergillus fumigatus Blastomyces dermatitidis Candida albicans Coccidioides immitis Cryptococcus neoformans Histoplasma capsulatum

Light microscopy reveals several small, ovoid bodies within a macrophage, raising strong suspicion for Histoplasma capsulatum. This dimorphic fungus is transmitted by inhalation, converts to a yeast form in the lungs, and replicates within the phagosome of macrophages. Patients with intact immunity usually have an asymptomatic or self-limited pulmonary infection that is contained over time by the cell-mediated immune response. However, those with impaired immunity (eg, advanced AIDS) are at risk for dissemination through the reticuloendothelial system due to the organism's affinity for mononuclear phagocytic cells. Common manifestations include hepatosplenomegaly, lymphadenopathy, and pancytopenia. Ulcerated lesions on the tongue are also very characteristic of disseminated histoplasmosis. Disseminated infection is often diagnosed when histopathology shows oval or round yeasts within macrophages. Culture on Sabouraud agar grows hyphae (Histoplasma is a mold at cooler temperatures). Histoplasma antigen in blood and urine can also be detected by immunoassay. (Choice A) Aspergillus fumigatus is a monomorphic fungus with only a mold form. It is seen in tissue as septate hyphae with acute-angle V-shaped branching. (Choices B and D) Both Blastomyces and Coccidioides are dimorphic fungi that are inhaled into the lungs and replicate within macrophages. However, Blastomyces is a large yeast with thick walls and broad-based buds that disseminates primarily to the skin (not the reticuloendothelial system), and Coccidioides immitis appears as large, thick-walled spherules containing endospores. (Choice C) Candida albicans is an ovoid yeast that forms pseudohyphae. Mucocutaneous Candida infections (eg, thrush) are common in patients with HIV, but invasive infections (eg, pneumonia) are rare. In addition, Candida is an extracellular pathogen. (Choice E) Cryptococcus neoformans is a large yeast with a thick polysaccharide capsule. It can replicate within macrophages and cause pulmonary infection in patients with advanced AIDS. However, dissemination usually leads to meningoencephalitis (eg, slowly worsening headache, confusion, neurologic symptoms), not hepatosplenomegaly and pancytopenia. Educational objective: Histoplasma capsulatum is a dimorphic fungus that exists as a small, ovoid yeast at tissue temperatures. It replicates within macrophages and spreads through the lymphatic and reticuloendothelial system. Immunocompetent patients usually have an asymptomatic pulmonary infection, but those who are immunocompromised (eg, advanced AIDS) can develop disseminated disease to the liver, spleen, and bone marrow.

A 32-year-old woman comes to the emergency department due to 4 weeks of worsening cough, shortness of breath, and low-grade fevers. She also has had an unintentional weight loss of 4.5 kg (10 lb) over the past 3 months. Temperature is 37.1 C (98.8 F), blood pressure is 120/70 mm Hg, pulse is 102/min, and respirations are 22/min. Pulse oximetry shows 86% on ambient air. Lung auscultation reveals bilateral crackles. There are no heart murmurs, and jugular venous pressure is normal. Chest x-ray reveals diffuse interstitial infiltrates. The patient undergoes bronchoscopy; methenamine silver staining of the bronchoalveolar lavage specimen is shown in the image. Crushed ping pong balls (ID=14840) Which of the following is the most appropriate pharmacotherapy for this patient's current condition? Caspofungin Ceftriaxone and azithromycin Doxycycline Ganciclovir Trimethoprim-sulfamethoxazole Vancomycin

Pneumocystis pneumonia is an AIDS-defining illness that is often seen in patients who were previously unaware of HIV infection. The underlying pathogen is an atypical fungus called Pneumocystis jirovecii. Healthy patients rapidly clear the inhaled organisms, but those with impaired cell-mediated immunity are at risk for a noninvasive infection that fills the alveoli and leads to slowly worsening pulmonary symptoms (eg, dry cough, shortness of breath), hypoxia, and a diffuse, bilateral interstitial infiltrate. Fever and weight loss are also usually present. P jirovecii cannot be cultured; therefore, definitive diagnosis requires identification of the organism in respiratory secretions. Methenamine silver stain is frequently used to identify the cell wall of the pathogen, which often appears as a crescent, a crushed ping-pong ball, or a circular ring around a clear center. First-line treatment with trimethoprim-sulfamethoxazole is usually curative. (Choice A) Invasive pulmonary aspergillosis may be treated with caspofungin and is most common in patients with severe immunosuppression (eg, AIDS, neutropenia). It usually presents with fever, cough, chest pain, dyspnea, and hemoptysis. Silver stain can be used to diagnose aspergillosis, but numerous hyphae (not cystic organisms) will be seen. (Choice B) Ceftriaxone and azithromycin are often used to treat community-acquired bacterial pneumonia, which usually presents acutely with productive cough, fever, and shortness of breath. Chest x-ray typically reveals a lobar infiltrate rather than a diffuse interstitial infiltrate. (Choice C) Doxycycline can be used to treat Mycoplasma pneumonia, which often presents with several weeks of incessant cough and chest x-ray showing interstitial infiltrates. However, significant hypoxia is uncommon, and M pneumoniae is not identified on silver stain. (Choice D) Patients with advanced AIDS are at risk for cytomegalovirus (CMV) pneumonia, which is treated with ganciclovir and can cause fever, cough, dyspnea, and diffuse pulmonary infiltrates. The presence of "owl's eye" inclusion bodies on histopathology strongly supports a CMV infection. (Choice F) Vancomycin inhibits cell wall synthesis and is primarily used for severe or drug-resistant, Gram-positive bacterial infections such as methicillin-resistant Staphylococcus aureus. Educational objective: Pneumocystis pneumonia is common in patients with advanced HIV and usually presents with slowly worsening cough and dyspnea, hypoxia, and bilateral interstitial infiltrates. Diagnosis requires visualizing the organism in respiratory secretions using special stains (eg, methenamine silver stain). First-line treatment is trimethoprim-sulfamethoxazole.

A 6-hour-old boy is evaluated in the newborn nursery due to temperature instability, grunting, and lethargy. The patient was born at full term to a 21-year-old woman who came to the emergency department in active labor with ruptured membranes; she had not received adequate prenatal care. On physical examination, the boy is hypothermic, tachycardic, and tachypneic, and he has mottled skin. Blood samples are obtained, and empiric antibiotics are initiated for the treatment of sepsis. Cultures on blood agar grow bacterial colonies with a narrow zone of clear hemolysis. When these bacteria are grown in the presence of Staphylococcus aureus, the zone of hemolysis is enhanced around the staphylococcal colonies. Which of the following is the most likely pathogen infecting this patient? Escherichia coli Enterococcus faecalis Staphylococcus epidermidis Streptococcus agalactiae Streptococcus pneumoniae

Streptococcus agalactiae (group B Streptococcus [GBS]) is often a normal commensal of the genitourinary and gastrointestinal tracts of pregnant women. Those who are colonized and do not receive intrapartum antibiotic prophylaxis can transmit the bacteria to the fetus during passage through the vaginal canal, leading to neonatal sepsis (eg, irritability, lethargy, grunting, tachypnea, temperature instability, hypotension) within hours of birth. GBS is a leading cause of early neonatal sepsis and can be identified by the following laboratory features: Narrow zone of beta-hemolysis (clear) when plated on blood agar. Production of CAMP factor, a phospholipase that enhances the activity of beta-hemolysins secreted by certain strains of Staphylococcus aureus. Testing for CAMP factor (CAMP test) can confirm the presence of GBS; this test plates suspected GBS colonies perpendicular to an appropriate S aureus strain and evaluates for enhanced areas of hemolysis (arrow-shaped zones in the image above). ID=15578 GBS is also catalase-negative (as are all streptococci), contains the Lancefield group B antigen in its cell wall, and hydrolyzes hippurate. (Choice A) Escherichia coli, a gram-negative rod, is a common cause of early neonatal sepsis. Although E coli generates beta-hemolysins, it does not produce CAMP factor; therefore, enhancement of S aureus hemolysis does not occur. (Choice B) Enterococcus faecalis, a gram-positive cocci, is a common cause of sepsis in preterm neonates but is rare in those born at term. In addition, it does not hemolyze red blood cells (gamma hemolytic) or generate camp factor. (Choice C) Staphylococcus epidermidis, a gram-positive coccus that grows in clusters, is a common skin commensal that does not cause hemolysis when plated on blood agar. (Choice E) Streptococcus pneumoniae, a lancet-shaped, gram-positive bacterium, is alpha-hemolytic (partial, green hemolysis) in aerobic conditions and beta-hemolytic in anaerobic conditions. It does not generate CAMP factor. Educational objective: Streptococcus agalactiae (group B Streptococcus), one of the most common causes of early neonatal sepsis, is typically identified by the presence of a narrow zone of beta-hemolysis when plated on blood agar. It is also identified by the production of CAMP factor, which enhances beta-hemolysis by particular strains of Staphylococcus aureus.

A 14-year-old girl comes to the physician with dark urine and facial puffiness. About 4 weeks before presentation, she had pustular skin lesions that broke down over a few days to form thick scabs in the lower extremities. Microscopic examination of the urine sediment shows red blood cell casts. The organism responsible for this patient's symptoms would most likely demonstrate which of the following? Bile solubility Catalase positivity Growth in hypertonic saline Optochin sensitivity Pyrridonyl arylamidase positivity

The patient had impetigo, a skin infection caused by Staphylococcus aureus and less commonly by Streptococcus pyogenes (Group A streptococci [GAS]). Her facial puffiness (likely due to sodium and water retention and possibly proteinuria) and hematuria (likely due to nephritis) suggest post-streptococcal glomerulonephritis. Staphylococcal infection is not complicated by glomerulonephritis, so S pyogenes is the most likely causative organism of her symptoms. Nonsuppurative complications of GAS infections include acute rheumatic fever (ARF) and glomerulonephritis, thought to be due to the formation of different anti-streptococcal antibodies that cross-react with heart tissue or result in immune complex deposition in the kidney. Glomerulonephritis is associated with prior streptococcal pharyngeal or skin infection (eg, cellulitis, impetigo, erysipelas); ARF is associated with prior streptococcal pharyngitis but not skin infection. Previous streptococcal infection is evidenced by elevated streptococcal antibodies (anti-streptolysin, anti-DNaseB, and anti-hyaluronidase). S pyogenes are catalase-negative, β-hemolytic, gram-positive cocci in chains. S pyogenes is unique among β-hemolytic streptococci as it is susceptible to bacitracin. Because the bacitracin test is not very specific for S pyogenes, it has been replaced in many laboratories by the pyrrolidonyl arylamidase (PYR) test; S pyogenes is PYR-positive. (Choices A and D) Streptococcus pneumoniae is bile-soluble and optochin-susceptible. These tests are used to differentiate S pneumoniae from other α-hemolytic streptococci such as those in the viridans group. (Choice B) Streptococci are catalase-negative, staphylococci are catalase-positive. (Choice C) Enterococci, not GAS, can grow in broth containing hypertonic (6.5%) saline. Both S pyogenes and enterococci are PYR-positive. Educational objective: Glomerulonephritis is associated with prior streptococcal pharyngeal or skin infection; acute rheumatic fever is associated with prior streptococcal pharyngitis but not skin infection. Streptococci are catalase-negative, staphylococci are catalase-positive. Streptococcus pyogenes (Group A Streptococcus) is β-hemolytic, bacitracin-susceptible, and pyrrolidonyl arylamidase-positive. Streptococcus pneumoniae is bile-soluble and optochin-susceptible.

A 12-year-old boy is brought to the emergency department due to a skin infection. Temperature is 38.4 C (101.1 F). Physical examination shows an area of erythema, warmth, and tenderness on his right distal leg. Laboratory results are notable for leukocytosis. The patient is started on intravenous nafcillin. Two days later, he shows limited response to antibiotic therapy. Light microscopy of pus obtained from the site shows gram-positive cocci in clusters, and sensitivity testing demonstrates little response to nafcillin but good response to vancomycin. Which of the following is the most likely explanation for nafcillin treatment failure in this patient? Active drug transport out of the cell Enzymatic degradation of the drug Mutation in DNA gyrase Mutation in RNA polymerase Poor interaction with binding proteins

The presence of gram-positive cocci in clusters that do not respond to nafcillin but show sensitivity to vancomycin suggests infection with methicillin-resistant Staphylococcus aureus (MRSA). S aureus is a common cause of skin infections in the United States. Most skin infections are minor, but S aureus can cause serious infections (eg, surgical wound infections, bloodstream infections in the presence of central intravenous catheters, pneumonia). S aureus strains that are resistant to oxacillin, nafcillin, and methicillin have been historically termed MRSA, but they are also resistant to all β-lactam agents, including penicillin, cephalosporins (except ceftaroline, a fifth-generation cephalosporin), and carbapenems. Methicillin (nafcillin) resistance is typically mediated by alterations in the structure of penicillin-binding proteins (PBP), the enzymes involved in cell wall synthesis. Altered PBPs, especially PBP2a, have greatly reduced affinity for β-lactam antimicrobial agents (except ceftaroline). (Choice A) Resistance to tetracycline and sulfonamides is due at least in part to a decrease in the levels of drug accumulation due to decreased uptake and/or increased efflux. (Choice B) Approximately 95% of S aureus isolates in the United States produce penicillinase, which causes penicillin resistance by breaking penicillin's β-lactam ring. However, nafcillin is penicillinase-stable (along with oxacillin and methicillin). (Choices C and D) Mutations in DNA gyrase cause resistance to quinolone antibiotics. Mutations in RNA polymerase cause resistance to rifampin. Educational objective: Methicillin-resistant Staphylococcus aureus is resistant to all β-lactam antibiotics, including beta-lactamase-resistant antibiotics, as it has an altered penicillin-binding protein that does not bind β-lactams effectively.

A 19-year-old man comes to the office with an ulcer on his penis. The patient first noticed the lesion 3 days ago. His temperature is 37.1 C (98.8 F). Physical examination shows an indurated and painless ulcer near the glans penis, with no surrounding erythema and no inguinal lymphadenopathy. The patient is a college student. He has no significant past medical history and takes no medications. He has no known drug allergies. The first-line treatment for this patient has structural similarities with which of the following? D-alanine-D-alanine D-glutamic acid-D-glutamic acid Folic acid L-alanine-D-glutamine N-acetylglucosamine N-acetylmuramic acid

This patient likely has a chancre due to primary syphilis, a sexually transmitted infection caused by the spirochete Treponema pallidum. Spirochetes are universally sensitive to penicillin because they are all gram-negative organisms with a peptidoglycan cell wall. Penicillins work by covalently binding to and inhibiting transpeptidase. This enzyme catalyzes the final cross-linking step in peptidoglycan cell wall formation: the joining of the amino acid in the third position of a peptidoglycan molecule to the terminal D-alanine-D-alanine of another peptidoglycan molecule. The structural similarity of penicillins to D-alanine-D-alanine likely facilitates their binding to transpeptidase. When transpeptidase is inhibited, cell wall synthesis ceases and cell wall degradation by bacterial autolysins proceeds unchecked. The weakened cell wall integrity causes osmotic lysis of the bacterium. Vancomycin, a glycopeptide antibiotic, directly binds to the D-alanine-D-alanine residues, preventing the incorporation of new subunits into the cell wall. Vancomycin acts at an earlier stage of cell wall formation than do the penicillins. (Choices B and D) D-glutamic acid-D-glutamic acid and L-alanine-D-glutamine are not sequences targeted by transpeptidase. Penicillin is not a structural analog of either molecule. (Choice C) Sulfonamide antibiotics are antimetabolites that compete with para-aminobenzoic acid for incorporation into folic acid. Trimethoprim blocks another step in folic acid synthesis by inhibiting the enzyme dihydrofolate reductase. (Choices E and F) Peptidoglycan precursor molecules consist of a disaccharide composed of N-acetylmuramic acid (MurNAc) and N-acetylglucosamine (GlcNAc). Educational objective: Penicillins, structurally similar to D-alanine-D-alanine, inhibit transpeptidase by binding covalently to its active site. The result is failed synthesis of the bacterial peptidoglycan cell wall.

A 22-year-old woman comes to the emergency department with fevers and arthralgias. She recently had unprotected sexual intercourse with a new male partner. Her temperature is 38.4 C (101.1 F), blood pressure is 118/76 mm Hg, pulse is 102/min, and respirations are 16/min. Examination shows several pustules on the dorsal aspects of her forearms. Her right wrist and ankle are tender on palpation. Gram-negative bacteria isolated from this patient's blood produce an enzyme that splits the IgA molecule at the hinge region. Which of the following is likely to be the most important role of this bacterial enzyme in the course of the infection? It facilitates mucosal adherence of bacteria It impairs complement-mediated cell lysis It impairs opsonization and phagocytosis It impairs secondary immune response generation It promotes intracellular survival

This patient likely has disseminated gonorrheal infection (triad of arthritis, dermatitis, and tenosynovitis in a sexually active woman) due to Neisseria gonorrhoeae. In mucosal regions as well as in secretions, IgA exists in its secretory form, composed of an IgA dimer (joined by a peptide J chain) and a peptide secretory component. IgA antibodies usually bind to pili and other membrane proteins involved in bacterial adherence to mucosa, thus inhibiting mucosal colonization by the microorganism. Certain bacteria (eg, N gonorrhoeae, N meningitidis, Streptococcus pneumoniae, Haemophilus influenzae) produce IgA proteases that cleave IgA at its hinge region (yielding Fab and compromised Fc fragments), thus decreasing its effectiveness. This facilitates bacterial adherence to mucosa (possibly due to easier bacterial access to mucosal surface or immune disguise by binding to released Fab fragments, among others). (Choice B) IgA is a weak activator of the classic complement pathway. Impairment of complement-mediated cell lysis can be accomplished by staphylococcal protein A, which binds the Fc portion of IgG, preventing complement fixation. (Choice C) Although IgA Fc-mediated functions such as opsonization are compromised by IgA protease, secretory IgA is a poor opsonin. Impairment of opsonization and phagocytosis is most frequently accomplished by encapsulated bacteria such as S pneumoniae, N meningitidis, and H influenzae. (Choice D) IgA does not play a role in the induction of memory lymphocytes necessary for mounting a secondary immune response (which requires the formation of memory B and T lymphocytes after initial antigen exposure). (Choice E) Promotion of intracellular survival is accomplished by inhibition of phagosome-lysosome fusion or resistance to lysosomal enzymes. These properties are expressed by Mycobacterium tuberculosis, M leprae, Legionella, and Ehrlichia. Educational objective: IgA protease is produced by Neisseria meningitidis, N gonorrhoeae, Streptococcus pneumoniae, and Haemophilus influenzae. This enzyme cleaves secretory IgA at its hinge region, rendering it ineffective. Secretory IgA exists on mucosal surfaces and in secretions and acts to bind and inhibit the action of pili as well as other cell surface antigens that normally mediate mucosal adherence and subsequent penetration.

A 24-year-old woman comes to the emergency department due to fevers and chills over the last 24 hours. Three weeks ago, she underwent a partial bowel resection for Crohn disease without complications. Since the surgery, the patient has had difficulty eating and has a peripherally inserted central venous catheter in place through which she is receiving parenteral nutrition. Her temperature is 38.2 C (101 F), blood pressure is 100/60 mm Hg, and pulse is 114/min. The surgical wound healed well, and her abdomen is soft and nontender. The catheter site on the left arm is mildly tender but shows no erythema. Chest x-ray reveals a properly positioned central venous line and is otherwise unremarkable. Admission blood cultures subsequently return positive for yeast. The organism responsible for this patient's bloodstream infection is most likely to exhibit which of the following characteristics? Broad-based budding morphology Endemic to southwestern United States Positive India ink stain Pseudohyphae with blastoconidia Right-angle branching hyphae

This patient who has a central venous catheter and is receiving parenteral nutrition has sepsis (fever, tachycardia) with blood cultures growing yeast. Patients receiving parenteral nutrition (through a central venous catheter) are at high risk for candidemia (Candida in the bloodstream). Candida can colonize the catheter, and the lipid emulsion in the parenteral nutrition solution is thought to promote the growth of some Candida species. The morphology of Candida includes branching pseudohyphae with blastoconidia (budding cells). Most species are susceptible to echinocandins, and C albicans is nearly always susceptible to fluconazole. (Choice A) Broad-based budding is characteristic of Blastomyces. This organism can cause pulmonary infection and occasional dissemination, but bloodstream infection in the setting of a vascular catheter is unlikely. (Choice B) Coccidioides is a dimorphic fungus endemic to the southwestern United States. This organism is inhaled and can cause pulmonary infection with occasional dissemination, particularly among immunocompromised patients. (Choice C) Cryptococcus is a yeast with a thick capsule that does not absorb India ink, resulting in a clear halo that can aid in the diagnosis of cryptococcal infection. The most common cryptococcal infection is meningitis, particularly among immunocompromised patients. Fungemia in the setting of a vascular catheter is uncommon. (Choice E) Right-angle branching hyphae are characteristic of mucormycosis, a mold infection primarily due to Mucorales or Rhizopus species. These infections can cause invasive disease in immunocompromised patients, including angioinvasive infections, but the organisms do not grow in blood cultures and are not associated with intravascular catheters. Educational objective: The presence of a central vascular catheter and receipt of parenteral nutrition are risk factors for candidemia. Candida display a morphology of branching pseudohyphae with blastoconidia.

A 3-year-old boy is brought to the clinic due to left ear pain over the past 3 days. Last week, the patient developed a runny nose and cough, which resolved 2 days prior to the development of ear pain. He has no chronic medical conditions. Temperature is 38.4 C (101.1 F). There is purulent drainage from the left ear. Otoscopic examination of the left ear reveals a normal-appearing external canal and an erythematous tympanic membrane with purulent fluid pooling within the distal external canal. Which of the following organisms is the most likely cause of this patient's diagnosis? Klebsiella pneumoniae Moraxella catarrhalis Proteus mirabilis Pseudomonas aeruginosa Staphyloccocus aureus Streptococcus pneumoniae

This patient with an erythematous tympanic membrane and purulent otorrhea has acute otitis media (AOM) complicated by tympanic membrane perforation, which can be obscured due to drainage. Acute middle ear infections occur most commonly in young children (age 6 months to 2 years) and are often due to Streptococcus pneumoniae. A viral upper respiratory infection (URI) typically precedes the development of AOM because it causes inflammatory edema of the respiratory mucosa that can lead to obstruction of the eustachian tube. The eustachian tube normally opens to equalize middle ear pressure and drain secretions. Therefore, obstruction leads to the accumulation of secretions, providing a rich medium for bacterial growth. Bacterial colonizers of the upper respiratory tract (eg, S pneumoniae, Haemophilus influenzae, Moraxella catarrhalis) then proliferate in the middle ear secretions. This overgrowth of bacteria and consequent immune response result in AOM. Suppurative fluid accumulation increases middle ear pressure and causes the tympanic membrane to become inflamed and bulge outward. If the eustachian tube remains obstructed, this increased pressure can rupture the tympanic membrane. (Choice A) Although Klebsiella pneumoniae can be a colonizer of the mouth and throat, it is unlikely to cause AOM in otherwise healthy children. Instead, it more often causes respiratory diseases in older, immunosuppressed patients (eg, pneumonia in patients with alcohol overuse). (Choice B) Moraxella catarrhalis is also an upper respiratory colonizer that can cause AOM. However, infection with it is less common than with S pneumoniae. (Choice C) Proteus mirabilis classically causes genitourinary infections; its ability to split urea alkalinizes the urine, increasing the formation of kidney stones. (Choice D) Pseudomonas aeruginosa is more commonly implicated in otitis externa than AOM. (Choice E) Staphylococcus aureus is typically found as a colonizer of the skin rather than the respiratory tract. Therefore, it is more often implicated in skin and soft tissue infections, including otitis externa. Educational objective: Acute otitis media usually occurs after a viral upper respiratory infection causes obstruction of the eustachian tube. This obstruction leads to the accumulation of secretions, which encourages bacterial growth. The most common bacterial causes are respiratory colonizers, including Streptococcus pneumoniae, Haemophilus influenzae, and Moraxella catarrhalis

Review exhibit: 1949 A 16-year-old boy comes to the office due to 3 days of burning with urination and yellow penile discharge. The patient is sexually active with one female partner; they engage in vaginal intercourse and do not use condoms. He has no other medical conditions, takes no medications, and has no allergies. Urine nucleic acid amplification test results are pending, and urethral Gram stain is shown in the exhibit. Appropriate treatment of this patient's condition involves antibiotics from which of the following drug classes? Fluoroquinolone and macrolide Fluoroquinolone and third-generation cephalosporine Macrolide only Penicillin only Tetracycline and third-generation cephalosporine Third-generation cephalosporin only1949

This patient with dysuria and penile discharge likely has infectious urethritis due to sexually transmitted infection. Most cases are caused by Neisseria gonorrhoeae, Chlamydia trachomatis, or Mycoplasma genitalium. The underlying pathogen is typically identified with nucleic acid amplification testing (NAAT) of urine, vaginal, or endocervical swab. However, light microscopy with Gram stain can be performed to rapidly identify N gonorrhoeae while awaiting NAAT results, as follows: Gonococcal urethritis: leukocytes with intracellular gram-negative diplococci Nongonococcal urethritis: leukocytes with no visible organisms; because Chlamydia and Mycoplasma do not have sufficient peptidoglycan in the cell wall to take up Gram stain, they cannot be ruled in or out with microscopy Patients with gonococcal urethritis who have coinfection with C trachomatis or uncertain C trachomatis status (as in this patient whose NAAT results are pending) require a third-generation cephalosporin (eg, ceftriaxone) and a tetracycline (eg, doxycycline) to cover gonococcus and possible C trachomatis coinfection. Patients with gonococcal urethritis who have negative C trachomatis NAAT should be treated with third-generation cephalosporin (eg, ceftriaxone) alone (Choice F). Patients with nongonococcal urethritis should be treated with macrolide (eg, azithromycin) or tetracycline (eg, doxycycline) monotherapy (Choice C). (Choices A and B) N gonorrhoeae is largely resistant to fluoroquinolones (eg, ciprofloxacin, levofloxacin), so this class of medication is not recommended. (Choice D) Penicillin is the treatment of choice for syphilis. However, empiric treatment is not recommended unless a chancre is present. Other stages of syphilis should only be treated when serology (eg, rapid plasma reagin, VDRL) is positive or the organism is otherwise identified. Educational objective: Infectious urethritis in men is typically caused by Neisseria gonorrhoeae, Chlamydia trachomatis, or Mycoplasma genitalium. Diagnosis is generally made by nucleic acid amplification testing, but microscopy with Gram stain can be used to quickly diagnose gonococcal infection. Patients with gonorrhea and positive or uncertain Chlamydia status require ceftriaxone and doxycycline; ceftriaxone alone can be used in those who do not have Chlamydia coinfection.

A 76-year-old hospitalized woman is evaluated for persistent fevers. Four days ago, she was admitted to the hospital for sepsis, for which she has received broad-spectrum empiric intravenous antibiotic therapy. Her temperature is 38.9 C (102 F), blood pressure is 110/80 mm Hg, pulse is 98/min, and respirations are 18/min. Cardiac examination reveals a new diastolic murmur. Multiple blood cultures drawn at different times grow Enterococcus. Echocardiogram reveals mitral valve vegetation suggestive of endocarditis. An intravenous antibiotic is added to the patient's treatment regimen for synergy. Several days later, she develops tinnitus and hearing loss attributed to this antibiotic. The additional antibiotic most directly affects which of the following processes? Cell wall cross-linking DNA unwinding Folic acid synthesis Maintenance of transmembrane potential mRNA genetic code reading tRNA changing

This patient with enterococcal endocarditis likely received an aminoglycoside (AG) such as gentamicin, a ribosome-targeting antibiotic. AGs are associated with ototoxicity (hearing loss, tinnitus) and nephrotoxicity. mRNA is translated into amino acids (AAs) and proteins on ribosomes. The small ribosomal subunit (30S in prokaryotes, 40S in eukaryotes) binds mRNA, which contains genetic information serving as a reading frame, and tRNA, which serves as an AA carrier. The larger ribosomal subunit (50S in prokaryotes, 60S in eukaryotes) contains peptidyltransferase, which catalyzes peptide bond formation between AAs In prokaryotes, a unique mRNA sequence (Shine-Dalgarno) is located upstream from initiation codon AUG, which recognizes an N-formylmethionine-tRNA. This sequence allows 30S binding to mRNA and this N-formylmethionine-tRNA (forming the prokaryotic 30S initiation complex). The energy for this reaction (GTP hydrolysis) brings the 50S subunit into the initiation complex, resulting in a 70S ribosomal unit. AGs irreversibly bind to bacterial 30S but not eukaryotic 40S; this causes genetic code misreading and bacterial protein synthesis inhibition. AGs also appear to impact translocation, whereby an aminoacyl-tRNA is shifted from the ribosomal A site (after initial binding) to the P site (after AA incorporation into the peptide chain during elongation) to the E site (after AA cleaving from its tRNA). The initial N-formylmethionine-tRNA notably binds at the P site. (Choice A) AGs need to diffuse inside the cell to reach the ribosome. As a result, they are often prescribed synergistically in combination with antibiotics affecting cell wall synthesis. These include penicillins, which impact cell wall cross-linking, or vancomycin. Vancomycin is also associated with ototoxicity, but it is a glycopeptide antibiotic that acts by directly inhibiting peptidoglycan synthesis, not cell wall cross-linking (which occurs at a later stage of cell wall synthesis). Although Enterococcus is generally resistant to AGs, these drugs are routinely used as synergistic agents in combination with penicillins or vancomycin in patients with enterococcal endocarditis; such combinations lead to improved bacterial killing and are the standard of care for this condition. (Choices B and C) Fluoroquinolones act on DNA gyrase to prevent DNA unwinding. Trimethoprim-sulfamethoxazole affects folic acid synthesis. These antibiotics are not classically associated with ototoxicity and are not used synergistically for enterococcal endocarditis. (Choice D) Daptomycin disrupts the bacterial membrane by creating transmembrane channels that cause intracellular ion leakage and cellular membrane depolarization. Its main side effects are myopathy and rhabdomyolysis. (Choice F) tRNA charging refers to initial covalent bonding between tRNA and the corresponding AA, facilitated by aminoacyl-tRNA synthetase; inhibitors of this enzyme are potential antibiotics. Educational objective: Aminoglycosides inhibit genetic code reading and protein synthesis by binding to the prokaryotic 30S ribosomal subunit.

A 4-year-old boy is brought to the physician by his mother with decreased appetite, abdominal cramps, and diarrhea over the last 3 days. The stools were initially watery but have now become bloody. On physical examination, his temperature is 100.4 F (38 C). There are signs of dehydration and he has a mildly distended abdomen with diffuse pain elicited on palpation. Stool studies are positive for fecal leukocytes and occult blood. Fecal cultures grow non-lactose fermenting, gram-negative rods on MacConkey agar. The bacteria ferment glucose without gas production, do not generate hydrogen sulfide when grown on triple sugar iron agar, and cannot replicate at refrigeration temperatures. Which of the following bacterial factors is the most important during the pathogenesis of this patient's disease? Exotoxin production Intestinal colonization Mucosal invasion Proliferation in lymph nodes Survival in the circulation

This patient's fever, abdominal cramps, and bloody diarrhea (dysentery) are consistent with bacterial enteritis. His stool culture results are most specific for a Shigella species (Shigella sonnei being most common in the United States). Shigella is transmitted via the fecal-oral route and is never a component of the normal bacterial gut flora. Shigellosis is primarily a pediatric disease but also shows a predilection for men who have sex with men and adults in skilled nursing facilities. Shigella species are non-lactose fermenting organisms that produce acid (not gas) during glucose fermentation (in contrast to Escherichia coli). Furthermore, Shigella species are non-motile and do not produce H2S (in contrast to Salmonella). Mucosal invasion is the essential pathogenic mechanism for Shigella infection. Shigella invades the gastrointestinal mucosa, particularly via the M cells that overlie Peyer's patches. After cell entry, Shigella is able to lyse its containment vacuole and enter the cytosolic compartment. It then can induce apoptosis of the host cell and spread to adjacent cells via protrusions created through host-cell actin polymerization. Shigella invasion triggers a robust host inflammatory response that is largely mediated by neutrophils. (Choice A) Some strains of Shigella produce shiga toxin, which damages the intestinal mucosa by inactivating the 60S ribosomal subunit, halting cellular protein synthesis. However, shiga toxin plays a minor, nonessential role in the disease process, as nontoxigenic strains also cause shigellosis. (Choice B) Intestinal colonization with Shigella does not occur as the bacterium is highly pathogenic; transmission of as few as 10-100 organisms is sufficient to cause disease. If Shigella is isolated from a stool culture, an active infection is taking place. (Choice D) Proliferation in lymph nodes is not a characteristic of Shigella. In contrast, both Salmonella typhi and Yersinia enterocolitica (grows at low temperatures like Listeria) can gain access to the lymphatics and proliferate in the mesenteric lymph nodes. Y enterocolitica can cause inflammation and enlargement of the lymphoid tissue around the appendix and terminal ileum ("pseudoappendicitis"), leading to right lower quadrant pain that can be confused with acute appendicitis. (Choice E) Shigella infections rarely cause bacteremia, as the bacterium is readily phagocytosed and destroyed after entering the bloodstream. Enteric bacteremia is more likely to be caused by E coli, S typhi (typhoid fever), Klebsiella, and Proteus. Educational objective: Shigella is a non-motile, non-lactose fermenting organism that does not produce H2S when grown on triple sugar iron agar. Mucosal invasion of the M cells that overlie Peyer's patches is an essential pathogenic mechanism for Shigella infection. Shigella then escapes the phagosome and spreads laterally to other epithelial cells via actin polymerization.

A 50-year-old woman comes to the emergency department due to fever and abdominal pain. The patient has a history of type 2 diabetes mellitus. Her temperature is 38.3 C (101 F). Abdominal examination is notable for right upper quadrant tenderness to palpation that is worsened by deep inspiration. Laboratory studies show leukocytosis. An ultrasound of the abdomen reveals multiple gallstones and gallbladder wall thickening with pericholecystic fluid. Blood cultures are drawn, and surgical consultation is requested. The patient is initiated on intravenous piperacillin combined with tazobactam. Which of the following is the most likely reason to use this drug combination in this patient? Decrease bacterial inactivation of piperacillin Decrease renal clearance of piperacillin Enhance bacterial penetration of piperacillin Increase bioavailability of piperacillin Provide additive antimicrobial activity

This patient's fever, leukocytosis, right upper quadrant pain with positive Murphy sign, and ultrasound findings (eg, gallstones, wall thickening, pericholecystic fluid) indicate acute cholecystitis. Antibiotic therapy is often administered to prevent secondary infection (eg, Escherichia coli, Klebsiella) due to biliary stasis, particularly in patients who have diabetes mellitus or immunodeficiency. The penicillin family of antibiotics (eg, piperacillin) contains a beta-lactam ring, which inhibits peptidoglycan cross-linking and causes bacterial cell wall disruptions. Resistance is often due to beta-lactamase, a bacterial enzyme that hydrolyzes and inactivates the beta-lactam ring. The coadministration of a beta-lactamase inhibitor such as tazobactam prevents beta-lactamase producing organisms from inactivating penicillins, which extends their spectrum of activity (broadens the range of microbes that can be treated). Some penicillin-derivative antibiotics (eg, late-generation cephalosporins, carbapenems) have beta-lactam rings with additional chemical components, which make them partially or completely resistant to beta-lactamases. (Choice B) Probenecid, a uricosuric agent used for hyperuricemia with gout, blocks the renal tubular excretion of penicillins and cephalosporins and is sometimes used to increase their serum level or prolong their half-life. (Choice C) Aminoglycosides inhibit bacterial protein synthesis by blocking the 30S ribosomal subunit. Penicillins are often coadministered with aminoglycosides to aid in their entry into the cell (penicillins disrupt the cell wall, which allow aminoglycosides to penetrate the bacteria). (Choice D) Piperacillin is always administered intravenously so bioavailability is 100%. Tazobactam inhibits beta-lactamase; it does not alter piperacillin metabolism or excretion. (Choice E) Additive antimicrobial activity occurs when 2 different antibiotic classes are used in combination, such as a fluoroquinolone (DNA gyrase inhibitor) with vancomycin (cell wall synthesis inhibitor). Beta-lactamase inhibitors block an enzyme that destroys penicillins; they do not independently provide antimicrobial activity so they are always used in combination with penicillin-class agents. Educational objective: Beta-lactamase inhibitors (eg, clavulanic acid, sulbactam, tazobactam) prevent beta-lactamase from inactivating penicillin class drugs, which extends the spectrum of their activity.

A 54-year-old man is brought to the emergency department due to a week of progressive headaches and confusion. He was treated for viral esophagitis 6 months ago and Pneumocystis pneumonia 2 months ago. Temperature is 38 C (100.4 F), blood pressure is 142/86 mm Hg, and pulse is 64/min. The patient is lethargic and oriented to person and place only. CT scan of the head shows a mildly increased ventricle size but no other abnormalities. Lumbar puncture reveals an elevated opening pressure, and cerebrospinal fluid analysis shows moderately increased protein concentration and pleocytosis. The latex agglutination test is positive for soluble polysaccharide antigen. Light microscopy of this patient's cerebrospinal fluid is most likely to reveal which of the following? Budding yeast Germ tubes Nonseptate hyphae Spherules with endospores Sporangium

This patient's opportunistic infections (viral esophagitis, Pneumocystosis pneumonia) raise suspicion for untreated AIDS. He now has headache, fever, confusion, and polysaccharide antigens in the cerebrospinal fluid (CSF) indicating likely cryptococcal meningitis. This opportunistic infection is caused by Cryptococcus neoformans, a yeast with a thick polysaccharide capsule. Diagnosis is often made by identifying polysaccharide (cryptococcal) antigens in the CSF. Round or oval budding yeast can also be visualized with India ink stain. Most patients have elevated opening pressures on lumbar puncture due to increased vascular permeability secondary to inflammation and/or yeasts obstructing CSF outflow from the arachnoid villi. (Choice B) Candida albicans forms germ tubes (true hyphae) if incubated in 37 C (98.6 F) serum for 3 hours. This test helps differentiate C albicans from other Candida species. C albicans meningitis is extremely rare (even in patients with HIV) and would not be associated with a positive polysaccharide antigen. (Choice C) Mucor and Rhizopus species are characterized by nonseptate hyphae that branch at wide angles. These fungi cause severe infections of the paranasal sinuses (mucormycosis) in patients who are immunosuppressed (eg, diabetes mellitus). (Choice D) Coccidioides immitis is associated with the formation of spherules, which then rupture to release endospores. This fungus primarily causes community-acquired pneumonia in endemic areas (eg, Arizona desert); central nervous system infection is quite rare and would not be associated with a positive polysaccharide antigen. (Choice E) Mold fungi (eg, Rhizopus) form sporangia, which produce and contain spores. Cryptococcus is a yeast and does not reproduce with sporangia. Educational objective: Cryptococcus neoformans causes meningoencephalitis in patients with untreated AIDS. Diagnosis can be made by detecting the polysaccharide capsule in cerebrospinal fluid using the latex agglutination test. India ink staining of cerebrospinal fluid is also used for diagnosis and can demonstrate round or oval budding yeast.


Ensembles d'études connexes

Management of Patients with Oncologic Disorders

View Set

Wong Ch 16:Health Problems of School-Age Children and Adolescents

View Set

Facility Safety and Patron Surveillance

View Set

fundamental of bio 1 lect Quiz 2 wrong answers

View Set

contract law: PRI - PRELICENSING

View Set

Chapter 42: Ecosystems Study Guide

View Set